innovativeugcnet@gmail.com
+91-99-5344-4088

Scanner UGC- NET JRF 1st Paper

Share to your friend

Teaching Aptitude 

Previous Year Question Paper 2009 to June 2019

June 2019 – Main Paper (Commerce)

  1. A teacher intends to find out the learning difficulties of students. Which of the following tests can be used ?
    1. Summative Test
    2. Formative Test
    3. Performance Test
    4. Diagnostic Test
  2. Student-Centric Classroom is meant for
    1. Addressing the individual differences
    2. Reducing teacher-oriented lectures
    3. Recalling the previous knowledge
    4. Engaging the entire class
  3. What is the main objective of formative evaluation ?
    1. To promote students to the next class
    2. To enhance students’ learning
    3. To enhance co-operation in class
    4. To understand the learning difficulties
  4. Below are given two Columns. Column-A presents the methods of teaching and Column-B presents the focus of a method. Match the two Columns.
    Column-A (Methods of Teaching)
    (a) Collaborative Method
    (b) Co-operative Method
    (c) Constructivist Method
    (d) Team teaching
    Column-B(Focus of a Method)
    (i) When expertise of teachers are used in teaching
    (ii) When emphasis is given on students as creator of knowledge
    (iii) When students share different aspects of a learning activity
    (iv) When students help each other to learn
    Select your answer from the options given below :
    (a) (b) (c) (d)
    1. (i) (ii) (iii) (iv)
    2. (iii) (iv) (ii) (i)
    3. (iv) (iii) (ii) (i)
    4. (ii) (iv) (iii) (i)
  5. Is SWAYAM ?
    1. Non-Governmental Organisation
    2. Digital Programme to achieve the principles of education
    3. On-line platform
    4. Name of a web site

June 2019 – Main Paper (Management)

  1. Peer group interaction in a classroom helps in
    1. Concept understanding
    2. Realization of One’s misunderstanding
    3. Rapport Building
    4. Questioning
  2. Inclusive education implies
    1. Ensuring learning outcome of every child to be the same
    2. Including the disabled in the main stream
    3. Provides compulsory education for children below 14 years
    4. Ensuring that no child is left behind in education
  3. Listening is important for a teacher to
    1. address the requirement of the learner
    2. gain knowledge
    3. keep the learner in good humour
    4. evaluate the learner for the progress made
  4. Reflective Practice implies use of one’s
    1. Cognition
    2. Metacognition
    3. Metacognition on cognition
    4. Reinforcement of learning
  5. When the learning outcomes of students are ensured with employment of less resources and effort on the part of a teacher and more initiative for self-learning is evident, which of the following expression will describe the teacher most appropriately?
    1. Teacher is successful
    2. Teacher is effective
    3. Teacher is intelligent
    4. Teacher is practical

December 2018 – Main Paper (Commerce)

  1. Which of the following are considered as key behaviours of effective teaching as evident from researches ?
    (a) Structuring through comments made for organizing what is to come
    (b) Asking process or content based questions
    (c) Lesson clarity implying the extent, to which teacher’s presentation is clear to the class
    (d) Using student ideas and contributions while the teacher is presenting the subject.<
    (e) Instructions variety involving teacher’s variablility and flexibility during presentations
    (f) Student success rate implying the rate at which students understand and correctly complete the exercises.
    1. (c), (e) and (f)
    2. (a), (c) and (f)
    3. (b), (c) and (d)
    4. (a), (b) and (c)
  2. Which of the following specify the factors affecting teaching. For your answer select from the code give below:
    (a) Availability of teaching aids and their use
    (b) School-community linkages
    (c) Interest of parents in the school programmes
    (d) Subject-knowledge of the teacher
    (e) Frequency of organizing co-curricular activities
    (f) Verbal and non-verbal communication skills of the teacher
    Code :
    1. (d), (e) and (f)
    2. (a), (d) and (f)
    3. (b), (c) and (e)
    4. (a), (b) and (c)
  3. In the two sets given below. Set-I embodies the domain of ‘ teaching competencies’ while Set-II provides the specific competencies which make a teacher effective. Match the two sets and select appropriate code.
    Set-I (Domain of teaching competencies)
    (a) Personality and attitude related competencies
    (b) Behavioural competencies
    (c) Substantive competencies
    (d) Style related competencies
    Set-II (Specific competencies)
    (i) Dynamism and flexibility
    (ii) Subject and general knowledge
    (iii) Good physique and appearance
    (iv) Self efficacy and locus of control
    (v) Teaching and managing
    a   b    c    d
    1. (ii) (iii) (iv) (v)
    2. (iii) (i) (ii) (v)
    3. (iv) (v) (ii) (i)
    4. (i) (ii) (iii) (iv)
  4. In which of the modalities of teaching-learning arrangements there is more scope for critical and creative interchange ?
    (a) Training session<
    (b) Dialogic presentations involving teachers, students and the content
    (c) Instructional presentations involving scope for information processing
    (d) Conditioning to promote sensitivity
  5. Below are given the characteristic feature of formative and summative assessment procedures used in various teaching-learning, contexts.
    (a) It is used for judging learning standard.
    (b) It is used for improvement of capacity and quality
    (c) Its use is extended to performance appraisal of teachers, students and institutions.
    (d) Is is used during instructions.
    (e) Assessment is formal and post instructions in nature.
    (f) It is based on participation of both teacher and student.
    Which of the given codes relate to formative procedures ?
    Code :
    1. (b), (c) and (d)
    2. (a), (c) and (e)
    3. (b), (d) and (f)
    4. (a), (b) and (c)

December 2018 – Main Paper (Management)

  1. The construction of thought process, which includes remembering, problem solving and decision making, is called as :
    (1) Psychology
    (2) Andragogy
    (3) Cognitive development
    (4) Pedagogy
  2. The field of study dealing with methods and principles used in adult education is known :
    (a) Anthropology
    (b) Pedagogy
    (c) Gerontology
    (d) Andragogy
  3. The field of study of the theory and practice of self-determined learning that focuses on the importance of knowing how to learn is known as :
    (a) Andragogy
    (b) Pedagogy
    (c) Heutagogy
    (d)Epistemology
  4. Which of the following indicates the concept of Zone of Proximal Development (APD) as suggested by psychologist Lev Vygotsky ?
    (1) A learner’s brain develops faster in childhood.
    (2) Friends interaction is very important for proper development of child.
    (3) What a learner can do with help and without help.
    (4) A learner learns effectively when it is supported with real life, examples.
  5. Which among the following is the correct set of four fundamental principles of Rabindranath Tagore’s educational philosopy ?
    (a) Humanism, Commercialisation, Urbanisation, Hinduism
    (b) Naturalism, Humanism, Internationalism, Idealism
    (c) Globalisation, Localisation, Transformation, Capitalism
    (d) Socialism, Spiritualism, Tourism, Commercialism

July 2018

  1. Which of the following set of statements best describes the nature and objectives of teaching ?
    Indicate your answer by selecting from the code.
    (a) Teaching and learning are integrally related.
    (b) There is no difference between teaching and training.
    (c) Concern of all teaching is to ensure some kind of transformation in students.
    (d) All good teaching is formal in nature.
    (e) A teacher is a senior person.
    (f) Teaching is a social act whereas learning is a personal act.
    Code :
    (1)   (a), (b) and (d)                                                                  (2)   (b), (c) and (e)
    (3)   (a), (c) and (f)
    (4)   (d), (e) and (f)
  2. Which of the following learner characteristics is highly related to effectiveness of teaching ?
    (1)  Prior experience of the learner
    (2)  Educational status of the parents of the learner
    (3)  Peer groups of the learner
    (4)  Family size from which the learner comes.
  3. In the two sets given below Set – I indicates methods of teaching while Set – II provides the basic requirements for success/effectiveness. Match the two sets and indicate your answer by choosing from the code:
    Set -I (Method of teaching)
    (a)  Lecturing
    (b)  Discussion in groups
    (c)  Brainstorming
    (d) Programmed Instructional procedure
    Set – II (Basic requirements for success/effectiveness)
    (i) Small step presentation with feedback provided
    (ii)Production of large number of ideas
    (iii) Content delivery in a lucid languageCode :
    (iv) Use of teaching-aids
    (v)  Theme based interaction among participants
    (a)    (b)        (c)        (d)
    (1)  (i)         (ii)       (iii)       (iv)
    (2)  (ii)        (iii)       (iv)       (v)
    (3)  (iii)       (v)        (ii)        (i)
    (4)  (iv)       (ii)        (i)         (iii)
  4. From the list of evaluation procedures given below identify those which will be called ‘formative evaluation’
    Indicate your answer by choosing from the code :
    (a) A teacher awards grades to students after having transacted the course work.
    (b) During interaction with students in the classroom, the teacher provides corrective feedback.
    (c)  The teacher gives marks to students on a unit test.
    (d) The teacher clarifies the doubts of students in the class itself.
    (e) The overall performance of a students is reported to parents at every three months interval.
    (f) The learner’s motivation is raised by the teacher through a question-answer session.
    Code :
    (1)  (a),  (b) and (c)
    (2)        (b), (c) and (d)
    (3)  (a),  (c) and (e)
    (4)        (b), (d) and (f)
  5. Assertion (A) : All teaching should aim at ensuring learning.
    Reason (R) : All learning results from teaching.
    Choose the correct answer from the following code :
    (1)  Both (A) and (R) are true, and (R) is the correct explanation of (A).
    (2)  Both (A) and (R) are true, but (R) is not the correct explanation of (A).
    (3)  (A) is true, but (R) is false.
    (4)  (A) is false, but (R) is true.

November 2017

  1. Which of the following set of statements best represents the nature and objective of teaching and learning ?
    (a) Teaching is like selling and learning is like buying.
    (b) Teaching is a social act while learning is a personal act.
    (c) Teaching implies learning whereas learning does not imply teaching.
    (d) Teaching is a kind of delivery of knowledge while learning is like receiving it.
    (e) Teaching is an interaction and is triadic in nature whereas learning is an active engagement in a subject domain.
    Code :
    (1) (a), (d) and (e)
    (2) (b), (c) and (e)
    (3) (a), (b) and (c)
    (4) (a), (b) and (d)
  2. From the list given below identify the learner characteristics which would facilitate teaching- learning system to become effective. Choose the correct code to indicate your answer.
    (a) Prior experience of learner
    (b) Learner’s family lineage
    (c) Aptitude of the learner
    (d) Learner’s stage of development
    (e) Learner’s food habits and hobbies
    (f) Learner’s religious affiliation
    Code :
    (1) (a), (c) and (d)
    (2) (d), (e) and (f)
    (3) (a), (d) and (e)
    (4) (b), (c) and (f)
  3. Assertion (A) : All teaching implies learning.
    Reason (R) : Learning to be useful must be derived from teaching.
    Choose the correct answer from the following :
    (1) Both (A) and (R) are true and (R) is the correct explanation of (A).
    (2) Both (A) and (R) are true but (R) is not the correct explanation of (A).
    (3) (A) is true, but (R) is false.
    (4) (A) is false, but (R) is true.
  4. On the basis of summative tests, a teacher is interpreting his/her students, performance in terms of their wellness life style evident in behaviour. This will be called :
    (1) Formative testing
    (2) Continuous and comprehensive evaluation
    (3) Norm – referenced testing
    (4) Criterion – referenced testing
  5. Which one of the following is a key behaviour in effective teaching ?
    (1) Using student ideas and contribution
    (2) Structuring
    (3) Instructional variety
    (4) Questioning

January 2017

  1. Which of the following set of statements represents acceptable propositions in respect of
    teaching-learning relationships ? Choose the correct code to indicate your answer.
    (i) When students fail in a test, it is the teacher who fails.
    (ii) Every teaching must aim at ensuring learning.
    (iii) There can be teaching without learning taking place.
    (iv) There can be no learning without teaching.
    (v) A teacher teaches but learns also.
    (vi) Real learning implies rote learning.
    Codes :(1) (ii), (iii), (iv) and (v)
    (2) (i), (ii), (iii) and (v)
    (3) (iii), (iv), (v) and (vi)
    (4) (i), (ii), (v) and (vi)
  2. Assertion (A) : Learning is a life long process.
    Reason (R) : Learning to be useful must be linked with life processes. Choose the correct
    answer from the following code :
    (1) Both (A) and (R) are true and (R) is the correct explanation of (A).
    (2) Both (A) and (R) are true, but (R) is not the correct explanation of (A).
    (3) (A) is true, but (R) is false.
    (4) (A) is false, but (R) is true.
  3. Effectiveness of teaching has to be judged in terms of
    (1) Course coverage
    (2) Students’ interest
    (3) Learning outcomes of students
    (4) Use of teaching aids in the classroom
  4. In which teaching method learner’s participation is made optimal and proactive ?
    (1) Discussion method
    (2) Buzz session method
    (3) Brainstorming session method
    (4) Project method
  5. One of the most powerful factors affecting teaching effectiveness is related to the
    (1) Social system of the country
    (2) Economic status of the society
    (3) Prevailing political system
    (4) Educational system
  6. Assertion (A) : Formative evaluation tends to accelerate the pace of learning.
    Reason (R) : As against summative evaluation, formative evaluation is highly reliable.Choose the correct answer from the following code :
    (1) Both (A) and (R) are true and (R) is the correct explanation of (A).
    (2) Both (A) and (R) are true, but (R) is not the correct explanation of (A).
    (3) (A) is true, but (R) is false.
    (4) (A) is false, but (R) is true.

August 2016

  1. A teacher in a classroom has immediate control over
    (1) the self, selected methods of communication and the message.
    (2) the audience, the noise and the reception.
    (3) the feedback, the technology and the audience experience.
    (4) the communication channel, other communicators, and external factors.
  2. Which of the following set of statements reflects the basic characteristics of teaching ?Select the correct alternative from the codes :(i) Teaching is the same as training.
    (ii) There is no difference between instruction and conditioning when we teach.
    (iii) Teaching is related to learning.
    (iv) Teaching is a ‘task’ word while learning is an ‘achievement’ word.
    (v) Teaching means giving information.
    (vi) One may teach without learning taking place.Codes :
    (1) (i), (ii) and (iii) (2) (iii), (iv) and (vi)
    (3) (ii), (iii) and (v) (4) (i), (iv) and (vi)
  3. Which of the following factors affect teaching ?Choose the correct code to indicate your answer.(i) Teacher’s internal locus of control.
    (ii) Learner-motivation.
    (iii) Teacher’s biographical data.
    (iv) Teacher’s self efficacy.
    (v) Learner’s interest in co-curricular activities.
    (vi) Teacher’s skill in managing and monitoring.
    (vii) Teacher’s knowledge of the subject including general knowledge.Codes :(1) (i), (ii), (iv), (vi) and (vii) (2) (i), (ii), (iii), (iv) and (v)
    (3) (iii), (iv), (v), (vi) and (vii) (4) (ii), (iii), (iv), (v) and (vi)
  4. Which combination of methods of teaching is likely to optimise learning ?(1) Lecturing, discussions and seminar method
    (2) Interactive discussions, planned lectures and PowerPoint based presentations
    (3) Interactive lecture sessions followed by buzz sessions, brainstorming and projects
    (4) Lecturing, demonstrations and PowerPoint based presentations
  5. Assertion (A) : Teaching aids have to be considered as effective supplements to instruction.
    Reason (R) : They keep the students in good humour.
    Choose the correct answer from the codes given below :
    Codes :
    (1) Both (A) and (R) are true and (R) is the correct explanation of (A).
    (2) Both (A) and (R) are true, but (R) is not the correct explanation of (A).
    (3) (A) is true, but (R) is false.
    (4) (A) is false, but (R) is true.
  6. Which of the following learner characteristics are likely to influence the effectiveness of teaching aids and evaluation systems to ensure positive results ?(1) Learner’s family background, age and habitation.
    (2) Learner’s parentage, socio-economic background and performance in learning of the concerned subject.
    (3) Learner’s stage of development, social background and personal interests.
    (4) Learner’s maturity level, academic performance level and motivational dispositions.

July 2016

  1. Select the alternative which consists of positive factors contributing to effectiveness of teaching :List of factors :(a) Teacher’s knowledge of the subject.
    (b) Teacher’s socio-economic background.
    (c) Communication skill of the teacher.
    (d) Teacher’s ability to please the students.
    (e) Teacher’s personal contact with students.
    (f) Teacher’s competence in managing and monitoring the classroom transactions.Codes :(1) (b), (c) and (d) (2) (c), (d) and (f)
    (3) (b), (d) and (e) (4) (a), (c) and (f)
  2. The use of teaching aids is justified on the grounds of(1) attracting students’ attention in the class room.
    (2) minimising indiscipline problems in the classroom.
    (3) optimising learning outcomes of students.
    (4) effective engagement of students in learning tasks.
  3. Identify the important element a teacher has to take cognizance of while addressing students in a classroom.(1) Avoidance of proximity (2) Voice modulation
    (3) Repetitive pause (4) Fixed posture
  4. As a teacher, select the best option to ensure your effective presence in the classroom.(1) Use of peer command (2) Making aggressive statements
    (3) Adoption of well-established posture (4) Being authoritarian
  5. The best way for providing value education is through(1) discussions on scriptural texts (2) lectures / discourses on values
    (3) seminars / symposia on values (4) mentoring / reflective sessions on values
  6. Which set of learner characteristics may be considered helpful in designing effective teaching-learning systems ? Select the correct alternative from the codes given below :(i) Prior experience of learners in respect of the subject.
    (ii) Interpersonal relationships of learner’s family friends.
    (iii) Ability of the learners in respect of the subject.
    (iv) Student’s language background.
    (v) Interest of students in following the prescribed dress code.
    (vi) Motivational-orientation of the students.Codes :(1) (i), (ii), (iii) and (iv) (2) (i), (iii), (iv) and (vi)
    (3) (ii), (iii), (iv) and (v) (4) (iii), (iv), (v) and (vi)

December 2015

  1. Which of the following attributes denote great strengths of a teacher ?(a) Full-time active involvement in the institutional management
    (b) Setting examples
    (c) Willingness to put assumptions to the test
    (d) Acknowledging mistakes
    Select the correct answer from the codes given below :
    (1) (a), (b) and (d) (2) (b), (c) and (d)
    (3) (a), (c) and (d) (4) (a), (b), (c) and (d)
  2. What are required for good teaching ?(a) Diagnosis (b) Remedy (c) Direction (d) FeedbackSelect the correct answer from the codes given below :(1) (a), (b), (c) and (d) (2) (a) and (b)
    (3) (b), (c) and (d) (4) (c) and (d)
  3. The main objectives of student evaluation of teachers are :(a) To gather information about student weaknesses.
    (b) To make teachers take teaching seriously.
    (c) To help teachers adopt innovative methods of teaching.
    (d) To identify the areas of further improvement in teacher traits.Identify the correct answer from the codes given below :(1) (a) and (b) only (2) (b), (c) and (d) only
    (3) (a), (b) and (c) only (4) (a) only

June 2015

  1. Which of the following is the highest level of cognitive ability ?(1) Knowing (2) Understanding (3) Analysing (4) Evaluating
  2. Which of the following factors does not impact teaching ?(1) Teacher’s knowledge
    (2) Class room activities that encourage learning
    (3) Socio-economic background of teachers and students
    (4) Learning through experience
  3. Which of the following statements about teaching aids are correct ?(a) They help in retaining concepts for longer duration.
    (b) They help students learn better.
    (c) They make teaching learning process interesting.
    (d) They enhance rote learning.Select the correct answer from the codes given below :(1) (a), (b), (c) and (d) (2) (a), (b) and (c)
    (3) (b), (c) and (d) (4) (a), (b) and (d)
  4. Techniques used by a teacher to teach include :(a) Lecture (b) Interactive lecture
    (c) Group work (d) Self studySelect the correct answer from the codes given below :(1) (a), (b) and (c) (2) (a), (b), (c) and (d)
    (3) (b), (c) and (d) (4) (a), (b) and (d)
  5. Achievement tests are commonly used for the purpose of :(1) Making selections for a specific job
    (2) Selecting candidates for a course
    (3) Identifying strengths and weaknesses of learners
    (4) Assessing the amount of learning after teaching
  6. A good teacher is one who :(1) gives useful information
    (2) explains concepts and principles
    (3) gives printed notes to students
    (4) inspires students to learn

December 2014

  1. A smart classroom is a teaching space which has(i) Smart portion with a touch panel control system.
    (ii) PC/Laptop connection and DVD/VCR player.
    (iii) Document camera and specialized software
    (iv) Projector and screenSelect the correct answer from the codes given below :(A) (i) and (ii) only (B) (ii) and (iv) only
    (C) (i), (ii) and (iii) only (D) (i), (ii), (iii) and (iv)
  2. Digital Empowerment means(i) Universal digit literacy
    (ii) Universal access to all digital resources.
    (iii) Collaborative digital platform for participative governance.
    (iv) Probability of all entitlements for individuals through cloud.Choose the correct answer from the codes given below :(A) (i) and (ii) only (B) (ii) and (iii) only
    (C) (i), (ii) and (iii) only (D) (i), (ii), (iii) and (iv)
  3. Which of the following is not a prescribed level of teaching ?(A) Memory (B) Understanding (C) Reflective (D) Differentiation
  4. Maximum participation of students during teaching is possible through(A) Lecture method (B) Demonstration method
    (C) Inductive method (D) Textbook method
  5. Instructional aids are used by the teacher to(A) glorify the class (B) attract the students
    (C) clarify the concepts (D) ensure discipline
  6. Attitude of the teacher that affects teaching pertains to(A) Affective domain (B) Cognitive domain
    (C) Connative domain (D) Psychomotor domain

December 2013

      1. While delivering lecture if there is some disturbance in the class, a teacher should(A) keep quiet for a while and then continue.
        (B) punish those causing disturbance.
        (C) motivate to teach those causing disturbance.
        (D) not bother of what is happening in the class.
      2. Effective teaching is a function of(A) Teacher’s satisfaction.
        (B) Teacher’s honesty and commitment.
        (C) Teacher’s making students learn and understand.
        (D) Teacher’s liking for professional excellence.
      3. The most appropriate meaning of learning is(A) Acquisition of skills (B) Modification of behaviour
        (C) Personal adjustment (D) Inculcation of knowledge
      4. Arrange the following teaching process in order :(i) Relate the present knowledge with previous one
        (ii) Evaluation
        (iii) Reteaching
        (iv) Formulating instructional objectives
        (v) Presentation of instructional materialsCode :(A) (i), (ii), (iii), (iv), (v)
        (B) (ii), (i), (iii), (iv), (v)
        (C) (v), (iv), (iii), (i), (ii)
        (D) (iv), (i), (v), (ii), (iii)
      5. Teacher’s role at higher education level is to(A) provide information to students.
        (B) promote self learning in students.
        (C) encourage healthy competition among students.
        (D) help students to solve their problems.

June 2014

    1. Which one of the following is considered a sign of motivated teaching ?(A) Students asking questions (B) Maximum attendance of the students
      (C) Pin drop silence in the classroom (D) Students taking notes
    2. Which one of the following is the best method of teaching ?(A) Lecture (B) Discussion (C) Demonstration (D) Narration
    3. Dyslexia is associated with(A) mental disorder (B) behavioural disorder
      (C) reading disorder (D) writing disorder

September 2013

        1. A good teacher must be(A) resourceful and autocratic (B) resourceful and participative
          (C) resourceful and authoritative (D) resourceful and dominant
        2. Which one of the following is the best method of teaching ?(A) Lecture method (B) Discussion method
          (C) Demonstration method (D) Question-Answer method
        3. Successful Communication in classroom teaching is(A) Circular (B) Reciprocal (C) Directional (D) Influential
        4. Which one of the following types of evaluation assesses the learning progress to provide continuous feedback to both teachers and students during instruction ?(A) Placement evaluation (B) Formative evaluation
          (C) Diagnostic evaluation (D) Summative
        5. Which one of the following is a scaled down teaching situation ?(A) Macro teaching (B) Team teaching
          (C) Cooperative teaching (D) Micro teaching
        6. While delivering lecture if there is some disturbance in the class, a teacher should(A) keep quiet for a while and then continue.
          (B) punish those causing disturbance.
          (C) motivate to teach those causing disturbance.
          (D) not bother of what is happening in the class.
        7. Effective teaching is a function of(A) Teacher’s satisfaction.
          (B) Teacher’s honesty and commitment.
          (C) Teacher’s making students learn and understand.
          (D) Teacher’s liking for professional excellence.

December 2012

        1. For an efficient and durable learning, learner should have(A) ability to learn only (B) requisite level of motivation only
          (C) opportunities to learn only (D) desired level of ability and motivation
        2. Classroom communication must be(A) Teacher centric (B) Student centric
          (C) General centric (D) Textbook centric
        3. The best method of teaching is to(A) impart information (B) ask students to read books
          (C) suggest good reference material (D) initiate a discussion and participate in it
        4. Interaction inside the classroom should generate(A) Argument (B) Information (C) Ideas (D) Controversy
        5. “Spare the rod and spoil the child”, gives the message that(A) punishment in the class should be banned.
          (B) corporal punishment is not acceptable.
          (C) undesirable behaviour must be punished.
          (D) children should be beaten with rods.
        6. The type of communication that the teacher has in the classroom, is termed as(A) Interpersonal (B) Mass communication
          (C) Group communication (D) Face-to-face communication

June 2013

        1. The black-board can be utilised best by a teacher for(A) putting the matter of teaching in black and white
          (B) making the students attentive
          (C) writing the important and notable points
          (D) highlighting the teacher himself

Nowadays the most effective mode of learning is

(A) self study (B) face-to-face learning
(C) e-learning (D) blended learning

        1. At the primary school stage, most of the teachers should be women because they(A) can teach children better than men.
          (B) know basic content better than men.
          (C) are available on lower salaries.
          (D) can deal with children with love and affection.
        2. Which one is the highest order of learning ?(A) Chain learning (B) Problem-solving learning
          (C) Stimulus-response learning (D) Conditioned-reflex learning
        3. A person can enjoy teaching as a profession when he(A) has control over students.
          (B) commands respect from students.
          (C) is more qualified than his colleagues.
          (D) is very close to higher authorities.
        4. “A diagram speaks more than 1000 words.” The statement means that the teacher should(A) use diagrams in teaching. (B) speak more and more in the class.
          (C) use teaching aids in the class. (D) not speak too much in the class.
        1. The aim of value education to inculcate in students is(A) the moral values (B) the social values
          (C) the political values (D) the economic values
        2. By which of the following methods the true evaluation of the students is possible ?(A) Evaluation at the end of the course. (B) Evaluation twice in a year.
          (C) Continuous evaluation. (D) Formative evaluation.
        3. Suppose a student wants to share his problems with his teacher and he visits the teacher’s house for the purpose, the teacher should(A) contact the student’s parents and solve his problem.
          (B) suggest him that he should never visit his house.
          (C) suggest him to meet the principal and solve the problem.
          (D) extend reasonable help and boost his morale.
        4. When some students are deliberately attempting to disturb the discipline of the class by making mischief, what will be your role as a teacher ?(A) Expelling those students.
          (B) Isolate those students.
          (C) Reform the group with your authority.
          (D) Giving them an opportunity for introspection and improve their behaviour.
        5. Which of the following belongs to a projected aid ?(A) Blackboard (B) Diorama (C) Epidiascope (D) Globe
        6. A teacher is said to be fluent in asking questions, if he can ask(A) meaningful questions
          (B) as many questions as possible
          (C) maximum number of questions in a fixed time
          (D) many meaningful questions in a fixed time
        7. Which of the following qualities is most essential for a teacher ?(A) He should be a learned person.
          (B) He should be a well dressed person.
          (C) He should have patience.
          (D) He should be an expert in his subject.

June 2012

        1. If you get an opportunity to teach a visually challenged student along with normal students, what type of treatment would you like to give him in the class ?(A) Not giving extra attention because majority may suffer.
          (B) Take care of him sympathetically in the classroom.
          (C) You will think that blindness is his destiny and hence you cannot do anything.
          (D) Arrange a seat in the front row and try to teach at a pace convenient to him.
        2. Which of the following does not belong to a projected aid ?(A) Overhead projector (B) Blackboard
          (C) Epidiascope (D) Slide projector
        3. For a teacher, which of the following methods would be correct for writing on the blackboard ?(A) Writing fast and as clearly as possible.
          (B) Writing the matter first and then asking students to read it.
          (C) Asking a question to students and then writing the answer as stated by them.
          (D) Writing the important points as clearly as possible.
        4. A teacher can be successful if he/she(A) helps students in becoming better citizens
          (B) imparts subject knowledge to students
          (C) prepares students to pass the examination
          (D) presents the subject matter in a well organized manner
        5. Dynamic approach to teaching means(A) Teaching should be forceful and effective
          (B) Teachers should be energetic and dynamic
          (C) The topics of teaching should not be static, but dynamic
          (D) The students should be required to learn through activities

December 2011

June 2011

        1. For maintaining an effective discipline in the class, the teacher should(A) Allow students to do what they like.
          (B) Deal with the students strictly.
          (C) Give the students some problem to solve.
          (D) Deal with them politely and firmly.
        2. An effective teaching aid is one which(A) is colourful and good looking (B) activates all faculties
          (C) is visible to all students (D) easy to prepare and use
        3. Those teachers are popular among students who(A) develop intimacy with them (B) help them solve their problems
          (C) award good grades (D) take classes on extra tuition fee
        4. The essence of an effective classroom environment is(A) a variety of teaching aids (B) lively student-teacher interaction
          (C) pin-drop silence (D) strict discipline
        5. On the first day of his class, if a teacher is asked by the students to introduce himself, he should(A) ask them to meet after the class
          (B) tell them about himself in brief
          (C) ignore the demand and start teaching
          (D) scold the student for this unwanted demand
        6. Moral values can be effectively inculcated among the students when the teacher(A) frequently talks about values (B) himself practices them
          (C) tells stories of great persons (D) talks of Gods and Goddesses

December 2010

  1. Some students in a class exhibit great curiosity for learning. It may be because such children
    (A) Are gifted
    (B) Come from rich families
    (C) Show artificial behaviour
    (D) Create indiscipline in the class
  2. The most important quality of a good teacher is
    (A) Sound knowledge of subject matter
    (B) Good communication skills
    (C) Concern for students’ welfare
    (D) Effective leadership qualities
  3. Which one of the following is appropriate in respect of teacher- student relationship ?
    (A) Very informal and intimate
    (B) Limited to classroom only
    (C) Cordial and respectful
    (D) Indifferent
  4. The academic performance of students can be improved if parents are encouraged to
    (A) supervise the work of their wards
    (B) arrange for extra tuition
    (C) remain unconcerned about it
    (D) interact with teachers frequently
  5. In a lively classroom situation, there is likely to be
    (A) occasional roars of laughter
    (B) complete silence
    (C) frequent teacher-student dialogue
    (D) loud discussion among students
  6. If a parent approaches the teacher to do some favour to his/her ward in the examination, the teacher should
    (A) try to help him
    (B) ask him not to talk in those terms
    (C) refuse politely and firmly
    (D) ask him rudely to go away

June 2010

  1. Which one of the following is the most important quality of a good teacher ?
    (A) Punctuality and sincerity
    (B) Content mastery
    (C) Content mastery and reactive
    (D) Content mastery and sociable
  2. The primary responsibility for the teacher’s adjustment lies with
    (A) The children
    (B) The principal
    (C) The teacher himself
    (D) The community
  3. As per the NCTE norms, what should be the staff strength for a unit of 100 students at B.Ed. level ?
    (A) 1 + 7
    (B) 1 + 9
    (C) 1 + 10
    (D) 1 + 5
  4. Research has shown that the most frequent symptom of nervous instability among teachers is
    (A) Digestive upsets
    (B) Explosive behaviour
    (C) Fatigue
    (D) Worry
  5. Which one of the following statements is correct ?
    (A) Syllabus is an annexure to the curriculum.
    (B) Curriculum is the same in all educational institutions.
    (C) Curriculum includes both formal, and informal education.
    (D) Curriculum does not include methods of evaluation.
  6. A successful teacher is one who is
    (A) Compassionate and disciplinarian
    (B) Quite and reactive
    (C) Tolerant and dominating
    (D) Passive and active

December 2009

  1. Which of the following skills are needed for present day teacher to adjust effectively with the classroom teaching ?
    1. Knowledge of technology
    2. Use of technology in teaching learning
    3. Knowledge of students’ needs
    4. Content mastery
    (A) 1 & 3
    (B) 2 & 3
    (C) 2, 3 & 4
    (D) 2 & 4
  2. The primary duty of the teacher is to
    (A) raise the intellectual standard of the students
    (B) improve the physical standard of the students
    (C) help all round development of the students
    (D) imbibe value system in the students
  3. Micro teaching is more effective
    (A) during the preparation for teaching-practice
    (B) during the teaching-practice
    (C) after the teaching-practice
    (D) always
  4. What quality the students like the most in a teacher ?
    (A) Idealist philosophy
    (B) Compassion
    (C) Discipline
    (D) Entertaining
  5. Which of the following is more interactive and student centric ?
    (A) Seminar
    (B) Workshop
    (C) Lecture
    (D) Group Discussion

June 2009

  1. Good evaluation of written material should not be based on :
    (A) Linguistic expression
    (B) Logical presentation
    (C) Ability to reproduce whatever is read
    (D) Comprehension of subject
  2. Why do teachers use teaching aid ?
    (A) To make teaching fun-filled
    (B) To teach within understanding level of students
    (C) For students’ attention
    (D) To make students attentive
  3. Attitudes, concepts, skills and knowledge are products of :
    (A) Learning
    (B) Research
    (C) Heredity
    (D) Explanation
  4. Which among the following gives more freedom to the learner to interact ?(A) Use of film
    (B) Small group discussion
    (C) Lectures by experts
    (D) Viewing country-wide classroom programme on TV
  5. Which of the following is not a product of learning ?
    (A) Attitudes
    (B) Concepts
    (C) Knowledge
    (D) Maturation
Share to your friend

Communication 

Previous Year Question Paper 2009 to June 2019

June 2019 – Main Paper (Commerce)

  1. In the context of a classroom teaching ‘Smiles or Frowns’ when used as non-verbal cues, tend to perform which of the following basic functions ?
    1. Contradicting the verbal cue function
    2. regulating the verbal cue function
    3. substitute function
    4. supplement function
  2. Non-verbal communication is mostly
    1. mechanical
    2. organic
    3. unstructured
    4. structured
  3. Below are given two sets in which Set 1 describes the types of listener involved in communication, while Set II indicates their characteristics:
    Set – I
    (Types of listener involved in Communication)
    (a) Non Listener
    (b) Marginal Listener
    (c) Evaluative Listener
    (d) Active Listener
    Set – II
    (Characteristics)
    (i) is engaged in information other than the one needed
    (ii) receives information without processing the significance in the context of communication
    (iii) looks into the relevance of the information for understanding its implication
    (iv) pays heed to the communicated information occasionally
    Match the two sets and give your answer by choosing from the options :
    (a) (b) (c) (d)
    1. (iv) (iii) (ii) (i)
    2. (iii) (iv) (i) (ii)
    3. (ii) (i) (iii) (iv)
    4. (i) (iv) (ii) (iii)
  4. Which of the following principles a good communicator should follow ?
    (a) He should stay organised
    (b) He should avoid the use of visuals
    (c) He should be authoritarian
    (d) He must adjust to the medium
    (e) He should prefer the method of story-telling
    Choose the correct answer from the options given below :
    (1) (a),(b) and (c)
    (2) (b),(c) and (e)
    (3) (b),(d) and (e)
    (4) (a), (d) and (e)
  5. Which of the following elements a good classroom communication should adopt ?
    (a) Concreteness
    (b) Courtesy
    (c) Filibustering
    (d) Fictionalisation
    (e) Coherence
    Choose the answer from the options given below :
    (1) (a),(b) and (d)
    (2) (b),(d) and (e)
    (3) (a),(b) and (e)
    (4) (a), (c) and (d)

June 2019- Main Paper (Management)

  1. Which of the following is a stage of intrapersonal communication ?
    1. Phatic stage
    2. Intimate stage
    3. Personal stage
    4. Transcendental communication
  2. Which of the following signifies non-verbal communication in an essential way ?
    1. Instructions written on a question paper
    2. Learner attending an online class
    3. The dress code followed by an individual
    4. Yoga instructor teaching yoga poses to the pupils
  3. Assertion (A) : Communication presupposes a shared, symbolic environment, a social relationship even among non-participants.
    Reason (R) : It leads to social interaction, which in combination with other factors contributes to a sense of community.
    Choose the correct option given below :
    1. Both (A) and (R) are true and R is the correct explanation of A
    2. Both (A) and (R) are true and R is not the correct explanation of A
    3. (A) is true but (R) is false
    4. (A) false but (R) is true
  4. Semantic barrier of communication is implied when the
    1. signal is lost before reaching the receiver
    2. message transmitted by the source is unclear
    3. rceiver’s attention is diverted
    4. receiver does not understand the meaning of the message
  5. Which of the following is a characteristic of modern mass media?
    1. Capital intensive
    2. Labour intensive
    3. Sustainable
    4. Developmental

December 2018- Main Paper (Commerce)

  1. Internal cues of non-verbal behavior in the classroom help diagnose :
    1. Spatial proximation
    2. Inter-personal dynamics
    3. Student movement
    4. Mass beliefs
  2. Assertion (A) : Classroom communication behaviors imply making the contents relevant.
    Reason (R) : It is none of the functions of teachers to make students organize information for effective interaction.
    1. Both (A) and (R) are true, but (R) is not the correct explanation of (A)
    2. (A) is true, but (R) is false
    3. (A) is false, but (R) is true
    4. Both (A) and (R) are true
  3. In pre-learning, preparation, an important stage of communication is :
    1. Iconisation of encoder
    2. Ignoring semantic noise
    3. Information acquisition and its processing
    4. Conditioned feedback
  4. Communication variables involved in a model of performance are :
    (a) Encoding
    (b) Decoding
    (c) Passive resistance
    (d) Channelisation
    (e) Indifference
    Select the correct option from the code given below :
    1. (b), (c), (d) and (e)
    2. (a), (b), (c) and (e) only
    3. (a), (c) and (d) only
    4. (a), (b) and (d) only
  5. To communicate effectively with students, teachers should use :
    (a) Affinity-seeking strategies
    (b) Immediacy behaviors
    (c) Humor
    (d) Collaborative filters
  6. Choose most appropriate code :
    1. (a), (b) and (c)
    2. (a), (c) and (d)
    3. (a) and (d) only
    4. (a) and (b) only

December 2018- Main Paper (Management)

  1. Signs are considered as secondary products of :
    1. Message
    2. Infotainment
    3. Channelisation
    4. Semantic accuracy
  2. The information seeking strategics of students are :
    (a) Entropy-oriented
    (b) Overt
    (c) Non-functional
    (d) Indirect
    (e) Third party
    (f) Testing
    Code :
    1. (b), (c), (e) and (f)
    2. (a), (b), (c) and (f)
    3. (a), (b), (d) and (e)
    4. (b), (d), (e) and (f)
  3. In a classroom, communication becomes effective when participants are :
    (a) Active
    (b) Empathetic
    (c) Heterophilous
    (d) Homophilous
    (e) Silent
    (f) Uncritical
    Code :
    1. (b), (c), (d) and (e)
    2. (a), (b) and (d)
    3. (a), (b), (c) and (d)
    4. (c), (e) and (f)
  4. The relational approach in classroom communication comes from :
    1. Mass communication
    2. Intra-personal communication
    3. Inter-personal communication
    4. Organisational communication
  5. Classroom speech is considered as a :
    1. Non-technical medium
    2. Mass medium
    3. Unorganised medium
    4. Technical medium

July 2018

  1. Differentiation between acceptance and non-acceptance of certain stimuli in classroom communication is the basis of :
    1. Selective affiliation to peer groups
    2. Selective attention
    3. morality
    4. expectation of performance
  2. Assertion (A) :- The initial messages to students in the classroom by a teacher need not be critical to establish interactions later.
    Reason (R): More control over the communication process means more control over what the students are learning.
    Code :-
    1. Both (A) and (R) are true, but (R) is not the correct explanation of (A)
    2. (A) is true, but (R) is false
    3. (A) is false, but (R) is true
    4. (A) and (R) are true, and (R) is the correct explanation of (A)
  3. Assertion (A) : To communicate well in the classroom is a natural ability.
    Reason (R) : Effective teaching in the classroom demands knowledge of the communication process.
    Code :-
    1. Both (A) and (R) are true, but (R) is not the correct explanation of (A).
    2. (A) is true, but (R) is false.
    3. (A) is false, but (R) is true.
    4. Both (A) and (R) are true, and (R) is the correct explanation of (A).
  4. Assertion (A) : Classroom communication is a transactional process.
    Reason (R) : A teacher does not operate under the assumption that students responses are purposive.
    Select the correct code for your answer :-
    1. Both (A) and (R) are true, but (R) is not the correct explanation of (A)
    2. (A) is true, but (R) is false
    3. (A) is false, but (R) is true
    4. Both (A) and (R) are true, and (R) is the correct explanation of (A)
  5. Which of the following set of statements is correct for describing the human communication process?
    a. Non-verbal communication can stimulate ideas.
    b. Communication is a learn ability.
    c .Communication is not a universal panacea.
    d. Communication can not break-down.
    e. More communication means more effective learning by students.
    f. Value of what is learn through classroom communication is not an issue for students
    Code :-
    1 (b), (d),(e),and (f)
    2 (a), (b),(c),and (d)
    3 (a), (d),(e),and (f)
    4 (a), (c),(e),and (f)

November 2017

  1. The interaction between a teacher and students creates a zone of proximal:
    (1) Difference
    (2) Confusion
    (3) Development
    (4) Distortion
  2. The spatial audio reproduction in a classroom can reduce the students:
    (1) Congnitive load in understanding
    (2) Respect for the teacher
    (3) Motivation for excellence
    (4) Interest in technology-orientation
  3. The classroom communication should essentially be:
    (1) Contrived
    (2) Empathatic
    (3) Abstract
    (4) Non-descriptive
  4. A good communicator begins his/her presentation with a:
    (1) Complex question
    (2) Non-sequitur
    (3) Repetitive phrase
    (4) Ice-breaker
  5. In a classroom, the probability of message reception can be enhanced by:
    (1) Establishing a viewpoint
    (2) Exposing the ignorance of students
    (3) Increasing the information load
    (4) Using high decibel audio tools

January 2017

  1. Effective communication pre-supposes
    (1) Non-alignment                                          (2) Domination
    (3) Passivity                                              (4) Understanding
  2. When verbal and non-verbal messages are contradictory, it is said that most people believe in
    (1) indeterminate messages                        (2) verbal messages
    (3) non-verbal messages                              (4) aggressive messages
  3. The typical feature of an information-rich classroom lecture is in the nature of being
    (1) Sedentary                                               (2) Staggered
    (3) Factual                                                 (4) Sectoral
  4. Expressive communication is driven by
    (1) Passive aggression                           (2) Encoder’s personality characteristics
    (3) External clues                               (4) Encoder-decoder contract
  5. Positive classroom communication leads to
    (1) Coercion                                             (2) Submission
    (3) Confrontation                                        (4) Persuasion
  6. Classroom communication is the basis of
    (1) Social identity                                      (2) External inanities
    (3) Biased passivity                                     (4) Group aggression

August 2016

  1. Internal and external factors that affect message reception by the students in the classroom are referred to as
    (1) feedback                                              (2) fragmentation
    (3) channelisation                                        (4) noise
  2. What do communicated words carry in a classroom situation ?
    (1) Inspiration, controversy and introspection
    (2) Diversion, criticism and irrationality
    (3) Insipidity, irrationality, and non-acceptance
    (4) Power, structure and tradition
  3. As a good classroom communicator, you are supposed to know your
    (1) audience emotions                                 (2) silent cues
    (3) artful pauses                                      (4) counter arguments
  4. Figure out the components of non-verbal communication in a classroom from the following :
    (1) Facial expression, cultural space and seating arrangement
    (2) Speed of utterance, feel good factor and acoustics
    (3) High sound, physical ambience and teacher-learner distance
    (4) Facial expression, kinesics and personal space
  5. Which of the following are the basic factors of effective listening ?
    (1) Opinionation, stare and glare and interruptions
    (2) Aggressive questioning, continuous cues and frequent movement
    (3) Me-too-ism, glancing sideways, and offering advice
    (4) Acknowledgement of thoughts, reflection, and asking open-ended questions

June 2016

  1. Imagine you are working in an educational institution where people are of equal status. Which method of communication is best suited and normally employed in such a context ?
    (1) Horizontal communication                          (2) Vertical communication
    (3) Corporate communication                           (4) Cross communication
  2. What are the barriers to effective communication ?
    (1) Moralising, being judgemental and comments of consolation.
    (2) Dialogue, summary and self-review.
    (3) Use of simple words, cool reaction and defensive attitude.
    (4) Personal statements, eye contact and simple narration.
  3. The choice of communication partners is influenced by factors of
    (1) Proximity, utility, loneliness                    (2) Utility, secrecy, dissonance
    (3) Secrecy, dissonance, deception                    (4) Dissimilarity, dissonance, deviance
  4. Every communicator has to experience
    (1) Manipulated emotions                              (2)Anticipatory excitement
    (3) The issue of homophiles                           (4) Status dislocation

December 2015

  1. Using the central point of the classroom communication as the beginning of a dynamic pattern of ideas is referred to as :
    (1) Systemisation
    (2) Problem – orientation
    (3) Idea protocol
    (4) Mind mapping
  2. Aspects of the voice, other than the speech are known as :
    (1) Physical language
    (2) Personal language
    (3) Para language
    (4) Delivery language
  3. Every type of communication is affected by its :
    (1) Reception
    (2) Transmission
    (3) Non-regulation
    (4) Context
  4. Attitudes, actions and appearances in the context of classroom communication are considered as :
    (1) Verbal
    (2) Non-verbal
    (3) Impersonal
    (4) Irrational
  5. Most often, the teacher – student communication is :
    (1) Spurious
    (2) Critical
    (3) Utilitarian
    (4) Confrontational
  6. In a classroom, a communicator’s trust level is determined by :
    (1) the use of hyperbole                                     (2) the change of voice level
    (3) the use of abstract concepts                             (4) eye contact

June 2015

  1. Effectiveness of communication can be traced from which of the following ?
    (a) Attitude surveys
    (b) Performance records
    (c) Students attendance
    (d) Selection of communication channel
    Select the correct answer from the codes given below :
    (1) (a), (b), (c) and (d)
    (2) (a), (b) and (c)
    (3) (b), (c) and (d)
    (4) (a), (b) and (d)
  2. Assertion (A) : Formal communication tends to be fast and flexible.
    Reason (R) : Formal communication is a systematic and orderly flow of information.
    (1) Both (A) and (R) are correct and (R) is correct explanation of (A)
    (2) Both (A) and (R) are correct, but (R) is not correct explanation of (A)
    (3) (A) is correct but, (R) is false
    (4) (A) is false but, (R) is correct
  3. Which of the following are the characteristic features of communication ?
    (a) Communication involves exchange of ideas, facts and opinions.
    (b) Communication involves both information and understanding.
    (c) Communication is a continuous process.
    (d) Communication is a circular process.
    Select the correct answer from the codes given below :
    (1) (a), (b) and (c)
    (2) (a), (b) and (d)
    (3) (b), (c) and (d)
    (4) (a), (b), (c) and (d)
  4. The term ‘grapevine’ is also known as :
    (1) Downward communication
    (2) Informal communication
    (3) Upward communication
    (4) Horizontal communication
  5. Which of the following is not a principle of effective communication ?
    (1) Persuasive and convincing dialogue
    (2) Participation of the audience
    (3) One-way transfer of information
    (4) Strategic use of grapevine
  6. In communication, the language is :
    (1) The verbal code
    (2) Intrapersonal
    (3) The symbolic code
    (4) The non-verbal code

December 2014

  1. The term ‘Yellow Journalism’ refers to
    (A) sensational news about terrorism and violence
    (B) sensationalism and exaggeration to attract readers / viewers.
    (C) sensational news about arts and culture
    (D) sensational news prints in yellow paper.
  2. In the classroom, the teacher sends the message either as words or images. The students are really
    (A) Encoders
    (B) Decoders
    (C) Agitators
    (D) Propagators
  3. Media is known as
    (A) First Estate
    (B) Second Estate
    (C) Third Estate
    (D) Fourth Estate
  4. The mode of communication that involves a single source transmitting information to a large number of receivers simultaneously, is called
    (A) Group Communication
    (B) Mass Communication
    (C) Intrapersonal Communication
    (D) Interpersonal Communication

June 2014

  1. Break-down in verbal communication is described as
    (A) Short circuit
    (B) Contradiction
    (C) Unevenness
    (D) Entropy
  2. The Telephone Model of Communication was first developed in the area of
    (A) Technological theory
    (B) Dispersion theory
    (C) Minimal effects theory
    (D) Information theory
  3. The Dada Saheb Phalke Award for 2013 has been conferred on

    (A) Karan Johar           (B) Amir Khan         (C) Asha Bhonsle        (D) Gulzar

  4. Photographs are not easy to

    (A) publish
    (B) secure
    (C) decode
    (D) change

  5. The grains that appear on a television set when operated are also referred to as
    (A) sparks
    (B) green dots
    (C) snow
    (D) rain drops
  6. In circular communication, the encoder becomes a decoder when there is
    (A) noise
    (B) audience
    (C) criticality
    (D) feedback
  7. Classroom communication is normally considered as
    (A) effective
    (B) cognitive
    (C) affective
    (D) selective

December 2013

  1. Users who use media for their own ends are identified as
    (A) Passive audience
    (B) Active audience
    (C) Positive audience
    (D) Negative audience
  2. Classroom communication can be described as
    (A) Exploration
    (B) Institutionalisation
    (C) Unsignified narration
    (D) Discourse
  3. Ideological codes shape our collective
    (A) Productions
    (B) Perceptions
    (C) Consumptions
    (D) Creations
  4. In communication, myths have power, but are
    (A) uncultural
    (B) insignificant
    (C) imprecise
    (D) unpreferred
  5. The first multi-lingual news agency of India was
    (A) Samachar
    (B) API
    (C) Hindustan Samachar
    (D) SamacharBharati
  6. Organisational communication can also be equated with
    (A) intra-personal communication.
    (B) inter-personal communication.
    (C) group communication.
    (D) mass communication.

September 2013

  1. Telephone is an example of
    (A) linear communication                         
    (B) non-linear communication
    (C) circular                                     
    (D) mechanised
  2. Means of grapevine communication are
    (A) formal           (B) informal             (C) critical        (D) corporate
  3. Communication issues at the international level are addressed by
    (A) ILO              (B) ITU                  (C) UNDP            (D) UNESCO
  4. Referential framing used by TV audience connects media with
    (A) reality          (B) falsity              (C) negativity      (D) passivity
  5. The communicated knowledge in a classroom is considered as
    (A) non-pervasive treasure
    (B) limited judgement
    (C) autonomous virtue
    (D) cultural capital
  6. Classroom communication is normally considered as
    (A) effective
    (B) affective
    (C) cognitive
    (D) non-selective

June 2013

  1. In the process of communication, which one of the following is in the chronological order ?
    (A) Communicator, Medium, Receiver, Effect, Message
    (B) Medium, Communicator, Message, Receiver, Effect
    (C) Communicator, Message, Medium, Receiver, Effect
    (D) Message, Communicator, Medium, Receiver, Effect
  2. Bengal Gazette, the first Newspaper in India was started in 1780 by
    (A) Dr. Annie Besant
    (B) James Augustus Hicky
    (C) Lord Cripson
    (D) A.O. Hume
  3. Press censorship in India was imposed during the tenure of the Prime Minister
    (A) Rajeev Gandhi
    (B) Narasimha Rao
    (C) Indira Gandhi
    (D) Deve Gowda
  4. Communication via New media such as computers, teleshopping, internet and mobile telephony is termed as
    (A) Entertainment
    (B) Interactive communication
    (C) Developmental communication
    (D) Communitarian
  5. Classroom communication of a teacher rests on the principle of
    (A) Infotainment
    (B) Edutainment
    (C) Entertainment
    (D) Enlightenment
  6. ______________ is important when a teacher communicates with his/her student.
    (A) Sympathy            (b) Empathy         (c) Apathy           (d) Antipathy

December 2012

  1. The English word ‘Communication’ is derived from the words
    (A) Communis and Communicare
    (B) Communist and Commune
    (C) Communism and Communalism
    (D) Communion and Common sense
  2. Chinese Cultural Revolution leader Mao Zedong used a type of communication to talk to the masses is known as
    (A) Mass line communication
    (B) Group communication
    (C) Participatory communication
    (D) Dialogue communication
  3. Conversing with the spirits and ancestors is termed as
    (A) Transpersonal communication
    (B) Intrapersonal communication
    (C) Interpersonal communication
    (D) Face-to-face communication
  4. The largest circulated daily newspaper among the following is
    (A) The Times of India
    (B) The Indian Express
    (C) The Hindu
    (D) The Deccan Herald
  5. The pioneer of the silent feature film in India was
    (A) K.A. Abbas
    (B) Satyajit Ray
    (C) B.R. Chopra
    (D) Dada Sahib Phalke
  6. Classroom communication of a teacher rests on the principle of
    (A) Infotainment
    (B) Edutainment
    (C) Entertainment
    (D) Power equation

June 2012

  1. Video-Conferencing can be classified as one of the following types of communication :
    (A) Visual one way
    (B) Audio-Visual one way
    (C) Audio-Visual two way
    (D) Visual two way
  2. All India Radio (A.I.R.) for broadcasting was named in the year
    (A) 1926
    (B) 1936
    (C) 1946
    (D) 1956
  3. In India for broadcasting TV programmes which system is followed ?
    (A) NTCS
    (B) PAL
    (C) NTSE
    (D) SECAM
  4. The term ‘DAVP’ stands for
    (A) Directorate of Advertising & Vocal Publicity
    (B) Division of Audio-Visual Publicity
    (C) Department of Audio-Visual Publicity
    (D) Directorate of Advertising & Visual Publicity
  5. The term “TRP” is associated with TV shows stands for
    (A) Total Rating Points
    (B) Time Rating Points
    (C) Thematic Rating Points
    (D) Television Rating Points

December 2011

  1. Photo bleeding means
    (A) Photo cropping
    (B) Photo placement
    (C) Photo cutting
    (D) Photo colour adjustment
  2. While designing communication strategy feed-forward studies are conducted by
    (A) Audience
    (B) Communicator
    (C) Satellite
    (d) Media
  3. In which language the newspapers have highest circulation?
    (A) English
    (B) Hindi
    (C) Bengali
    (D) Tamil
  4. Aspect ratio of TV Screen is
    (A) 4 : 3
    (B) 3 : 4
    (C) 2 : 3
    (D) 2 : 4
  5. Communication with oneself is known as
    (A) Organisational Communication
    (B) Grapewine Communication
    (C) Interpersonal Communication
    (D) Intrapersonal Communication
  6. The term ‘SITE’ stands for
    (A) Satellite Indian Television Experiment
    (B) Satellite International Television Experiment
    (C) Satellite Instructional Television Experiment
    (D) Satellite Instructional Teachers Education

June 2011

  1. Public communication tends to occur within a more
    (A) complex structure
    (B) political structure
    (C) convenient structure
    (D) formal structure
  2. Transforming thoughts, ideas and messages into verbal and non-verbal signs is referred to as
    (A) channelisation
    (B) mediation
    (C) encoding
    (D) decoding
  3. Effective communication needs a supportive
    (A) economic environment
    (B) political environment
    (C) social environment
    (D) multi-cultural environment
  4. A major barrier in the transmission of cognitive data in the process of communication is an individual’s
    (A) personality
    (B) expectation
    (C) social status
    (D) coding ability
  5. When communicated, institutionalised stereotypes become
    (A) myths
    (B) reasons
    (C) experiences
    (D) convictions
  6. In mass communication, selective perception is dependent on the receiver’s
    (A) competence
    (B) pre-disposition
    (C) receptivity
    (D) ethnicity

December 2010

  1. Media that exist in an interconnected series of communication – points are referred to as
    (A) Networked media
    (B) Connective media
    (C) Nodal media
    (D) Multimedia
  2. The information function of mass communication is described as
    (A) diffusion
    (B) publicity
    (C) surveillance
    (D) diversion
  3. An example of asynchronous medium is
    (A) Radio
    (B) Television
    (C) Film
    (D) Newspaper
  4. In communication, connotative words are
    (A) explicit
    (B) abstract
    (C) simple
    (D) cultural
  5. A message beneath a message is labelled as
    (A) embedded text
    (B) internal text
    (C) inter-text
    (D) sub-text
  6. In analog mass communication, stories are
    (A) static
    (B) dynamic
    (C) interactive
    (D) exploratory

June 2010

  1. In communication, a major barrier to reception of messages is
    (A) audience attitude
    (B) audience knowledge
    (C) audience education
    (D) audience income
  2. Post-modernism is associated with
    (A) newspapers
    (B) magazines
    (C) radio
    (D) television
  3. Didactic communication is
    (A) intra-personal
    (B) inter-personal
    (C) organisational
    (D) relational
  4. In communication, the language is
    (A) the non-verbal code
    (B) the verbal code
    (C) the symbolic code
    (D) the iconic code
  5. Identify the correct sequence of the following :
    (A) Source, channel, message, receiver
    (B) Source, receiver, channel, message
    (C) Source, message, receiver, channel
    (D) Source, message, channel, receiver
  6. Assertion (A) : Mass media promote a culture of violence in the society.
    Reason (R) : Because violence sells in the market as people themselves are violent in character.
    (A) Both (A) and (R) are true and (R) is the correct explanation of (A).
    (B) Both (A) and (R) are true, but
    (R) is not the correct explanation of (A).
    (C) (A) is true, but (R) is false.
    (D) Both (A) and (R) are false.

December 2009

  1. The country which has the distinction of having the two largest circulated newspapers in the world is
    (A) Great Britain
    (B) The United States
    (C) Japan
    (D) China
  2. The chronological order of non-verbal communication is
    (A) Signs, symbols, codes, colours
    (B) Symbols, codes, signs, colours
    (C) Colours, signs, codes, symbols
    (D) Codes, colours, symbols, signs
  3. Which of the following statements is not connected with communication ?
    (A) Medium is the message.
    (B) The world is an electronic cocoon.
    (C) Information is power.
    (D) Telepathy is technological.
  4. Communication becomes circular when
    (A) the decoder becomes an encoder
    (B) the feedback is absent
    (C) the source is credible
    (D) the channel is clear
  5. Assertion (A) : For an effective classroom communication at times it is desirable to use the projection technology.
    Reason (R) : Using the projection technology facilitates extensive coverage of course contents.
    (A) Both (A) and (R) are true, and (R) is the correct explanation.
    (B) Both (A) and (R) are true, but (R) is not the correct explanation.
    (C) (A) is true, but (R) is false.
    (D) (A) is false, but (R) is true.

June 2009

  1. Which one of the following Telephonic Conferencing with a radio link is very popular throughout the world ?
    (A) TPS
    (B) Telepresence
    (C) Video conference
    (D) Video teletext
  2. Which is not 24 hours news channel ?
    (A) NDTV 24×7
    (B) ZEE News
    (C) Aaj tak
    (D) Lok Sabha channel
  3. The main objective of F.M. station in radio is :
    (A) Information, Entertainment and Tourism
    (B) Entertainment, Information and Interaction
    (C) Tourism, Interaction and Entertainment
    (D) Entertainment only
  4. In communication chatting in internet is :
    (A) Verbal communication
    (B) Non verbal communication
    (C) Parallel communication
    (D) Grapevine communication
  5. Match List-I with List-II and select the correct answer using the codes given below :List-I
    (Artists)
    (a) Pandit Jasraj
    (b) Kishan Maharaj
    (c) Ravi Shankar
    (d) Udai ShankarList-II
    (Art)
    (i) Hindustani vocalist
    (ii) Sitar
    (iii) Tabla
    (iv) Dance
    Codes :
    (a) (b) (c) (d)
    (A) (i) (ii) (iii) (iv)
    (B) (i) (iii) (iv) (ii)
    (C) (i) (iii) (ii) (iv)
    (D) (iii) (ii) (i) (iv)
Share to your friend

People, Development and Environment 

Previous Year Question Paper 2009 to June 2019

June 2019 – Main Paper (Commerce)

  1. One of the parameters used to characterise the air quality at a location is PM2.5. Here, the suffix ‘2.5’ refers to
    1. average number of suspected particle in 1.0 cm3 of air
    2. size of suspended particles in certain units
    3. concentration of oxides of sulphur and nitrogen
    4. concentration of suspended particles in 2.5 m3 of air
  2. Choose the se of non-renewable resources
    1. Uranium, natural gas, soil
    2. Natural gas, uranium, phosphates
    3. Petroleum, uranium, geothermal energy
    4. Shale gas, uranium, soil
  3. Which of the following diseases are due to polluted water ?
    (a) Giardia
    (b) Dengue
    (c) Amoebiasis
    (d) Typhoid
    Choose the correct answer from the options given below :
    (1) (a),(b) and (d)
    (2) (b),(c) and (d)
    (3) (a),(b), (c) and (d)
    (4) (a), (c) and (d)
  4. Which of the following is non-vehicular pollutant?
    1. chlorofluorocarbon
    2. carbon-monoxide
    3. hydrocarbon
    4. Particulate matter
  5. Which of the following is the largest sink of CO2 gas?
    1. Forests
    2. Oceans
    3. Ice sheets
    4. Grasslands

June 2019 – Main Paper (Management)

  1. Which pollutant is the major source of marine pollution?
    1. Agricultural run-off
    2. Oil spill
    3. Industrial waste water
    4. Sewage
  2. Which of the following statements best explains the concept of sustainable development ?
    1. Use of natural resources minimally
    2. Use of natural resources for our benefit
    3. Use of natural resources judiciously so that they are available for future generations
    4. Preserve our natural resources for our future generation
  3. The target set by Indian government for generating power from wind energy by the year 2022 is
    1. 60 GW
    2. 50 GW
    3. 40 GW
    4. 15 GW
  4. Permafrost is defining characteristic of which biome?
    1. Taiga
    2. Tundra
    3. Grassland
    4. Desert
  5. The mangroves exist in
    1. Fresh water system
    2. Temperate areas
    3. Semi-arid areas
    4. Saline waters
  6. Chemical oxygen demand in eutrophic water is
    1. Low
    2. High
    3. Medium
    4. Extremely low

December 2018 – Main Paper (Commerce)

December 2018 – Main Paper (Management)

November 2017

January 2017

July 2016

December 2015

June 2015

December 2014

June 2014

December 2013

September 2013

June 2013

December 2012

June 2012

December 2011

June 2011

December 2010

June 2010

December 2009

June 2009

  1. The longitudinal approach of research deals with:
    1 Horizontal researches
    2 Long-term researches
    3 Short-term researches
    4 None of the above
  2. Which of the following is not a common element of a research proposal and a research report ?
    1 Objectives and hypotheses
    2 Implications for further research
    3 References to previous research
    3 Sources of information/data
  3. In the process of statistical inference, the Type II Error is committed when we
    1 Accept a true null hypothesis
    2 Reject a true null hypothesis
    3 Reject a false null hypothesis
    4 Accept a false null hypothesis
  4. If a researchers conducts a research on finding out which administrative style contributes more to institutional effectiveness. This will be an example of
    1 Basic Research
    2 Action Research
    3 Applied Research
    3 Fundamental Research
  5. The core ingredients of a dissertation are:
    1 Introduction; Data collection; Data analysis; Conclusions and recommendations.
    2 Executive summary; Literature review; Data gathered; Conclusions; Bibliography.
    3 Research plan; Research data; Analysis; References.
    4 Introduction; Literature review; Research methods; Results; Discussion; Conclusion
  6. The research antagonistic to ex-post facto research is
    1 experimental studies
    2 library researches
    3 normative researches
    4 all of the above
  7. What is positivism?
    1 Positivism is an epistemological position that advocates the application of the methods of the natural sciences to the study of social reality.
    2 Positivism involves a epistemological process between theory and the empirical phenomenon; this results in the production of ‘reflexive narratives, not explanatory models or theoretical propositions’.
    3 Positivism is a way of thinking about the research process and staying positive.
    4 Positivism is an outlook in life about seeing things positively.
  8. If your research philosophy reflects the principles of positivism, then you will probably:
    1 not accept any argument about your research conclusions.
    2 adopt the philosophical stance of the natural scientist.
    3 want to be sure that all your answers are beyond dispute.
    4 reject the possibility of working with numbers
  9. The branch of philosophy that deals with the issue of what is knowledge is referred to as?
    1 Positivism
    2 Post-positivism
    3 Ontology
    4 Epistemology
  10. The word paradigm means:
    1 a theoretical framework.
    2 being forced to do something.
    3 a type of sampling.
    4 a branch of physics.
  11. What is triangulation?
    1 Using three quantitative or three qualitative methods in a project.
    2 Cross-checking the results found by different research strategies.
    3 Allowing theoretical concepts to emerge from the data.
    4 Drawing a triangular diagram to represent the relations between three participants
  12. Which of the following is a non-probability sample?
    1 Quota sample
    2 Cluster sample
    3 Both 1 and 2
    4 None of these
  13. Hypothesis can not be stated in
    1 null and question form terms
    2 declarative terms
    3 general terms
    4 directional terms
  14. Why is it important that personal data about research participants are kept within secure, confidential records?
    1 So that the participants cannot find out what has been written about them.
    2 In case individuals, places, or organizations can be harmed through identification or disclosure of personal information.
    3 So that government officials, teachers, and other people in authority can have easy access to the data.
    4 To enable the researcher to track down individuals and find out more about their lives.
  15. Which method is most commonly associated with a lack of informed consent?
    1 Qualitative content analysis
    2 In-depth interviewing
    3 Covert observation
    4 Structured interviewing
  16. What problem does a research organization face when drawing up an ethical code?
    1 Identifying relevant legislation that should guide behaviour.
    2 Reflecting the difficulty of making truly ethical decisions.
    3 Incorporating assessments for the ethical behaviour of participants.
    4 All of the above.
  17. According to APA format, the list of references which appear at the end of your paper is
    called
    1 Bibliography
    2 Works Cited
    3 Sources used
    4 List of references
  18. Identify the correct parenthetical citation for a quote that comes from page seven in this
    book:
    Tannen, D. (1998). The argument culture. Toronto, Ontario, Canada: Random House.
    1 Every issue we see discussed on television appears to be set up as an argument: “In the argument culture, criticism, attack, or opposition are the predominant if not the only ways of responding to people or ideas” (Tannen, 1998).
    2 Every issue we see discussed on television appears to be set up as an argument: “In the argument culture, criticism, attack, or opposition are the predominant if not the only ways of responding to people or ideas” (Tannen, 1998, p. 7).
    3 Every issue we see discussed on television appears to be set up as an argument: “In the argument culture, criticism, attack, or opposition are the predominant if not the only ways of responding to people or ideas” (The argument culture, 1998, p. 7).
    4 Every issue we see discussed on television appears to be set up as an argument: “In the argument culture, criticism, attack, or opposition are the predominant if not the only ways of responding to people or ideas” (Tannen 7).
  19. Which of the following is not a database containing information to be used during the literature review?
    1 ERIC
    2 PsychINFO
    3 SocioFILE
    4 All of the above are potentially useful databases
  20. Which research paradigm is based on the pragmatic view of reality?
    1 quantitative research
    2 qualitative research
    3 mixed research
    4 none of the above
  21. Good ‘research ethics’ means
    1 Not disclosing the holdings of shares/stocks in a company that sponsors your research.
    2 Assigning a particular research problem to one Ph.D./research student only.
    3 Discussing with your colleagues confidential data from a research paper that you are reviewing for an academic journal.
    4 Submitting the same research manuscript for publishing in more than one journal.
  22. Which of the following are the characteristics of a seminar?
    (a) It is a form of academic instruction.
    (b) It involves questioning, discussion and debates.
    (c) It involves large groups of individuals.
    (d) It needs involvement of skilled persons.
    Select the correct answer from the codes given below :
    1 (a), (b) and (c)
    2 (b), (c) and (d)
    3 (a), (b) and (d)
    4 (a), (c) and (d)
  23. Normal Probability Curve
    1 is positively skewed
    2 is negatively skewed
    3 is not skewed
    4 has a right tail
  24. Which of the following steps are required to design a questionnaire ?
    (a) Writing primary and secondary aims of the study.
    (b) Review of the current literature.
    (c) Prepare a draft of questionnaire.
    (d) Revision of the draft.
    Select the correct answer from the codes given below :
    1 (a), (b) and (c)
    2 (a), (c) and (d)
    3 (b), (c) and (d)
    4 (a), (b), (c) and (d)
  25. Arrange the following steps of research in correct sequence :
    a. Identification of research problem
    b. Listing of research objectives
    c. Collection of data
    d. Methodology
    e. Data analysis
    f. Results and discussion
    1 a – b – c – d – e – f
    2 a – b – d – c – e – f
    3 b – a – c – d – e – f
    3 b – a – d – c – e – f
Share to your friend

Research Aptitude 

Previous Year Question Paper 2009 to June 2019

June 2019 – Main Paper (Commerce)

  1. A tentative proposition with unknown validity that specifies a relations more variables is called
    1. research problem
    2. Research proposal
    3. Research design
    4. Research hypothesis
  2. Snowball sampling is the process of selecting a sample using
    1. Networks
    2. Groups
    3. Snowballs
    4. Computer Programs
  3. In which of the following research paradigms the focus of concern is on interpreting reality in terms of participants perspective ?
    1. Experimental Research
    2. Ex-post facto Research
    3. Ethnographic Research
    4. Survey Research
  4. Research design that needs to collect factual information on a continuing basis is classified as
    1. Longitudinal Study Design
    2. Pre and Post Study Design
    3. Cross-Sectional Study Design
    4. Double-Control Study Design

June 2019 – Main Paper (Management)

  1. Peer group interaction in a classroom helps in
    1. Concept understanding
    2. Realization of One’s misunderstanding
    3. Rapport Building
    4. Questioning
  2. Inclusive education implies
    1. Ensuring learning outcome of every child to be the same
    2. Including the disabled in the main stream
    3. Provides compulsory education for children below 14 years
    4. Ensuring that no child is left behind in education
  3. Listening is important for a teacher to
    1. address the requirement of the learner
    2. gain knowledge
    3. keep the learner in good humour
    4. evaluate the learner for the progress made
  4. Reflective Practice implies use of one’s
    1. Cognition
    2. Metacognition
    3. Metacognition on cognition
    4. Reinforcement of learning
  5. When the learning outcomes of students are ensured with employment of less resources and effort on the part of a teacher and more initiative for self-learning is evident, which of the following expression will describe the teacher most appropriately?
    1. Teacher is successful
    2. Teacher is effective
    3. Teacher is intelligent
    4. Teacher is practical

December 2018 – Main Paper (Commerce)

  1. In which of the following, target related specifications will be considered necessary ?
    1. Seminars
    2. Symposium
    3. Workshops
    4. Conferences
  2. In the two sets given below. Set-I embodies the list of research types while Set-II provides their nature and characteristics.
    Match the two sets and give your answer from the code which follows :
    Set-I(Research types)
    (a) Experimental research
    (b) Expost facto research
    (c) Participant observation based research
    (d) Grounded theory approach based research
    Set-II (Nature of characteristics)
    (i) Generalizations follow rather than precede the observations in due course
    (ii) Emphasizing the access to reality by sharing the perspectives of people
    (iii) Getting at causal facts by retrospective analysis
    (iv) Describing the status and conditions as obtainable
    (v) Controlling the extraneous variables and observing the effect of independent variable manipulated by the researcher on dependent variable
    a b c d
    1. (i) (ii) (iii) (iv)
    2. (iii) (i) (iv) (ii)
    3. (ii) (iii) (iv) (v)
    4. (v) (iii) (ii) (i)
  3. Which of the following research methods the emphasis is laid on naturalistic settings and meaning-giving process ?
    1. Case study method
    2. Descriptive survey method
    3. Experimental method
    4. Ex post facto method
  4. Which of the following implies the correct sequence in an action research paradigm?
    1. Act, reflect, plan and observe
    2. Observe, reflect, plan and act
    3. Plan, act, observe and reflect
    4. Reflect, act, observe and plan
  5. The issue of research ethics is pertinent at which of the following stages of research ?
    1. Deciding the quantitative or qualitative tracks or both for pursuing the research problem
    2. Data collection, data analysis and reporting of research findings/results
    3. Identification, definition and delimitation of research problem
    4. Defining the population, sampling procedures and techniques

December 2018 – Main Paper (Management)

  1. The four approaches to knowing answers to research questions, according to Kerlinger, are :
    (a) Method of tenacity
    (b) Method of intuition
    (c) Method of authority
    (d) Method of science
    (e) Method of creativity
    (f) Method of non-functionality
    Code :
    1. (a), (b), (c) and (d)
    2.(c), (d), (e) and (f)
    3.(b), (c), (d) and (e)
    4.(b), (d), (e) and (f)
  2. Which method is appropriate to reduce random error ?
    1. Administer the test after a gap of one week
    2. Select large sample size
    3. Administer the treatment in the same way every time
    4. Administer manipulation checks
  3. One way to measure the extent to which a measure is free of random error is to compute its :
    1. Content validity
    2. Test-retest reliability
    3. Experimenter’s bias
    4. Demand characteristics
  4. Qualitative researchers criticise quantitative research for its tendency towards :
    1. Emphasis on evidence
    2. Reductionism
    3. Obsession with creativity
    4. Focus on empiricism
  5. In relation to code of conduct in research, what does APA stand for ?
    1. American Psychological Association
    2. Association of Psychological Assessments
    3. Advanced Psychological Authority
    4. Australian Psychological Association

July 2018

  1. There are two sets given below.
    Set – I specifies the types of research, while Set – II indicates their characteristics.
    Match the two and give your answer by selecting the appropriate code.
    Set – I (Research types)
    (a) Fundamental research
    (b) Applied research
    (c) Action research
    (d) Evaluative research
    Set – II (Characteristics)
    (i) Finding out the extent of perceived impact of an intervention
    (ii) Developing an effective explanation through theory building
    (iii) Improving an existing situation through use of interventions
    (iv) Exploring the possibility of a theory for use in various situations
    (v) Enriching technological resources
    Code :
    (a) (b) (c) (d)
    (1) (ii) (iv) (iii) (i)
    (2) (v) (iv) (iii) (ii)
    (3) (i) (ii) (iii) (iv)
    (4) (ii) (iii) (iv) (v)
  2. Which of the sets of activities best indicate the cyclic nature of action research strategy ?
    (1) Reflect, Observe, Plan, Act
    (2) Observe, Act, Reflect, Plan
    (3) Act, Plan, Observe, Reflect
    (4) Plan, Act, Observe, Reflect
  3. Which of the following sequences of research steps is nearer to scientific method ?
    (1) Suggested solution of the problem, Deducing the consequences of the solution, Perceiving the problem situation, Location of the difficulty and testing the solutions.
    (2) Perceiving the problem situation, Locating the actual problem and its definition, Hypothesizing, Deducing the consequences of the suggested solution and Testing the hypothesis in action.
    (3) Defining a problem, Identifying the causes of the problem, Defining a population, Drawing a sample, Collecting data and Analysing results.
    (4) Identifying the causal factors, Defining the problem, Developing a hypothesis, Selecting a sample, Collecting data and arriving at generalizations and Conclusions.
  4. The problem of ‘research ethics’ is concerned with which aspect of research activities ?
    (1) Following the prescribed format of a thesis
    (2) Data analysis through qualitative or quantitative techniques
    (3) Defining the population of research
    (4) Evidence based research reporting
  5. In which of the following activities, potential for nurturing creative and critical thinking is relatively greater ?
    (1) Preparing research summary
    (2) Presenting a seminar paper
    (3) Participation in research conference
    (4) Participation in a workshop

November 2017

  1. Which of the following research types focuses on ameliorating the prevailing situations ?
    (1) Fundamental Research
    (2) Applied Research
    (3) Action Research
    (4) Experimental Research
  2. A researcher attempts to evaluate the effect of method of feeding on anxiety – proneness of children. Which method of research would be appropriate for this ?
    (1) Case study method
    (2) Experimental method
    (3) Ex-post-facto method
    (4) Survey metho
  3. In which of the following arrangements a wider spectrum of ideas and issues may be made possible ?
    (1) Research Article
    (2) Workshop mode
    (3) Conference
    (4) Symposium
  4. In finalizing a thesis writing format which of the following would form part of supplementary pages ?
    (1) List of tables and figures
    (2) Table of contents
    (3) Conclusions of the study
    (4) Bibliography and Appendices
  5. Which of the following is susceptible to the issue of research ethics ?
    (1) Inaccurate application of statistical techniques
    (2) Faulty research design
    (3) Choice of sampling techniques
    (4) Reporting of research findings

January 2017

  1. The principal of a school conducts an interview session of teachers and students with a view to explore the possibility of their enhanced participation in school programmes. This endeavour may be related to which type of research ?
    (1) Evaluation Research
    (2) Fundamental Research
    (3) Action Research
    (4) Applied Research
  2. In doing action research what is the usual sequence of steps ?
    (1) Reflect, observe, plan, act
    (2) Plan, act, observe, reflect
    (3) Plan, reflect, observe, act
    (4) Act, observe, plan, reflect
  3. Which sequence of research steps is logical in the list given below ?
    (1) Problem formulation, Analysis, Development of Research design, Hypothesis making, Collection of data, Arriving at generalizations and conclusions.
    (2) Development of Research design, Hypothesis making, Problem formulation, Data analysis, Arriving at conclusions and data collection.
    (3) Problem formulation, Hypothesis making, Development of a Research design, Collection of data, Data analysis and formulation of generalizations and conclusions.
    (4) Problem formulation, Deciding about the sample and data collection tools,
    Formulation of hypothesis, Collection and interpretation of research evidence.
  4. Below are given two sets – research methods (Set-I) and data collection tools (Set-II). Match the two sets and indicate your answer by selecting the correct code :
    Set – I (Research Methods)
    a. Experimental method
    b. Ex post-facto method
    c. Descriptive survey method
    d. Historical method
    Set – II (Data Collection Tools)
    i. Using primary and secondary sources
    ii. Questionnaire
    iii. Standardized tests
    iv. Typical characteristic tests
    Codes :
    a b c d
    (1) ii i iii iv
    (2) iii iv ii i
    (3) ii iii i iv
    (4) ii iv iii i
  5. The issue of ‘research ethics’ may be considered pertinent at which stage of research ?
    (1) At the stage of problem formulation and its definition
    (2) At the stage of defining the population of research
    (3) At the stage of data collection and interpretation
    (4) At the stage of reporting the findings.
  6. In which of the following, reporting format is formally prescribed ?
    (1) Doctoral level thesis
    (2) Conference of researchers
    (3) Workshops and seminars
    (4) Symposia

August 2016

  1. In a Ph.D. thesis which one is the correct sequence for showing scheme of Chapterisation ?
    (1) Survey of related studies, Introduction, Design of the study, Data-Analysis and interpretation, Conclusions and generalisations, suggested further study, References, Appendix
    (2) Introduction, Design of study, Data Analysis and Interpretation, Generalizations, Conclusions and Survey of related studies and suggestions for further research, References and Appendix
    (3) Introduction, Survey of related studies, Design of study, Data-presentation; analysis & Interpretation, Formulation of generalization & Conclusions, Suggestions for further research, References & Appendix
    (4) Survey of related studies, References, Introduction, Design of study, Data analysis and interpretation, Conclusions and generalizations, Suggestions for further research, Appendix
  2. Which of the following is not the critical feature of qualitative research ?
    (1) Actual settings are the direct source of data.
    (2) Data take the forms of words or pictures.
    (3) Seeking to establish relationships among measured social facts.
    (4) Researcher becomes immersed in the situation, present or past related to the phenomena.
  3. A detailed description of methodology of research is required in
    (1) Thesis/Dissertation
    (2) Symposium/Workshop
    (3) Seminar paper/Articles
    (4) Conference and Seminar Papers
  4. Research ethics has a direct connection more often with which stages of research ?
    (1) Defining and delimiting the scope of research.
    (2) Problem formulation and reporting of research findings.
    (3) Defining the population and deciding the sampling technique for research.
    (4) Deciding about statistical techniques and data analysis.
  5. The purpose of formative evaluation is to
    (1) grade students’ learning outcomes.
    (2) accelerate students’ learning performance.
    (3) check students’ performance graph.
    (4) provide feedback to teacher effectiveness.
  6. There are two sets given below, Set-I consists of methods of research and Set-II indicates their procedural characteristics.
    Match the two sets and choose your answer from the codes given below :
    Set – I (Research method)
    a. Experimental method
    b. Expost facto method
    c. Descriptive method
    d. Symbolic interactionism
    e. Action research
    Set – II(Procedural Characteristics)
    i. Interventions to ameliorate a given situation
    ii.Explaining patterns of behaviour in terms of meanings and their representations which people share.
    iii. Manipulating an independent variable in controlled conditions and measuring its effect on dependent variable.
    iv. Depending on primary and secondary sources of data.
    v. Obtaining information concerning the current status of phenomena.
    vi. Observing the effect on dependent variable and making probes into factors/variables which explain it.
    vii. Exegetic analysis.
    Codes :
    a b c d e
    (1) i iii iv v vii
    (2) iii iv v vi vii
    (3) i ii iii iv v
    (4) iii vi v ii i

July 2016

  1. Match the items of the first set with that of the second set in respect of evaluation system.Choose the correct code :
    Set – I
    a. Formative evaluation
    b. Summative evaluation
    c. Continuous and comprehensive evaluation
    d. Norm and criterion referenced test
    Set – II
    i. Evaluating cognitive and co-cognitive aspects with regularity
    ii. Tests and their interpretations based on a group and certain yardsticks
    iii. Grading the final learning outcomes
    iv. Quizzes and discussions
    Codes :
    a b c d
    (1) iv iii i ii
    (2) i ii iii iv
    (3) iii iv ii i
    (4) i iii iv ii
  2. A researcher intends to explore the effect of possible factors for the organization of effective mid-day meal interventions. Which research method will be most appropriate for this study ?
    (1) Historical method
    (2) Descriptive survey method
    (3) Experimental method
    (4) Ex-post-facto method
  3. Which of the following is an initial mandatory requirement for pursuing research ?(1) Developing a research design
    (2) Formulating a research question
    (3) Deciding about the data analysis procedure
    (4) Formulating a research hypothesis
  4. The format of thesis writing is the same as in(1) preparation of a research paper/article
    (2) writing of seminar presentation
    (3) a research dissertation
    (4) presenting a workshop / conference paper
  5. In qualitative research paradigm, which of the following features may be considered critical ?
    (1) Data collection with standardised research tools.
    (2) Sampling design with probability sample techniques.
    (3) Data collection with bottom-up empirical evidences.
    (4) Data gathering to take place with top-down systematic evidences.
  6. From the following list of statements identify the set which has negative implications for ‘research ethics’ :
    (i) A researcher critically looks at the findings of another research.
    (ii) Related studies are cited without proper references.
    (iii) Research findings are made the basis for policy making.
    (iv) Conduct of practitioner is screened in terms of reported research evidences.
    (v) A research study is replicated with a view to verify the evidences from other researches.
    (vi) Both policy making and policy implementing processes are regulated in terms of preliminary studies.
    Codes :
    (1) (i), (ii) and (iii)
    (2) (ii), (iii) and (iv)
    (3) (ii), (iv) and (vi)
    (4) (i), (iii) and (v)
  7. In a research on the effect of child-rearing practices on stress-proneness of children in completing school projects, the hypothesis formulated is that ‘child rearing practices do influence stress-proneness’. At the data-analysis stage a null hypothesis is advanced to find out the tenability of research hypothesis. On the basis of the evidence available, the null hypothesis is rejected at 0.01 level of significance. What decision may be warranted in respect of the research hypothesis ?
    (1) The research hypothesis will also be rejected.
    (2) The research hypothesis will be accepted.
    (3) Both the research hypothesis and the null hypothesis will be rejected.
    (4) No decision can be taken in respect of the research hypothesis.

December 2015

  1. What are the characteristics of Continuous and Comprehensive Evaluation ?
    (a) It increases the workload on students by taking multiple tests.
    (b) It replaces marks with grades.
    (c) It evaluates every aspect of the student.
    (d) It helps in reducing examination phobia.
    Select the correct answer from the codes given below :
    (1) (a), (b), (c) and (d)
    (2) (b) and (d)
    (3) (a), (b) and (c)
    (4) (b), (c) and (d)
  2. Which of the following statements is not true in the context of participatory research ?
    (1) It recognizes knowledge as power.
    (2) It emphasises on people as experts.
    (3) It is a collective process of enquiry.
    (4) Its sole purpose is production of knowledge.
  3. Which of the following statements is true in the context of the testing of a hypothesis ?
    (1) It is only the alternative hypothesis, that can be tested.
    (2) It is only the null hypothesis, that can be tested.
    (3) Both, the alternative and the null hypotheses can be tested.
    (4) Both, the alternative and the null hypotheses cannot be tested.
  4. Which of the following are the basic rules of APA style of referencing format ?
    (a) Italicize titles of shorter works such as journal articles or essays
    (b) Invert authors’ names (last name first)
    (c) Italicize titles of longer works such as books and journals
    (d) Alphabetically index reference list
    Select the correct answer from the codes given below :
    (1) (a) and (b)
    (2) (b), (c) and (d)
    (3) (c) and (d)
    (4) (a), (b), (c) and (d)
  5. Which of the following are the characteristics of a seminar ?
    (a) It is a form of academic instruction.
    (b) It involves questioning, discussion and debates.
    (c) It involves large groups of individuals.
    (d) It needs involvement of skilled persons.
    Select the correct answer from the codes given below :
    (1) (b) and (c)
    (2) (b) and (d)
    (3) (b), (c) and (d)
    (4) (a), (b) and (d)
  6. A researcher is interested in studying the prospects of a particular political party in an urban area. What tool should he prefer for the study ?
    (1) Rating scale
    (2) Interview
    (3) Questionnaire
    (4) Schedule
  7. Ethical norms in research do not involve guidelines for :
    (1) Thesis format
    (2) Copyright
    (3) Patenting policy
    (4) Data sharing policies

June 2015

  1. Which of the following statements regarding the meaning of research are correct ?(a) Research refers to a series of systematic activity or activities undertaken to find out the solution of a problem.
    (b) It is a systematic, logical and an unbiased process wherein verification of hypothesis, data analysis, interpretation and formation of principles can be done.
    (c) It is an intellectual enquiry or quest towards truth.
    (d) It leads to enhancement of knowledge.
    Select the correct answer from the codes given below :
    (1) (a), (b) and (c)
    (2) (b), (c) and (d)
    (3) (a), (c) and (d)
    (4) (a), (b), (c) and (d)
  2. A good thesis writing should involve :
    (a) reduction of punctuation and grammatical errors to a minimum.
    (b) careful checking of references.
    (c) consistency in the way the thesis is written.
    (d) a clear and well written abstract.
    Select the correct answer from the codes given below :
    (1) (a), (b), (c) and (d)
    (2) (a), (b) and (c)
    (3) (a), (b) and (d)
    (4) (b), (c) and (d)
  3. Jean Piaget gave a theory of cognitive development of humans on the basis of his :
    (1) Fundamental Research
    (2) Applied Research
    (3) Action Research
    (4) Evaluation Research
  4. “Male and female students perform equally well in a numerical aptitude test.” This statement indicates a :
    (1) research hypothesis
    (2) null hypothesis
    (3) directional hypothesis
    (4) statistical hypothesis
  5. The conclusions/findings of which type of research cannot be generalized to other situations ?
    (1) Historical Research
    (2) Descriptive Research
    (3) Experimental Research
    (4) Causal Comparative Research
  6. Which of the following steps are required to design a questionnaire ?
    (a) Writing primary and secondary aims of the study.
    (b) Review of the current literature.
    (c) Prepare a draft of questionnaire.
    (d) Revision of the draft.
    Select the correct answer from the codes given below :
    (1) (a), (b) and (c)
    (2) (a), (c) and (d)
    (3) (b), (c) and (d)
    (4) (a), (b), (c) and (d)

December 2014

  1. Diagnostic evaluation ascertains
    (A) Students performance at the beginning of instructions.
    (B) Learning progress and failures during instructions.
    (C) Degree of achievement of instructions at the end.
    (D) Causes and remedies of persistent learning problems during instructions.
  2. When planning to do as social research, it is better to
    (A) approach the topic with an open mind
    (B) do a pilot study before getting stuck into it
    (C) be familiar with literature on the topic
    (D) forget about theory because this is a very practical
  3. When academicians are called to deliver lecture or presentation to an audience on certain topics or a set of topics of educational nature, it is called
    (A) Training Program
    (B) Seminar
    (C) Workshop
    (D) Symposium
  4. The core elements of a dissertation are
    (A) Introduction; Data Collection; Data Analysis; Conclusions and Recommendations
    (B) Executive Summary; Literature review; Data gathered; Conclusions; Bibliography
    (C) Research Plan; Research Data; Analysis; References
    (D) Introduction; Literature Review; Research Methodology; Results; Discussion and Conclusion
  5. What is a Research Design ?
    (A) A way of conducting research that is not grounded in theory.
    (B) The choice between using qualitative or quantitative methods.
    (C) The style in which you present your research findings e.g. a graph.
    (D) A framework for every stage of the collection and analysis of data.
  6. ‘Sampling Cases’ means
    (A) Sampling using a sampling frame
    (B) Identifying people who are suitable for research
    (C) Literally the researcher’s brief case
    (D) Sampling of people, newspapers, television programmes etc.
  7. The frequency distribution of a research data which is symmetrical in shape similar to a normal distribution but center peak is much higher, is
    (A) Skewed
    (B) Mesokurtic
    (C) Leptokurtic
    (D) Platykurtic

June 2014

  1. Who among the following, propounded the concept of paradigm ?
    (A) Peter Haggett
    (B) Von Thunen
    (C) Thomas Kuhn
    (D) John K. Wright
  2. In a thesis, figures and tables are included in
    (A) the appendix
    (B) a separate chapter
    (C) the concluding chapter
    (D) the text itself
  3. A thesis statement is
    (A) an observation
    (B) a fact
    (C) an assertion
    (D) a discussion
  4. The research approach of Max Weber to understand how people create meanings in natural settings is identified as
    (A) positive paradigm
    (B) critical paradigm
    (C) natural paradigm
    (D) interpretative paradigm
  5. Which one of the following is a non- probability sampling ?
    (A) Simple random
    (B) Purposive
    (C) Systematic
    (D) Stratified
  6. Identify the category of evaluation that assesses the learning progress to provide continuous feedback to the students during instruction.
    (A) Placement
    (B) Diagnostic
    (C) Formative
    (D) Summative
  7. The research stream of immediate application is
    (A) Conceptual research
    (B) Action research
    (C) Fundamental research
    (D) Empirical research

December 2013

  1. Which is the main objective of research ?
    (A) To review the literature
    (B) To summarize what is already known
    (C) To get an academic degree
    (D) To discover new facts or to make fresh interpretation of known facts
  2. Sampling error decreases with the
    (A) decrease in sample size
    (B) increase in sample size
    (C) process of randomization
    (D) process of analysis
  3. The principles of fundamental research are used in
    (A) action research
    (B) applied research
    (C) philosophical research
    (D) historical research
  4. The population information is called parameter while the corresponding sample information is known as
    (A) Universe
    (B) Inference
    (C) Sampling design
    (D) Statistics
  5. The sequential operations in scientific research are
    (A) Co-variation, Elimination of Spurious Relations, Generalisation, Theorisation
    (B) Generalisation, Co-variation, Theorisation, Elimination of Spurious Relations
    (C) Theorisation, Generalisation, Elimination of Spurious Relations, Co-variation
    (D) Elimination of Spurious Relations, Theorisation, Generalisation, Co-variation.
  6. In sampling, the lottery method is used for
    (A) Interpretation
    (B) Theorisation
    (C) Conceptualisation
    (D) Randomisation

September 2013

  1. Which one of the following is not a type of experimental method ?
    (A) Single group experiment
    (B) Residual group experiment
    (C) Parallel group experiment
    (D) Rational group experiment
  2. Which one of the following is not a non-parametric test ?
    (A) t-test
    (B) Sign test
    (C) Chi-square test
    (D) Run test
  3. Read the following statements – one labelled as Assertion (A) and the other as Reason (R) :
    Assertion (A) : Qualitative research in India has a long way to go to reach international standards.
    Reason (R) : Because the funding agencies do not support qualitative research in academic institutions.
    Find the correct answer from the codes given below :
    (A) Both (A) and (R) are true and (R) is the correct explanation of (A).
    (B) Both (A) and (R) are true, but (R) is not the correct explanation of (A).
    (C) (A) is true, but (R) is false.
    (D) Both (A) and (R) are false.
  4. Identify the correct sequence of research steps :(A) Selection of topic, review of literature, data collection, interpretation of findings
    (B) Review of literature, selection of topic, data collection, interpretation of findings
    (C) Selection of topic, data collection, review of literature, interpretation of findings
    (D) Selection of topic, review of literature, interpretation of findings, data collection
  5. Deconstruction is a popular method of research in(A) Basic Science
    (B) Applied Science
    (C) Social Science
    (D) Literature
  6. With which one of the following techniques communality is associated ?(A) Univariate analysis
    (B) Factor analysis
    (C) Case studies
    (D) SWOT analysis
  7. The variable which impacts the relationship between an independent variable and a dependent variable is known as(A) antecedent variable
    (B) precedent variable
    (C) predictor variable
    (D) control variable
  8. Which one of the following is a non- probability sampling method ?(A) Simple Random Sampling
    (B) Stratified Sampling
    (C) Cluster Sampling
    (D) Quota Sampling

June 2013

  1. Which one of the following references is written as per Modern Language Association (MLA) format ?(A) Hall, Donald. Fundamentals of Electronics, New Delhi : Prentice Hall of India, 2005
    (B) Hall, Donald, Fundamentals of Electronics, New Delhi : Prentice Hall of India, 2005
    (C) Hall, Donald, Fundamentals of Electronics, New Delhi – Prentice Hall of India, 2005
    (D) Hall, Donald. Fundamentals of Electronics. New Delhi : Prentice Hall of India, 2005
  2. A workshop is(A) a conference for discussion on a topic.
    (B) a meeting for discussion on a topic.
    (C) a class at a college or a university in which a teacher and the students discuss a topic.
    (D) a brief intensive course for a small group emphasizing the development of a skill or technique for solving a specific problem.
  3. A working hypothesis is(A) a proven hypothesis for an argument.
    (B) not required to be tested.
    (C) a provisionally accepted hypothesis for further research.
    (D) a scientific theory.
  4. A research paper(A) is a compilation of information on a topic.
    (B) contains original research as deemed by the author.
    (C) contains peer-reviewed original research or evaluation of research conducted by others.
    (D) can be published in more than
  5. Which one of the following belongs to the category of good ‘research ethics’ ?
    (A) Publishing the same paper in two research journals without telling the editors.
    (B) Conducting a review of the literature that acknowledges the contributions of other people in the relevant field or relevant prior work.
    (C) Trimming outliers from a data set without discussing your reasons in a research paper.
    (D) Including a colleague as an author on a research paper in return for a favour even though the colleague did not make a serious contribution to the paper.
  6. Which of the following sampling methods is not based on probability ?(A) Simple Random Sampling
    (B) Stratified Sampling
    (C) Quota Sampling
    (D) Cluster Sampling

December 2012

  1. Which one of the following is an indication of the quality of a research journal ?(A) Impact factor
    (B) h-index
    (C) g-index
    (D) i10-index
  2. Good ‘research ethics’ means
    (A) Not disclosing the holdings of shares/stocks in a company that sponsors your research.
    (B) Assigning a particular research problem to one Ph.D./research student only.
    (C) Discussing with your colleagues confidential data from a research paper that you are reviewing for an academic journal.
    (D) Submitting the same research manuscript for publishing in more than one journal.
  3. Which of the following sampling methods is based on probability ?
    (A) Convenience sampling
    (B) Quota sampling
    (C) Judgement sampling
    (D) Stratified sampling
  4. Which one of the following references is written according to American Psychological Association (APA) format ?(A) Sharma, V. (2010). Fundamentals of Computer Science.New Delhi : Tata McGraw Hill
    (B) Sharma, V. 2010. Fundamentals of Computer Science.New Delhi : Tata McGraw Hill
    (C) Sharma.V. 2010. Fundamentals of Computer Science, New Delhi : Tata McGraw Hill
    (D) Sharma, V. (2010), Fundamentals of Computer Science, New Delhi : Tata McGraw Hill
  5. Arrange the following steps of research in correct sequence :
    1. Identification of research problem
    2. Listing of research objectives
    3. Collection of data
    4. Methodology
    5. Data analysis
    6. Results and discussion
    (A) 1 – 2 – 3 – 4 – 5 – 6
    (B) 1 – 2 – 4 – 3 – 5 – 6
    (C) 2 – 1 – 3 – 4 – 5 – 6
    (D) 2 – 1 – 4 – 3 – 5 – 6
  6. Identify the incorrect statement :
    (A) A hypothesis is made on the basis of limited evidence as a starting point for further investigations.
    (B) A hypothesis is a basis for reasoning without any assumption of its truth.
    (C) Hypothesis is a proposed explanation for a phenomenon.
    (D) Scientific hypothesis is a scientific theory.

June 2012

  1. The research that aims at immediate application is
    (A) Action Research
    (B) Empirical Research
    (C) Conceptual Research
    (D) Fundamental Research
  2. When two or more successive footnotes refer to the same work which one of the following expressions is used ?
    (A) ibid
    (B) et.al
    (C) op.cit :
    (D) loc.cit.
  3. Nine year olds are taller than seven year olds. This is an example of a reference drawn from
    (A) Vertical study
    (B) Cross-sectional study
    (C) Time series study
    (D) Experimental study
  4. Conferences are meant for
    (A) Multiple target groups
    (B) Group discussions
    (C) Show-casing new Research
    (D) All the above
  5. Ex Post Facto research means
    (A) The research is carried out after the incident
    (B) The research is carried out prior to the incident
    (C) The research is carried out along with the happening of an incident.
    (D) The research is carried out keeping in mind the possibilities of an incident.
  6. Research ethics do not include
    (A) Honesty
    (B) Subjectivity
    (C) Integrity
    (D) Objectivity

December 2011

  1. A hypothesis is a
    (A) law
    (B) canon
    (C) postulate
    (D) supposition
  2. Suppose you want to investigate the working efficiency of nationalised bank in India, which one of the following would you follow ?
    (A) Area Sampling
    (B) Multi-stage Sampling
    (C) Sequential Sampling
    (D) Quota Sampling
  3. Controlled group condition is applied in
    (A) Survey Research
    (B) Historical Research
    (C) Experimental Research
    (D) Descriptive Research
  4. Workshops are meant for
    (A) giving lectures
    (B) multiple target groups
    (C) showcase new theories
    (D) hands on training/experience
  5. Which one of the following is a research tool ?
    (A) Graph
    (B) Illustration
    (C) Questionnaire
    (D) Diagram
  6. Research is not considered ethical if it
    (A) tries to prove a particular point.
    (B) does not ensure privacy and anonymity of the respondent.
    (C) does not investigate the data scientifically.
    (D) is not of a very high standard.

June 2011

  1. A research paper is a brief report of research work based on
    (A) Primary Data only
    (B) Secondary Data only
    (C) Both Primary and Secondary Data
    (D) None of the above
  2. Newton gave three basic laws of motion. This research is categorized as
    (A) Descriptive Research
    (B) Sample Survey
    (C) Fundamental Research
    (D) Applied Research
  3. A group of experts in a specific area of knowledge assembled at a place and prepared a syllabus for a new course. The process may be termed as
    (A) Seminar
    (B) Workshop
    (C) Conference
    (D) Symposium
  4. In the process of conducting research ‘Formulation of Hypothesis” is followed by
    (A) Statement of Objectives
    (B) Analysis of Data
    (C) Selection of Research Tools
    (D) Collection of Data
  5. The essential qualities of a researcher are
    (A) spirit of free enquiry
    (B) reliance on observation and evidence
    (C) systematization or theorizing of knowledge
    (D) all the above
  6. Research is conducted to
    I. Generate new knowledge
    II. Not to develop a theory
    III. Obtain research degree
    IV. Reinterpret existing knowledgeWhich of the above are correct ?
    (A) I, III & II
    (B) III, II & IV
    (C) II, I & III
    (D) I, III & IV

December 2010

  1. Which of the following variables cannot be expressed in quantitative terms ?
    (A) Socio-economic Status
    (B) Marital Status
    (C) Numerical Aptitude
    (D) Professional Attitude
  2. A doctor studies the relative effectiveness of two drugs of dengue fever. His research would be classified as
    (A) Descriptive Survey
    (B) Experimental Research
    (C) Case Study
    (D) Ethnography
  3. The term ‘phenomenology’ is associated with the process of
    (A) Qualitative Research
    (B) Analysis of Variance
    (C) Correlational Study
    (D) Probability Sampling
  4. The ‘Sociogram’ technique is used to study
    (A) Vocational Interest
    (B) Professional Competence
    (C) Human Relations
    (D) Achievement Motivation
  5. Which of the following phrases is not relevant to describe the meaning of research as a process ?
    (A) Systematic Activity
    (B) Objective Observation
    (C) Trial and Error
    (D) Problem Solving
  6. Which of the following is not an example of a continuous variable ?
    (A) Family size
    (B) Intelligence
    (C) Height
    (D) Attitude

June 2010

  1. Action research means
    (A) A longitudinal research
    (B) An applied research
    (C) A research initiated to solve an immediate problem
    (D) A research with socio- economic objective
  2. Research is
    (A) Searching again and again
    (B) Finding solution to any problem
    (C) Working in a scientific way to search for truth of any problem
    (D) None of the above
  3. A common test in research demands much priority on
    (A) Reliability
    (B) Useability
    (C) Objectivity
    (D) All of the above
  4. Which of the following is the first step in starting the research process ?
    (A) Searching sources of information to locate problem.
    (B) Survey of related literature
    (C) Identification of problem
    (D) Searching for solutions to the problem
  5. If a researcher conducts a research on finding out which administrative style contributes more to institutional effectiveness ? This will be an example of
    (A) Basic Research
    (B) Action Research
    (C) Applied Research
    (D) None of the above
  6. Normal Probability Curve should be
    (A) Positively skewed
    (B) Negatively skewed
    (C) Leptokurtic skewed
    (D) Zero skewed

December 2009

  1. A null hypothesis is
    (A) when there is no difference between the variables
    (B) the same as research hypothesis
    (C) subjective in nature
    (D) when there is difference between the variables
  2. The research which is exploring new facts through the study of the past is called
    (A) Philosophical research
    (B) Historical research
    (C) Mythological research
    (D) Content analysis
  3. Action research is
    (A) An applied research
    (B) A research carried out to solve immediate problems
    (C) A longitudinal research
    (D) Simulative research
  4. The process not needed in Experimental Researches is
    (A) Observation
    (B) Manipulation
    (C) Controlling
    (D) Content Analysis
  5. Manipulation is always a part of
    (A) Historical research
    (B) Fundamental research
    (C) Descriptive research
    (D) Experimental research
  6. Which correlation co-efficient best explains the relationship between creativity and intelligence ?
    (A) 1.00
    (B) 0.6
    (C) 0.5
    (D) 0.3

June 2009

  1. How can the objectivity of the research be enhanced ?
    (A) Through its impartiality
    (B) Through its reliability
    (C) Through its validity
    (D) All of these
  2. Action-research is :(A) An applied research
    (B) A research carried out to solve immediate problems
    (C) A longitudinal research
    (D) All the above
  3. The basis on which assumptions are formulated :(A) Cultural background of the country
    (B) Universities
    (C) Specific characteristics of the castes
    (D) All of these
  4. Which of the following is classified in the category of the developmental research ?
    (A) Philosophical research
    (B) Action research
    (C) Descriptive research
    (D) All the above
  5. We use Factorial Analysis
    (A) To know the relationship between two variables
    (B) To test the Hypothesis
    (C) To know the difference between two variables
    (D) To know the difference among the many variables
Share to your friend

Comprehension

Previous Year Question Paper 2009 to June 2019

June 2019 – Main Paper (Commerce)

An important difference between human beings and animals is the property of language. Animals have a rudimentary information process but this process does not have the sophistication and complexity of human language. If two dogs communicate, they pass on information but they can’t discuss details the way humans can. While humans are blessed to possess the language faculty, most of the time they don’t utilize it to its greatest potential. Perhaps this is because human beings tend to react rather than respond. And a lot of the message of languages is lost in this reaction.
In the daily use of language, people forget the complex process involved in order to put even one word in place. The wonder of this process lies in the fact that it is involuntary in nature, the whole set of chain processes right from the command in the brain to the forming of words from the message required to the movement of the vocal chords and corresponding movements in the lungs and mouth takes place within seconds. Perhaps the whole suggestion of ‘think before you speak’ may lead to a silent world!
In the world of daily interaction, language becomes a marker and a means of projecting identity. The people around us identify us with a certain style of using language and unknowingly, each language user follows this style by habit. It is not a bad idea to record one’s spoken communication from time to time, as this will assist in removing any unwanted elements that the speech may be expressing. Similarly, it is a good idea to step back once in a while and read one’s written communication objectively.

  1. Animals do not possess language because
    1. the power of speech is missing
    2. the life breath is missing
    3. the level of complexity is missing
    4. the information-sharing is missing
  2. When human beings react rather than respond
    1. The lose the message
    2. The lose control
    3. They become emotional
    4. They can’t speak
  3. The world will become a silent place if people will think
    1. too much
    2. before listening
    3. before responding
    4. before speaking
  4. Which of the following forms is assumed by the language style of a person ?
    1. a reaction
    2. a habit
    3. a response
    4. a thought
  5. The process of language is a wonder because it is
    1. a reaction
    2. a response
    3. smooth
    4. involuntary

June 2019 – Main Paper (Management)

A business will usually go through a clear set of stages that will make up its organisational life cycle. These stages include introduction, early growth, continuous growth, maturity and decline. The first stage, the introduction stage, it the start-up phase where a business decides what its core strengths and capabilities are and starts selling its product or service. At this early stage, the founder or founders will be a part of every aspect of the daily process of the business. The main goal at this stage is to take off to a good start and make a place in the market.

The next stage, the early growth phase, aims at increased sales and more development. The focus at this stage remains on the original product or service but the effort is to increase the market share and venture into related products or services. The main goal is to move the founder to a more managerial role so more time is spent on managing and building the business. At this stage, documents and policies need to be developed so any member of the organisation can see the business any time.

The third stage, the continuous growth stage, requires a systematic structure and more formal relationships among its participants. At this stage, the resource requirements of the business need careful handling. The focus is on the expansion of the business, keeping in mind its core strength and capability. A formal organisational structure and a clear delegation plan are important at this stage.

At the fourth stage of maturity, a business often slows down as the level of innovative energy may have become weak and the formal structures may have become obstacles. A lot of care is required to prevent decline.

  1. When will a business create its place in the market?
    1. At maturity
    2. At introduction
    3. At early growth
    4. At late growth
  2. Venturing into related products and services does not mean that a business will
    1. ignore the original product
    2. move the founder to a more managerial role
    3. delegate more responsibilities
    4. ignore the related products and services
  3. The founder of a business does not move to a more managerial role at which stage ?
    1. Early growth stage
    2. Maturity stage
    3. Decline stage
    4. Start up stage
  4. The resource requirements of a business need careful handling at which stage ?
    1. Fourth
    2. First
    3. Third
    4. Second
  5. The slowing down of a business can happen if,
    1. there is a decline in the market share
    2. the level of innovative energy weakens
    3. the managerial roles are not clear
    4. the level of investment declines

December 2018 – Main Paper (Commerce)

Read the following passage carefully and answer question numbers from 11 to 15 :

It is easy to see that there is nothing particularly unusual, or especially contrary to reason, for a person to choose to pursue a goal that is not exclusively confined to his or her own self-interest. As Adam Smith noted, we do have many different motivations, taking us well beyond the single-minded pursuit of our interest. There is nothing contrary to reason in our willingness to do things that are not entirely self-serving. Some of these motivations, like ‘humanity, justice, generosity and public spirit’, may even be very productive for society, as Smith noted. There tends to be, however, more resistance to accepting the possibility that people may have good reasons even to go beyond the pursuit of their own goals. The argument runs : if you are consciously not pursuing what you think are your goals, then clearly those cannot be your goals. Indeed many authors have taken the view that the claim that one can have reason not to be confined to the pursuit of one’s goals is ‘nonsensical’ since even strongly heterogeneous or altruistic agents cannot pursue other people’s goals without making their own. The point here is that in denying that rationality demands that you must act single-mindedly according to your own goals. You do not necessarily dedicate yourself to the promotion of others. We can reason our way towards following decent rules of behavior that we see being fair to others as well. This can restrain the unique dominance of single-minded pursuit of our own goals. There is nothing particularly mysterious about our respect for sensible rules of conduct. This can qualify the pursuit of what we rightly and reasonably-see as goals that we would in general like to advance. What we can say about your choice ? There is no difficulty in understanding that you are not averse to helping your neighbor-or anyone else-pursue his or her well-being. But it so happens that you do not think that your neighbour’s well-being is, in fact, best advanced by his wasting time on playing a silly game. Your action is not corollary of any general pursuit of well-being.

  1. The observation of Adam Smith in going beyond self-serving interest is :
    1. Motivations like justice and public spirit
    2. Issues contrary to reasons.
    3. Having one’s own goals
    4. Willingness to do different things
  2. What can stand in the way of single-minded pursuit of one’s goals?
    1. Giving priority to our own goals
    2. Pursuit of paradoxical parameters
    3. Compulsion to consider the goals of others
    4. Respect for sensible rules of conduct
  3. The moral derived from the passage is
    1. Not to think of our neighbour
    2. Force people to take on the other people’s goals as their own
    3. Assist your neighbours to engage in any activity of their choice
    4. Pro-active move to support others
  4. In the view of many authors, not pursuing one’s own goal is :
    1. Rational
    2. Altruistic
    3. Natural
    4. Nonsensical
  5. According to passage, decent rules of behaviour are needed:
    1. Due to dominance of our own goals
    2. For being fair to others
    3. Because for self-imposed restraints
    4. To be rational

December 2018 – Main Paper (Management)

Read the following passage carefully and answer questions 11 to 15.

For all the disagreement in the industry about the future of aviation, there is perfect accord on one point: There is going to be a lot more of it. The world’s air passengers flew a combined 7.64 trillion kilometers in 2017. By 2037, that will rise to 18.97 trillion kilometers, with about 40 % of the increase happening within five intra-regional markets: China, India, North-America, Europe and South-east Asia. That is sparking a battle over the biggest bottleneck holding back this growth: apirports. The governments that still own many of them should be more open to privatisation to cover a $ 78 billion funding gap in needed capital investments. Airlines, airports’ biggest customers, see things differently. Costs at privatised terminals are higher and governments should be cautious about such actions in the interests of expanding the aviation sector as a whole. Privatising an airport does not necessarily make it more efficient. A study has found there was little difference between the performance of airports 100% owned by commercially-oriented government corporations and those majority controlled by private businesses. The key is instead to avoid structures where the incentives for managers are confused or misaligned, such as where private companies are brought in as minority investors or where managers are essentially bureaucrats swayed by political imperatives. There is a better solution out there, but it is not likely to be very attractive to incumbent airlines, airports, or passengers enamoured of the current generation of gleaming terminals: build more, cheaper airports.

Which of the following is the focal inference of the author in the passage ?

1. Government-owned airports are known for high performance.
2. Growth of the aviation industry is distorted
3. Airports are not an obstacle to the growth of aviation industry.
4. Private ownership of airports does not mean efficient management.

Why should the governments be open to privatisation of airports?

1. For necessary capital
2. Because they own many airports.
3. Because there is a battle for growth
4. To limit the government control of aviation industry.

What is the key for making airports function efficiently ?
1. Provide misaligned incentives to managers.
2. Make private companies as minority shareholders.
3. Avoid structures of confused incentives to managers.
4. Recognise political imperatives of bureaucrats appointed as managers.

How do airlines see the move for privatisation of airports ?

(a) Feel that costs are high
(b) Governments should be cautions
(c) Interest of the aviation sector is important
(d) It decentralises the authority to control

Code :
1. (a), (c) and (d)
2. (a), (b) and (c)
3. (d), (a) and (b)
4. (c), (d) and (a)

What is the growth rate in aviation apart from the five intra-regional markets ?

1. 20 percent 2. 30 percent 3. 40 percent 4. 10 percent

July 2018

Read the following passage carefully and answer questions from 11 to 15 :

If India has to develop her internal strengths, the nation has to focus on the technological imperatives, keeping in mind three dynamic dimensions : the people, the overall economy and the strategic interests. These technological imperatives also take into account a ‘fourth’ dimension, time, an offshoot of modern day dynamism in business, trade, and technology that leads to continually shifting targets. We believe that technological strengths are especially crucial in dealing with this fourth dimension underlying continuous change in the aspirations of the people, the economy in the global context, and the strategic interests. The progress of technology lies at the heart of human history. Technological strengths are the key to creating more productive employment in an increasingly competitive market place and to continually upgrade human skills. Without a pervasive use of technologies, we cannot achieve overall development of our people in the years to come. The direct linkages of technology to the nation’s strategic strengths are becoming more and more clear, especially since 1990s. India’s own strength in a number of core areas still puts it in a position of reasonable strength in geopolitical context. Any nation aspiring to become a developed one needs to have strengths in various strategic technologies and also the ability to continually upgrade them through its own creative strengths. For people-oriented actions as well, whether for the creation of large scale productive employment or for ensuring nutritional and health security for people, or for better living conditions, technology is the only vital input. The absence of greater technological impetus could lead to lower productivity and wastage of precious natural resources. Activities with low productivity or low value addition, in the final analysis hurt the poorest most. The technological imperatives to lift our people to a new life, and to a life they are entitled to is important. India, aspiring to become a major economic power in terms of trade and increase in GDP, cannot succeed on the strength of turnkey projects designed and built abroad or only through large-scale imports of plant machinery, equipment and know how. Even while being alive to the short-term realities, medium and long-term strategies to develop core technological strengths within our industry are vital for envisioning a developed India.

  1. According to the above passage, which of the following are indicative of the fourth dimension ?
    (a) Aspirations of people
    (b) Modern day dynamism
    (c) Economy in the global context
    (d) Strategic interests
    Code :
    (1) (a), (b) and (c) only
    (2) (b), (c) and (d) only
    (3) (a), (c) and (d) only
    (4) (a), (b) and (d) only
  2. More productive employment demands :
    (1) Pervasive use of technology
    (2) Limiting competitive market place
    (3) Geo-political considerations
    (4) Large industries
  3. Absence of technology would lead to :
    (a) Less pollution
    (b) Wastage of precious natural resources
    (c) Low value addition
    (d) Hurting the poorest most
    Code :
    (1) (a), (b) and (c) only
    (2) (b), (c) and (d) only
    (3) (a), (b) and (d) only
    (4) (a), (c) and (d) only
  4. The advantage of technological inputs would result in :
    (1) Unbridled technological growth
    (2) Importing plant machinery
    (3) Sidelining environmental issues
    (4) Lifting our people to a life of dignity
  5. Envisioning a developed India requires :
    (1) Aspiration to become a major economic player
    (2) Dependence upon projects designed abroad
    (3) Focus on short-term projects
    (4) Development of core technological strengths

November 2017

Read the passage carefully and answer question numbers from 11 to 15.

Climate change is considered to be one of the most serious threats to sustainable development, with adverse impacts on the environment, human health, food security, economic activity, natural resources and physical infrastructure. Global climate varies naturally. According to the Inter-governmental Panel on Climate Change (IPCC), the effects of climate change have already been observed, and scientific findings indicate that precautionary and prompt action is necessary. Vulnerability to climate change is not just a function of geography or dependence on natural resources; it also has social, economic and political dimensions which influence how climate change affects different groups. Poor people rarely have insurance to cover loss of property due to natural calamities i.e. drought, floods, super cyclones etc. The poor communities are already struggling to cope with the existing challenges of poverty and climate variability and climate change could push many beyond their ability to cope or even survive. It is vital that these communities are helped to adapt to the changing dynamics of nature. Adaptation is a process through which societies make themselves better able to cope with an uncertain future. Adapting to climate change entails taking the right measures to reduce the negative effects of climate change (or exploit the positive ones) by making the appropriate adjustments and changes. These range from technological options such as increased sea defences or flood proof houses on stilts to behavioural change at the individual level, such as reducing water use in times of drought. Other strategies include early warning systems for extreme events, better water management, improved risk management, various insurance options and biodiversity conservation. Because of the speed at which climate change is happening due to global temperature rise, it is urgent that the vulnerability of developing countries to climate change is reduced and their capacity to adapt is increased and national adaptation plans are implemented. Adapting to climate change will entail adjustments and changes at every level from community to national and international. Communities must build their resilience, including adopting appropriate technologies while making the most of traditional knowledge, and diversifying their livelihoods to cope with current and future climate stress. Local coping strategies and knowledge need to be used in synergy with government and local interventions. The need of adaptation interventions depends on national circumstances. There is a large body of knowledge and experience within local communities on coping with climatic variability and extreme weather events. Local communities have always aimed to adapt to variations in their climate. To do so, they have made preparations based on their resources and their knowledge accumulated through experience of past weather patterns. This includes times when they have also been forced to react to and recover from extreme events, such as floods, drought and hurricanes. Local coping strategies are an important element of planning for adaptation. Climate change is leading communities to experience climatic extremes more frequently, as well as new climate conditions and extremes. Traditional knowledge can help to provide efficient, appropriate and time – tested ways of advising and enabling adaptation to climate change in communities who are feeling the effects of climate changes due to global warming.

Given below are the factors of vulnerability of poor people to climate change. Select the code that contains the correct answer.
(a) Their dependence on natural resources (b) Geographical attributes
(c) Lack of financial resources (d) Lack of traditional knowledge
Code :
(A) (a), (b) and (c) (B) (b), (c) and (d) (C) (a), (b), (c) and (d) (D) (c) only
Adaptation as a process enables societies to cope with :
(a) An uncertain future (b) Adjustments and changes
(c) Negative impact of climate change (d) Positive impact of climate change
Select the most appropriate answer from the following code :
(A) (a),(b) (c) and (d) (B) (a) and (c) (C) (b), (c) and (d) (D) (c) only
To address the challenge of climate change, developing countries urgently require :
(A) Imposition of climate change tax
(B) Implementation of national adaptation policy at their level
(C) Adoption of short-term plans
(D) Adoption of technological solutions
The traditional knowledge should be used through :
(A) Its dissemination
(B) Improvement in national circumstances
(C) Synergy between government and local interventions
(D) Modern technology
The main focus of the passage is on :
(A) Combining traditional knowledge with appropriate technology
(B) Co-ordination between regional and national efforts
(C) Adaptation to climate change
(D) Social dimensions of climate change

January 2017

Read the following passage carefully and answer questions from 7 to 12 :
The last great war, which nearly shook the foundations of the modern world, had little impact on Indian literature beyond aggravating the popular revulsion against violence and adding to the growing disillusionment with the ‘humane pretensions’ of the Western World. This was eloquently voiced in Tagore’s later poems and his last testament, Crisis in Civilisation. The Indian intelligentsia was in a state of moral dilemma. On the one hand, it could not help sympathising with England’s dogged courage in the hour of peril, with the Russians fighting with their backs to the wall against the ruthless Nazi hordes, and with China groaning under the heel of Japanese militarism; on the other hand, their own country was practically under military occupation of their own soil, and an Indian army under Subhas Bose was trying from the opposite camp to liberate their country. No creative impulse could issue from such confusion of loyalties. One would imagine that the achievement of Indian independence in 1947, which came in the wake of the Allies’ victory and was followed by the collapse of colonialism in the neighbouring countries of South-East Asia, would have released an upsurge of creative energy. No doubt it did, but unfortunately it was soon submerged in the great agony of partition, with its inhuman slaughter of the innocents and the uprooting of millions of people from their homeland, followed by the martyrdom of Mahatma Gandhi. These tragedies, along with Pakistan’s invasion of Kashmir and its later atrocities in Bangladesh, did indeed provoke a poignant writing, particularly in the languages of the regions most affected, Bengali, Hindi, Kashmiri, Punjabi, Sindhi and Urdu. But poignant or passionate writing does not by itself make great literature. What reserves of enthusiasm and confidence survived these disasters have been mainly absorbed in the task of national reconstruction and economic development. Great literature has always emerged out of chains of convulsions. Indian literature is richer today in volume, range and variety than it ever was in the past.

What was the impact of the last great war on Indian literature?
(A) It had no impact. (B) It aggravated popular revulsion against violence.
(C) It shook the foundations of literature. (D) It offered eloquent support to the Western World.
What did Tagore articulate in his last testament ?
(A) Offered support to Subhash Bose.
(B) Exposed the humane pretensions of the Western World.
(C) Expressed loyalty to England.
(D) Encouraged the liberation of countries.
What was the stance of Indian intelligentsia during the period of great war ?
(A) Indifference to Russia’s plight.
(B) They favoured Japanese militarism.
(C) They prompted creativity out of confused loyalties.
(D) They expressed sympathy for England’s dogged courage.
Identify the factor responsible for the submergence of creative energy in Indian literature.
(A) Military occupation of one’s own soil. (B) Resistance to colonial occupation.
(C) Great agony of partition. (D) Victory of Allies.
What was the aftermath that survived tragedies in Kashmir and Bangladesh ?
(A) Suspicion of other countries (B) Continuance of rivalry
(C) Menace of war (D) National reconstruction
The passage has the message that
(A) Disasters are inevitable.
(B) Great literature emerges out of chains of convulsions.
(C) Indian literature does not have a marked landscape.
(D) Literature has no relation with war and independence.

July 2016

Read the following passage carefully and answer questions from 5 to 10 :
Many aspects of the motion-picture industry and its constituent companies are dissimilar to those observable in advanced-technology industries and firms. For instance, company longevity does not represent a consistent concern across the two organisational contexts. In the advanced-technology company for example, one new-product innovation – which is expected to generate financial returns to the firm – is insufficient for the company to be successful.Rather, a stream of new product innovations is required. By contrast with the independent production company of this case, each new film – which is expected to generate financial returns to the principals – is sufficient for the company to be successful. Any subsequent new films involving the firm’s participants will be produced by a different independent company.

As another instance, people’s learning is expected to have different contributors and beneficiaries across the two organizational contexts. In the advanced-technology company, for example, each new product innovation provides an opportunity for participants on the project team to learn and acquire experience, and this same company intends to retain such participants, hence, benefit from their increased experience on the next project. By contrast with the independent production company, each new film provides an opportunity for participants on the project team to learn and acquire this experience also, but this same company has little or no expectation of retaining such participants, and hence, benefitting from their increased experience in the next project.

Experience is paramount in the motion-picture industry. Generally, on film projects, budgets are very tight, and schedules are very demanding. People are hired largely based on their experience and are expected to perform well immediately when called to do so. There is negligible slack time or margin for learning through trial and error, but experienced people learn exactly through trial and error. Because experience is valued so highly and film-production houses have such short time, entry into the industry is very difficult for most people. Further, the role played by schools and colleges is minimal in this industry. Some skills and techniques can be learned and refined through formal education (e.g., acting schools, theatre, film degrees), but the majority come through direct experience. Mentoring plays an important role. True, the film business focuses heavily on exploitation over exploration. Yet success of the industry as a whole is critically dependent upon learning and exploration overtime.

Answer the following questions :

What is not a consistent concern across the two organisational contexts ?
(1) Dissimilarity (2) Product package
(3) Financial return (4) Company longevity

What will be sufficient for an independent production company to be successful ?
(1) New product innovations (2) Financial returns from each new film
(3) Active role by firm’s participants (4) Organisational context

What does an advanced-technology company expect from the learning experience of its
participants ?
(1) Benefit for the next project (2) Opportunity for more learning
(3) Little expectation of retaining them(4) Help in marketing the previous product

What is not the expectation of an independent production company in the case of its
participants ?
(1) Absence from the next project. (2) Retention for the next project.
(3) Participation in the current project. (4) Use of opportunity to acquire experience.

Why do film production houses value experience highly ?
(1) Because of the importance of trial and error methods.
(2) Because of the margin for learning.
(3) Because of short time horizons.
(4) Because it allows easy entry to everyone into the film world.

According to the author, what has been the focus of film business ?
(1) Formal education (2) Mentoring
(3) Exploitation (4) Indirect experience

December 2015

Read the following passage carefully and answer question numbers 13 to 17.

I did that thing recently where you have to sign a big card which is a horror unto itself, especially as the keeper of the Big Card was leaning over me at the time. Suddenly I was on the spot, a rabbit in the headlights, torn between doing a fun message or some sort of in-joke or a drawing. Instead overwhelmed by the myriad options available to me, I decided to just write : “Good luck, best, Joel”.

It was then that I realised, to my horror, that I had forgotten how to write. My entire existence is “tap letters into computer”. My shopping lists are hidden in the notes function of my phone. If I need to remember something I send an e-mail to myself. A pen is something I chew when I’m struggling to think. Paper is something I pile beneath my laptop to make it a more comfortable height for me to type on.

A poll of 1,000 teens by the stationers, Bic found that one in 10 don’t own a pen, a third have never written a letter, and half of 13 to 19 years – old have never been forced to sit down and write a thank you letter. More than 80% have never written a love letter, 56% don’t have letter paper at home. And a quarter have never known the unique torture of writing a birthday card. The most a teen ever has to use a pen is on an exam paper.
Bic, have you heard of mobile phones ? Have you heard of e-mail, facebook and snap chatting ? This is the future. Pens are dead. Paper is dead. Handwriting is a relic.
“Handwriting is one of the most creative outlets we have and should be given the same importance as other art forms such as sketching, painting or photography.”

Answer the following questions :

When confronted with signing a big card, the author felt like “a rabbit in the headlight”. What does this phrase mean?
(A) A state of confusion (B) A state of pleasure
(C) A state of anxiety (D) A state of pain

According to the author, which one is not the most creative outlet of pursuit ?
(A) Handwriting (B) Photography (C) Sketching (D) Reading

The entire existence of the author revolves round :
(a) Computer (b) Mobile phone (c) Typewriter

Identify the correct answer from the codes given below :

(A) (b) only (B) (a) and (b) only (C) (a), (b) and (c) (D) (b) and (c) only

How many teens, as per the Bic survey, do not own a pen?
(1) 800 (2) 560 (3) 500 (4) 100

What is the main concern of the author?

(1) That the teens use social networks for communication.
(2) That the teens use mobile phones.
(3) That the teens use computer.
(4) That the teens have forgotten the art of handwriting

June 2015

Read the following passage carefully and answer questions 13 to 18.

Story telling is not in our genes. Neither it is an evolutionary history. It is the essence of what makes us Human.
Human beings progress by telling stories. One event can result in a great variety of stories being told about it. Sometimes those stories differ greatly. Which stories are picked up and repeated and which ones are dropped and forgotten often determines how we progress. Our history, knowledge and understanding are all the collections of the few stories that survive. This includes the stories that we tell each other about the future. And how the future will turn out depends partly, possibly largely, on which stories we collectively choose to believe.

Some stories are designed to spread fear and concern. This is because some story-tellers feel that there is a need to raise some tensions. Some stories are frightening, they are like totemic warnings : “Fail to act now and we are all doomed.” Then there are stories that indicate that all will be fine so long as we leave everything upto a few especially able adults. Currently, this trend is being led by those who call themselves “rational optimists”. They tend to claim that it is human nature to compete and to succeed and also to profit at the expense of others. The rational optimists however, do not realize how humanity has progressed overtime through amiable social networks and how large groups work in less selfishness and in the process accommodate rich and poor, high and low alike. This aspect in story-telling is considered by the ‘Practical Possibles’, who sit between those who say all is fine and cheerful and be individualistic in your approach to a successful future, and those who ordain pessimism and fear that we are doomed.
What the future holds for us is which stories we hold on to and how we act on them. Answer the following questions:

Our knowledge is a collection of :
(A) all stories that we have heard during our life-time (B) some stories that we remember
(C) a few stories that survive (D) some important stories
Story telling is :
(A) an art (B) a science
(C) in our genes (D) the essence of what makes us human
How the future will turn out to be, depends upon the stories ?
(A) We collectively choose to believe in (B) Which are repeatedly narrated
(C) Designed to spread fear and tension (D) Designed to make prophecy
Rational optimists :
(a) Look for opportunities.
(b) Are sensible and cheerful.
(c) Are selfishly driven.
Identify the correct answer from the codes given below :
(A) (a), (b) and (c) (B) (a) only (C) (a) and (b) only (D) (b) and (c) only
Humans become less selfish when :
(A) they work in large groups (B) they listen to frightening stories
(C) they listen to cheerful stories (D) they work in solitude
‘Practical Possibles’ are the ones who :
(A) follow Midway Path (B) are doom-mongers
(C) are self-centre (D) are cheerful and carefree

December 2014

Read the following passage carefully and answer questions 13 to 17 :

The literary distaste for politics, however, seems to be focused not so much on the largely murky practice of politics in itself as a subject of literary representation but rather more on how it is often depicted in literature, i.e., on the very politics of such representation. A political novel often turns out to be not merely a novel about politics but a novel with a politics of its own, for it seeks not merely to show us how things are but has fairly definite ideas about how things should be, and precisely what one should think and do in order to make things move in that desired direction. In short, it seeks to convert and enlist the reader to a particular cause or ideology; it often is (in an only too familiar phrase) not literature but propaganda. This is said to violate the very spirit of literature which is to broaden our understanding of the world and the range of our sympathies rather than to narrow them down through partisan commitment. As John Keats said, ‘We hate poetry that has a palpable design upon us’.

Another reason why politics does not seem amenable to the highest kind of literary representation seems to arise from the fact that politics by its very nature is constituted of ideas and ideologies. If political situations do not lend themselves to happy literary treatment, political ideas present perhaps an even greater problem in this regard. Literature, it is argued, is about human experiences rather than about intellectual abstractions; it deals in what is called the ‘felt reality’ of human flesh and blood, and in sap and savour (rasa) rather than in arid and lifeless ideas. In an extensive discussion of the matter in her book Ideas and the Novel, the American novelist Mary McCarthy observed that ‘ideas are still today felt to be unsightly in the novel’ though that was not so in ‘former days’, i.e., in the 18th and 19th centuries. Her formulation of the precise nature of the incompatibility between ideas on the one hand and the novel on the other betrays perhaps a divided conscience in the matter and a sense of dilemma shared by many writers and readers : ‘An idea cannot have loose ends, but a novel, I almost think, needs them. Nevertheless, there is enough in common for the novelists to feel… the attraction of ideas while taking up arms against them – most often with weapons of mockery.’

According to the passage, a political novel often turns out to be a
(A) Literary distaste for politics (B) Literary representation of politics
(C) Novel with its own politics (D) Depiction of murky practice of politics
A political novel reveals
(A) Reality of the things (B) Writer’s perception
(C) Particular ideology of the readers (D) The spirit of literature
The constructs of politics by its nature is
(A) Prevalent political situation (B) Ideas and Ideologies
(C) Political propaganda (D) Understanding of human nature
Literature deals with
(A) Human experiences in politics (B) Intellectual abstractions
(C) Dry and empty ideas (D) Felt reality of human life
The observation of the novelist, Mary McCarthy reveals
(A) unseen felt ideas of today in the novel
(B) dichotomy of conscience on political ideas and novels
(C) compatibility between idea and novel
(D) endless ideas and novels

June 2014

Traditional Indian Values must be viewed both from the angle of the individual and from that of the geographically delimited agglomeration of peoples or groups enjoying a common system of leadership which we call the ‘State’. The Indian ‘State’s’ special feature is the peaceful, or perhaps mostly peaceful, co-existence of social groups of various historical provenances which mutually adhere in a geographical, economic, and political sense, without ever assimilating to each other in social terms, in ways of thinking, or even in language. Modern Indian law will determine certain rules, especially in relation to the regime of the family, upon the basis of how the loin-cloth is tied, or how the turban is worn, for this may identify the litigants as members of a regional group, and therefore as participants in its traditional law, though their ancestors left the region three or four centuries earlier. The use of the word ‘State’ above must not mislead us. There was no such thing as a conflict between the individual and the State, atleast before foreign governments became established, just as there was no concept of state ‘sovereignty’ or of any church-and-state dichotomy.

Modern Indian ‘secularism’ has an admittedly peculiar feature : It requires the state to make a fair distribution of attention and support amongst all religions. These blessed aspects of India’s famed tolerance (Indian kings so rarely persecuted religious groups that the exceptions prove the rule) at once struck Portuguese and other European visitors to the West Coast of India in the sixteenth century, and the impression made upon them in this and other ways gave rise, at one remove, to the basic constitution of Thomas More’s Utopia. There is little about modern India that strikes one at once as Utopian : but the insistence upon the inculcation of norms, and the absence of bigotry and institutionalized exploitation of human or natural resources, are two very different features which link the realities of India and her tradition with the essence of all Utopians.

Which of the following is a special feature of the Indian State ?
(A) Peaceful co-existence of people under a common system of leadership
(B) Peaceful co-existence of social groups of different historical provenances attached to each other in a geographical, economic and political sense
(C) Social integration of all groups
(D) Cultural assimilation of all social groups.
The author uses the word ‘State’ to highlight
(A) Antagonistic relationship between the state and the individual throughout the period of history.
(B) Absence of conflict between the state and the individuals upto a point in time.
(C) The concept of state sovereignty.
(D) Dependence on religion.
Which one is the peculiar feature of modern Indian ‘Secularism’ ?
(A) No discrimination on religious considerations (B) Total indifference to religion
(C) No space for social identity (D) Disregard for social law
The basic construction of Thomas More’s Utopia was inspired by
(A) Indian tradition of religious tolerance.
(B) Persecution of religious groups by Indian rulers.
(C) Social inequality in India.
(D) European perception of Indian State.
What is the striking feature of modern India ?
(A) A replica of Utopian State (B) Uniform laws
(C) Adherence to traditional values (D) Absence of Bigotry

December 2013

Heritage conservation practices improved worldwide after the International Centre for the Study of the Preservation and Restoration of Cultural Property (ICCROM) was established with UNESCO’s assistance in 1959. The inter-governmental organisation with 126 member states has done a commendable job by training more than 4,000 professionals, providing practice standards, and sharing technical expertise. In this golden jubilee year, as we acknowledge its key role in global conservation, an assessment of international practices would be meaningful to the Indian conservation movement. Consistent investment, rigorous attention, and dedicated research and dissemination are some of the positive lessons to imbibe. Countries such as Italy have demonstrated that prioritising heritage with significant budget provision pays. On the other hand, India, which is no less endowed in terms of cultural capital, has a long way to go. Surveys indicate that in addition to the 6,600 protected monuments, there are over 60,000 equally valuable heritage structures that await attention. Besides the small group in the service of Archaeological Survey of India, there are only about 150 trained conservation professionals. In order to overcome this severe shortage the emphasis has been on setting up dedicated labs and training institutions. It would make much better sense for conservation to be made part of mainstream research and engineering institutes, as has been done in Europe.

Increasing funding and building institutions are the relatively easy part. The real challenge is to redefine international approaches to address local contexts. Conservation cannot limit itself to enhancing the art-historical value of the heritage structures, which international charters perhaps over emphasise. The effort has to be broad-based : It must also serve as a means to improving the quality of life in the area where the heritage structures are located. The first task therefore is to integrate conservation efforts with sound development plans that take care of people living in the heritage vicinity. Unlike in western countries, many traditional building crafts survive in India, and conservation practices offer an avenue to support them. This has been acknowledged by the Indian National Trust for Art and Cultural Heritage charter for conservation but is yet to receive substantial state support. More strength for heritage conservation can be mobilised by aligning it with the green building movement. Heritage structures are essentially eco-friendly and conservation could become a vital part of the sustainable building practices campaign in future.

The outlook for conservation heritage changed
(A) after the establishment of the International Centre for the Study of the Preservation and Restoration of Cultural Property.
(B) after training the specialists in the field.
(C) after extending UNESCO’s assistance to the educational institutions.
(D) after ASI’s measures to protect the monuments.
The inter-government organization was appreciated because of
(A) increasing number of members to 126.
(B) imparting training to professionals and sharing technical expertise.
(C) consistent investment in conservation.
(D) its proactive role in renovation and restoration.
Indian conservation movement will be successful if there would be
(A) Financial support from the Government of India.
(B) Non-governmental organisations role and participation in the conservation movement.
(C) consistent investment, rigorous attention, and dedicated research and dissemination of awareness for conservation.
(D) Archaeological Survey of India’s meaningful assistance.
As per the surveys of historical monuments in India, there is very small number of protected monuments. As per given the total number of monuments and enlisted number of protected monuments, percentage comes to
(A) 10 percent (B)11 percent (C) 12 percent (D) 13 percent
What should India learn from Europe to conserve our cultural heritage ?
(i) There should be significant budget provision to conserve our cultural heritage.
(ii) Establish dedicated labs and training institutions. (iii) Force the government to provide sufficient funds.
(iv) Conservation should be made part of mainstream research and engineering institutes.

Choose correct answer from the codes given below :
(A) (i), (ii), (iii), (iv) (B) (i), (ii). (iv) (C) (i), (ii) (D) (i), (iii), (iv)

INTACH is known for its contribution for conservation of our cultural heritage. The full form of INTACH is
(A) International Trust for Art and Cultural Heritage.
(B) Intra-national Trust for Art and Cultural Heritage
(C) Integrated Trust for Art and Cultural Heritage
(D) Indian National Trust for Art and Cultural Heritage

September 2013

I had occasion to work with her closely during the Women’s International Year in 1975 when she was chairing a Steering Committee and made me the member in charge of publicity.

Representatives from different political parties and women’s organizations were on the committee and though the leftists claimed a sort of proprietary right over her, Aruna encouraged and treated all members alike. It was not her political affiliations or her involvement in a particular cause, which won her respect and recognition, but her utter honesty in public life, her integrity and her compassion for the oppressed which made her an adorable person. She had the courage to differ with and defy the mightiest in the land; yet her human spirit prompted her to work in the worst of slums to offer succour to the poor and the exploited.

In later years – around late eighties and early nineties – Aruna Asaf Ali’s health began to deteriorate. Though her mind remained alert, she could not actively take up her pet causes – action for women’s advancement, planning for economic justice, role of media, reaffirmation of values in public affairs etc. Slowly, her movements were restricted and Aruna who had drawn sustenance from common people, from her involvement in public life, became a lonely person. She passed away in July 1996.

Which Committee was chaired by Aruna ?
(A) Women’s International Year’s Committee
(B) Steering Committee of Women’s International Year
(C) A Committee of Publicity
(D) Women’s Organizations
Who were made the members of the Committee of Publicity? Choose the answer from codes given below :
(i) Representatives from different political parties.
(ii) Representatives from the leftist parties.
(iii) Representatives from the women’s organizations.
(iv) None of the above.

Codes :

(A) (i), (iii) (B) (i), (ii) (C) (i), (ii), (iii) (D) (iv)

Aruna earned respect because of

(A) she identified with the leftists (B) she did not associate with any political party
(C) chairing a Steering Committee (D) she identified with women’s organizations

Who tried to monopolize Aruna as their proprietary right ?

(A) Women Organizations (B) Leftists (C) Steering Committee (D) Some Political Parties

Aruna’s health began to deteriorate from

(A) 1985 – 2002 (B) 1998 – 2000 (C) 1981 – 2000 (D) 1989 – 2001

Aruna’s pet cause(s) in her life was/ were

(A) Role of media (B) Economic justice
(C) Reaffirmation of values in public affairs (D) All the above

June 2013

The Taj Mahal has become one of the world’s best known monuments. This domed white marble structure is situated on a high plinth at the southern end of a four-quartered garden, evoking the gardens of paradise, enclosed within walls measuring 305 by 549 metres. Outside the walls, in an area known as Mumtazabad, were living quarters for attendants, markets, serais and other structures built by local merchants and nobles. The tomb complex and the other imperial structures of Mumtazabad were maintained by the income of thirty villages given specifically for the tomb’s support. The name Taj Mahal is unknown in Mughal chronicles, but it is used by contemporary Europeans in India, suggesting that this was the tomb’s popular name. In contemporary texts, it is generally called simply the Illuminated Tomb (Rauza-i- Munavvara).

Mumtaz Mahal died shortly after delivering her fourteenth child in 1631. The Mughal court was then residing in Burhanpur. Her remains were temporarily buried by the grief- stricken emperor in a spacious garden known as Zainabad on the bank of the river Tapti. Six months later her body was transported to Agra, where it was interred in land chosen for the mausoleum. This land, situated south of the Mughal city on the bank of the Jamuna, had belonged to the Kachhwaha rajas since the time of Raja Man Singh and was purchased from the then current raja, Jai Singh. Although contemporary chronicles indicate Jai Singh’s willing cooperation in this exchange, extant _farmans (imperial commands) indicate that the final price was not settled until almost two years after the mausoleum’s commencement. Jai Singh’s further cooperation was insured by imperial orders issued between 1632 and 1637 demanding that he provide stone masons and carts to transport marble from the mines at Makrana, within his “ancestral domain”, to Agra where both the Taj Mahal and Shah Jahan’s additions to the Agra fort were constructed concurrently.

Work on the mausoleum was commenced early in 1632. Inscriptional evidence indicates much of the tomb was completed by 1636. By 1643, when Shah Jahan most lavishly celebrated the ‘Urs ceremony for Mumtaz Mahal’, the entire complex was virtually complete.

Marble stone used for the construction of the Taj Mahal was brought from the ancestral domain of Raja Jai Singh. The name of the place where mines of marble is

(A) Burhanpur (B) Makrana (C) Amber (D) Jaipur

The popular name Taj Mahal was given by

(A) Shah Jahan (B) Tourists (C) Public (D) European travellers

Point out the true statement from the following :

(A) Marble was not used for the construction of the Taj Mahal.
(B) Red sand stone is non-visible in the Taj Mahal complex.
(C) The Taj Mahal is surrounded by a four-quartered garden known as Chahr Bagh.
(D) The Taj Mahal was constructed to celebrate the ‘Urs ceremony for Mumtaz Mahal’.

In the contemporary texts the Taj Mahal is known

(A) Mumtazabad (B) Mumtaz Mahal (C) Zainabad (D) Rauza-i-Munavvara

The construction of the Taj Mahal was completed between the period

(A) 1632 – 1636A.D. (B) 1630 – 1643A.D.
(C) 1632 – 1643A.D. (D) 1636 – 1643A.D.

The documents indicating the ownership of land, where the Taj Mahal was built, known as

(A) Farman (B) Sale Deed (C) Sale-Purchase Deed (D) None of the above

December 2012

Read the following passage carefully and answer the questions (55 to 60) :
The popular view of towns and cities in developing countries and of urbanization process is that despite the benefits and comforts it brings, the emergence of such cities connotes environmental degradation, generation of slums and squatters, urban poverty, unemployment, crimes, lawlessness, traffic chaos etc. But what is the reality ? Given the unprecedential increase in urban population over the last 50 years from 300 million in 1950 to 2 billion in 2000 in developing countries, the wonder really is how well the world has coped, and not how badly.

In general, the urban quality of life has improved in terms of availability of water and sanitation, power, health and education, communication and transport. By way of illustration, a large number of urban residents have been provided with improved water in urban areas in Asia’s largest countries such as China, India, Indonesia and Philippines. Despite that, the access to improved water in terms of percentage of total urban population seems to have declined during the last decade of 20th century, though in absolute numbers, millions of additional urbanites, have been provided improved services. These countries have made significant progress in the provision of sanitation services too, together, providing for an additional population of more than 293 million citizens within a decade (1990-2000). These improvements must be viewed against the backdrop of rapidly increasing urban population, fiscal crunch and strained human resources and efficient and quality- oriented public management.

The popular view about the process of urbanization in developing countries is
(A) Positive (B) Negative (C) Neutral (D) Unspecified
The average annual increase in the number of urbanites in developing countries, from 1950 to 2000 A.D. was close to
(A) 30 million (B) 40 million (C) 50 million (D) 60 million
The reality of urbanization is reflected in
(A) How well the situation has been managed.
(B) How badly the situation has gone out of control.
(C) How fast has been the tempo of urbanization.
(D) How fast the environment has degraded.
Which one of the following is not considered as an indicator of urban quality of life ?
(A) Tempo of urbanization (B) Provision of basic services
(C) Access to social amenities (D) All the above
The author in this passage has tried to focus on
(A) Extension of Knowledge (B) Generation of Environmental Consciousness
(C) Analytical Reasoning (D) Descriptive Statement
In the above passage, the author intends to state
(A) The hazards of the urban life (B The sufferings of the urban life
(C) The awareness of human progress (D) The limits to growth

June 2012

James Madison said, “A people who mean to be their own governors must arm themselves with power that knowledge gives.” In India, the Official Secrets Act, 1923 was a convenient smokescreen to deny members of the public access to information. Public functioning has traditionally been shrouded in secrecy. But in a democracy in which people govern themselves, it is necessary to have more openness. In the maturing of our democracy, right to information is a major step forward; it enables citizens to participate fully in the decision-making process that affects their lives so profoundly. It is in this context that the address of the Prime Minister in the Lok Sabha is significant. He said, “I would only like to see that everyone, particularly our civil servants, should see the Bill in a positive spirit; not as a draconian law for paralyzing Government, but as an instrument for improving Government-Citizen interface resulting in a friendly, caring and effective Government functioning for the good of our People.” He further said, “This is an innovative Bill, where there will be scope to review its functioning as we gain experience. Therefore, this is a piece of legislation, whose working will be kept under constant reviews.”

The Commission, in its Report, has dealt with the application of the Right to Information in Executive, Legislature and Judiciary. The judiciary could be a pioneer in implementing the Act in letter and spirit because much of the work that the Judiciary does is open to public scrutiny, Government of India has sanctioned an e-governance project in the Judiciary for about ‘ 700 crores which would bring about systematic classification, standardization and categorization of records. This would help the judiciary to fulfil its mandate under the Act. Similar capacity building would be required in all other public authorities. The transformation from non-transparency to transparency and public accountability is the responsibility of all three organs of State.

A person gets power
(A) by acquiring knowledge (B) from the Official Secrets Act, 1923
(C) through openings (D) by denying public information
Right to Information is a major step forward to
(A) enable citizens to participate fully in the decision making process
(B) to make the people aware of the Act
(C) to gain knowledge of administration
(D) to make the people Government friendly
The Prime Minister considered the Bill
(A) to provide power to the civil servants
(B) as an instrument for improving Government-citizen interface resulting in a friendly, caring and effective Government
(C) a draconian law against the officials
(D) to check the harassment of the people
The Commission made the Bill effective by
(A) extending power to the executive authorities
(B) combining the executive and legislative power
(C) recognizing Judiciary a pioneer in implementing the act in letter and spirit
(D) educating the people before its implementation
The Prime Minister considered the Bill innovative and hoped that
(A) It could be reviewed based on the experience gained on its functioning.
(B) The civil servants would see the Bill in a positive spirit.
(C) It would not be considered as a draconian law for paralyzing Government
(D) All the above
The transparency and public accountability is the responsibility of three organs of the State. These three organs are
(A) Lok Sabha, Rajya Sabha and Judiciary
(b) Lok Sabha, Rajya Sabha and Executive
(C) Judiciary, Legislature and the Commission
(D) Legislature, Executive and Judiciary

December 2011

The catalytic fact of the twentieth century is uncontrollable development, consumerist society, political materialism, and spiritual devaluation. This inordinate development has led to the transcendental ‘second reality’ of sacred perception that biologically transcendence is a part of human life. As the century closes, it dawns with imperative vigour that the ‘first reality’ of enlightened rationalism and the ‘second reality’ of the Beyond have to be harmonised in a worthy state of man. The de facto values describe what we are, they portray the ‘is’ of our ethic, they are est values (Latin est means is). The ideal values tell us what we ought to be, they are esto values (Latin esto ‘ought to be’). Both have to be in the ebb and flow of consciousness. The ever new science and technology and the ever-perennial faith are two modes of one certainty, that is the wholeness of man, his courage to be, his share in Being.

The materialistic foundations of science have crumbled down. Science itself has proved that matter is energy, processes are as valid as facts, and affirmed the non-materiality of the universe. The encounter of the ‘two cultures’, the scientific and the humane, will restore the normal vision, and will be the bedrock of a ‘science of understanding’ in the new century. It will give new meaning to the ancient perception that quantity (measure) and quality (value) coexist at the root of nature. Human endavours cannot afford to be humanistically irresponsible.

The problem raised in the passage reflects overall on

(A) Consumerism (B) Materialism
(C) Spiritual devaluation (D) Inordinate development

The ‘de facto’ values in the passage means

(A) What is (B) What ought to be (C) What can be (D) Where it is

According to the passage, the ‘first reality’ constitutes

(A) Economic prosperity (B) Political development
(C) Sacred perception of life (D) Enlightened rationalism

Encounter of the ‘two cultures’, the scientific and the human implies

(A) Restoration of normal vision (B) Universe is both material and non-material
(C) Man is superior to nature (D) Co-existence of quantity and quality in nature

The contents of the passage are

(A) Descriptive (B) Prescriptive (C) Axiomatic (D) Optional

The passage indicates that science has proved that

(A) universe is material (B) matter is energy
(C) nature has abundance (D) humans are irresponsible

June 2011

All historians are interpreters of text if they be private letters, Government records or parish birth lists or whatever. For most kinds of historians, these are only the necessary means to understanding something other than the texts themselves, such as a political action or a historical trend, whereas for the intellectual historian, a full understanding of his chosen texts is itself the aim of his enquiries. Of course, the intellectual history is particularly prone to draw on the focus of other disciplines that are habitually interpreting texts for purposes of their own, probing the reasoning that ostensibly connects premises and conclusions. Furthermore, the boundaries with adjacent sub disciplines are shifting and indistinct : the history of art and the history of science both claim a certain autonomy, partly just because they require specialised technical skills, but both can also be seen as part of a wider intellectual history, as is evident when one considers, for example, the common stock of knowledge about cosmological beliefs or moral ideals of a period.

Like all historians, the intellectual historian is a consumer rather than a producer of ‘methods’. His distinctiveness lies in which aspect of the past he is trying to illuminate, not in having exclusive possession of either a corpus of evidence or a body of techniques. That being said, it does seem that the label ‘intellectual history’ attracts a disproportionate share of misunderstanding.

It is alleged that intellectual history is the history of something that never really mattered. The long dominance of the historical profession by political historians bred a kind of philistinism, an unspoken belief that power and its exercise was ‘what mattered’. The prejudice was reinforced by the assertion that political action was never really the outcome of principles or ideas that were ‘more flapdoodle’. The legacy of this precept is still discernible in the tendency to require ideas to have ‘licensed’ the political class before they can be deemed worthy of intellectual attention, as if there were some reasons why the history of art or science, of philosophy or literature, were somehow of interest and significance than the history of Parties or Parliaments. Perhaps in recent years the mirror-image of this philistinism has been more common in the claim that ideas of any one is of systematic expression or sophistication do not matter, as if they were only held by a minority.
Answer the following questions :

An intellectual historian aims to fully understand
(A) the chosen texts of his own (B) political actions
(C) historical trends (D) his enquiries

Intellectual historians do not claim exclusive possession of
(A) conclusions (B) any corpus of evidence
(C) distinctiveness (D) habitual interpretation
The misconceptions about intellectual history stem from
(A) a body of techniques (B) the common stock of knowledge
(C) the dominance of political historians (D) cosmological beliefs
What is philistinism ?
(A) Reinforcement of prejudice (B) Fabrication of reasons
(C) The hold of land-owning classes (D) Belief that power and its exercise matter
Knowledge of cosmological beliefs or moral ideas of a period can be drawn as part of
(A) literary criticism (B) history of science
(C) history of philosophy (D) intellectual history
The claim that ideas of any one is of systematic expression do not matter, as if they were held by a minority, is
(A) to have a licensed political class (B) a political action
(C) a philosophy of literature (D) the mirror-image of philistinism

For Visually Handicapped Candidates only
Read the following passage carefully and answer the questions from 47 to 52 :
All the perceptions of the human mind resolve themselves into two distinct kinds, which I shall call impressions and ideas. The difference betwixt these consists in the degrees of force and liveliness, with which they strike upon the mind, make their way into our thought or consciousness. Those perceptions which enter with most force and violence, we may name impressions; and under this name I comprehend all our sensations, passions and emotions, as they make their first appearance in the soul. By ideas I mean the faint images of these in thinking and reasoning; such as, for instance, are all the perceptions excited by the present discourse, excepting only those which arise from the sight and touch and excepting the immediate pleasure or uneasiness it may occasion. I believe it will not be very necessary to employ many words in explaining this distinction. Every one of himself will readily believe, perceive the difference betwixt feeling and thinking. The common degrees of these are easily distinguished; though it is not impossible but in particular instances they may very nearly approach to each other. Thus, in sleep, in a fever, in madness, or in any very violent emotions of soul, our ideas may approach to our impressions : as on the other hand it sometimes happens, that our impressions are so faint and low, that we cannot distinguish them from our ideas. But not withstanding this near resemblance in a few instances, they are in general so very different, that no one can make a scruple to rank them under distinct heads, and assign to each a peculiar name to mark the difference.

There is another division of our perceptions, which it will be convenient to observe, and which extends itself both to our impressions and ideas. This division is into simple and complex. Simple perceptions, or impressions and ideas, are such as admit of no distinction nor separation. The complex are the contrary to these, and may be distinguished into parts. Though a particular colour, taste and smell, are qualities all united together in this apple, it is easy to perceive they are not the same, but are at least distinguishable from each other.

The passage mainly discusses the origin of

(A) impressions (B) ideas (C) sensations (D) passions

Perceptions that have force may be called

(A) impression (B) emotions (C) difference (D) thinking

Which of the following is excepted from perceptions ?

(A) Faint images of reasoning
(B) Force of liveliness
(C) Those which arise from the sight and touch
(D) The common degree of distinction

Ideas may approach impressions in

(A) resemblance (B) madness (C) readiness (D) consciousness
Simple perceptions and impressions have no

(A) ideas (B) scruples (C) separation (D) quality

Impressions and ideas, in general, are so different, each can be given a particular name to identity

(A) the force (B) the similarity (C) the difference (D) the feeling

December 2010

Read the following passage carefully and answer questions from 5 to 10 :

It should be remembered that the nationalist movement in India, like all nationalist movements, was essentially a bourgeois movement. It represented the natural historical stage of development, and to consider it or to criticise it as a working-class movement is wrong. Gandhi represented that movement and the Indian masses in relation to that movement to a supreme degree, and he became the voice of Indian people to that extent. The main contribution of Gandhi to India and the Indian masses has been through the powerful movements which he launched through the National Congress. Through nation-wide action he sought to mould the millions, and largely succeeded in doing so, and changing them from a demoralised, timid and hopeless mass, bullied and crushed by every dominant interest, and incapable of resistance, into a people with self- respect and self-reliance, resisting tyranny, and capable of united action and sacrifice for a larger cause.

Gandhi made people think of political and economic issues and every village and every bazaar hummed with argument and debate on the new ideas and hopes that filled the people. That was an amazing psychological change. The time was ripe for it, of course, and circumstances and world conditions worked for this change. But a great leader is necessary to take advantage of circumstances and conditions. Gandhi was that leader, and he released many of the bonds that imprisoned and disabled our minds, and none of us who experienced it can ever forget that great feeling of release and exhilaration that came over the Indian people.
Gandhi has played a revolutionary role in India of the greatest importance because he knew how to make the most of the objective conditions and could reach the heart of the masses, while groups with a more advanced ideology functioned largely in the air because they did not fit in with those conditions and could therefore not evoke any substantial response from the masses.

It is perfectly true that Gandhi, functioning in the nationalist plane, does not think in terms of the conflict of classes, and tries to compose their differences. But the action he has indulged and taught the people has inevitably raised mass consciousness tremendously and made social issues vital. Gandhi and the Congress must be judged by the policies they pursue and the action they indulge in. But behind this, personality counts and colours those policies and activities. In the case of very exceptional person like Gandhi the question of personality becomes especially important in order to understand and appraise him. To us he has represented the spirit and honour of India, the yearning of her sorrowing millions to be rid of their innumerable burdens, and an insult to him by the British Government or others has been an insult to India and her people.

Which one of the following is true of the given passage ?

(A) The passage is a critique of Gandhi’s role in Indian movement for independence.
(B) The passage hails the role of Gandhi in India’s freedom movement.
(C) The author is neutral on Gandhi’s role in India’s freedom movement.
(D) It is an account of Indian National Congress’s support to the working-class movement.

The change that the Gandhian movement brought among the Indian masses was

(A) Physical (B) Cultural (C) Technological (D) Psychological

To consider the nationalist movement or to criticise it as a working-class movement was wrong because it was a

(A) historical movement (B) voice of the Indian people
(C) bourgeois movement (D) movement represented by Gandhi

Gandhi played a revolutionary role in India because he could

(A) preach morality (B) reach the heart of Indians
(C) see the conflict of classes (D) lead the Indian National Congress

Groups with advanced ideology functioned in the air as they did not fit in with

(A) objective conditions of masses (B) the Gandhian ideology
(C) the class consciousness of the people (D) the differences among masses

The author concludes the passage by

(A) criticising the Indian masses
(B) the Gandhian movement
(C) pointing out the importance of the personality of Gandhi
(D) identifying the sorrows of millions of Indians

FOR VISUALLY HANDICAPPED CANDIDATES ONLY
Read the following passage carefully and answer the questions from 47 to 52.
[This is from the address delivered by Swami Vivekananda at the World’s Parliament of Religions, Chicago (on 11th September 1893)].

My thanks to some of the speakers on this platform who, referring to the delegates from the orient, have told you that these men from far-off nations may well claim the honour of bearing to different lands the idea of toleration.
I am proud to belong to a religion which has taught the world both tolerance and universal acceptance. We believe not only in universal toleration, but we accept all religions as true. I am proud to belong to a nation which has sheltered the persecuted and the refugees of all religions and all nations of the earth. I am proud to tell you that we have gathered in our bosom the purest remnant of the Israelites who came to Southern India and took refuge with us in the very year in which their holy temple was shattered to pieces by Roman tyranny. I am proud to belong to the religion which has sheltered and is still fostering the remnant of the grand Zoroastrian nation. I will quote to you, brethern, a few lines from a hymn which I remember to have repeated from my earliest boyhood, which is every day repeated by millions of human beings : “As the different streams having their sources in different places all mingle their water in the sea, so, O Lord, the different paths which men take through different tendencies, various though they appear, crooked or straight, all lead to thee”.

The present convention, which is one of the most august assemblies ever held, is in itself a vindication, a declaration to the world of the wonderful doctrine preached in the Gita : “Whosoever comes to me, through whatsoever form, I reach him; all men are struggling through paths which in the end lead to Me.” Sectarianism, bigotry, and its terrible descendant, fanaticism, have long possessed this beautiful earth. They have filled the earth with violence, drenched it often and often with human blood, destroyed civilisation and sent whole nations to despair. Had it not been for these horrible demons, human society would be far more advanced than it is now. But their time is come: and I fervently hope that the bell that tolled this morning in honour of this convention may be the death-knell of all fanaticism, of all persecutions with the sword or with the pen, and of all uncharitable feelings between persons wending their way to the same goal.

Swami Vivekananda said his religion accepted all religions as

(A) from far-off nations (B) of different classes
(C) true (D) of several sects

The purest remnant of the Israelites took refuge in

(A) Rome (B) the United States
(C) Southern India (D) Zoroastrian nation

The different paths of men, though various in appearance, crooked or straight, lead to

(A) universal toleration (B) the Lord
(C) sheltering the persecuted (D) universal acceptance

Swami Vivekananda quoted a declaration to the world from

(A) the Bible (B) the Gita
(C) the welcome speech (D) the Hindu doctrine

The human society would have been more advanced had it not been for

(A) delegates from the Orient (B) those horrible demons
(C) the universal toleration (D) the Roman tyranny

The passage concludes with a hope that the convention would sound the end of

(A) different ideas (B) toleration of others
(C) all persecutions (D) charitable feelings

June 2010

Read the following passage carefully and answer the questions 7 to 12.

The phrase “What is it like ?” stands for a fundamental thought process. How does one go about observing and reporting on things and events that occupy segments of earth space ? Of all the infinite variety of phenomena on the face of the earth, how does one decide what phenomena to observe ? There is no such thing as a complete description of the earth or any part of it, for every microscopic point on the earth’s surface differs from every other such point. Experience shows that the things observed are already familiar, because they are like phenomena that occur at home or because they resemble the abstract images and models developed in the human mind.

How are abstract images formed ? Humans alone among the animals possess language; their words symbolize not only specific things but also mental images of classes of things. People can remember what they have seen or experienced because they attach a word symbol to them.

During the long record of our efforts to gain more and more knowledge about the face of the earth as the human habitat, there has been a continuing interplay between things and events. The direct observation through the senses is described as a percept; the mental image is described as a concept. Percepts are what some people describe as reality, in contrast to mental images, which are theoretical, implying that they are not real.

The relation of Percept to Concept is not as simple as the definition implies. It is now quite clear that people of different cultures or even individuals in the same culture develop different mental images of reality and what they perceive is a reflection of these preconceptions. The direct observation of things and events on the face of the earth is so clearly a function of the mental images of the mind of the observer that the whole idea of reality must be reconsidered.

Concepts determine what the observer perceives, yet concepts are derived from the generalizations of previous percepts. What happens is that the educated observer is taught to accept a set of concepts and then sharpens or changes these concepts during a professional career. In any one field of scholarship, professional opinion at one time determines what concepts and procedures are acceptable, and these form a kind of model of scholarly behaviour.

The problem raised in the passage reflects on

(A) thought process (B) human behaviour
(C) cultural perceptions (D) professional opinion

According to the passage, human beings have mostly in mind

(A) Observation of things (B) Preparation of mental images
(C) Expression through language (D) To gain knowledge

Concept means

(A) A mental image (B) A reality (C) An idea expressed in language form (D) All the above

The relation of Percept to Concept is

(A) Positive (B) Negative (C) Reflective (D) Absolute

In the passage, the earth is taken as

(A) The Globe (B) The Human Habitat (C) A Celestial Body (D) A Planet

Percept means

(A) Direct observation through the senses (B) A conceived idea
(C) Ends of a spectrum (D) An abstract image

[ For Blind Students Only ]

Read the following passage carefully and answer the questions 35 to 40.

During the thousands of years since the earliest records of the history of ideas, learned people have accounted for the order they perceived in the universe in different ways. The accounts range along a continuum from arbitrary rule by humanlike deities, through rule by a deity subject to law, through various kinds of cause and effect relations, to abstract mathematical law. These do not represent successive stages of increasing sophistication, for all of them can be found in the thinking of ancient Greek philosophers as well as in the contemporary world. Rule by a deity or deities is a very ancient concept. In Sumeria, the religious leaders saw a world ruled by living beings like humans but endowed with superhuman powers and with immortality. Each of these beings was responsible for the control and maintenance of some features of the world, such as the flow of rivers, the rise and fall of the tides, the shift of the winds, the productivity of the harvest and the abundance of game animals. The deities competed with one another and reacted arbitrarily and often vindictively to human acts. Other cultures explained matters in terms of a single deity whose acts were frequently subject to the bestowal of human favour. A very different way of accounting for an ordered universe is the recognition of cause and effect sequences that take place in accordance with general law. In some cases the notion of a single deity is retained, but the acts of this deity are not arbitrary. Some would say that this God is the law. The idea of la w itself is an anthropomorphism – that is, a reflection of human experience. Those who break divine laws are subject to punishment, but those who act in harmony with the law are rewarded. Of course, there is a great difference between human law and scientific law : human law governs the behaviour of things, and events are subject to law, but
scientific law is a general description of events.

Order in the universe is perceived through various kinds of relations between

(A) men and matters (B) cause and effect
(C) law and punishment (D) sophistication and crudity

In Sumeria, religious leaders saw the world as ruled by human beings endowed with

(A) rational thinking (B) religious powers
(C) superhuman powers (D) intuitive powers

Law as a reflection of human experience is

(A) punishment or reward for breaking or following it.
(B) that which governs human behaviour.
(C) a general description of events.
(D) that governs natural events.

According to the author, various accounts of the world are not evolutionary because

(A) they are found in the thinking of ancient Greek philosophers
(B) ideas do not grow
(C) they have no relation to abstract mathematical law
(D) they are neither ancient nor contemporary

The Sumerian view of the deities is that

(A) they are governed by a law
(B) they are competing with one another
(C) they are compassionate to human beings
(D) they are governed by a single deity

What is the other way of understanding the world which is different from that of the Sumerians ?

(A) World governed by cause and effect sequences.
(B) World governed by a single deity.
(C) World is governed by a deity whose will is not arbitrary.
(D) World is governed by law.

December 2009

Read the following passage and answer the Question Nos. 13 to 18 :

The decisive shift in British Policy really came about under mass pressure in the autumn and winter of 1945 to 46 – the months which Perderel Moon while editing Wavell’s Journal has perceptively described as ‘The Edge of a Volcano’. Very foolishly, the British initially decided to hold public trials of several hundreds of the 20,000 I.N.A. prisoners (as well as dismissing from service and detaining without trial no less than 7,000). They compounded the folly by holding the first trial in the Red Fort, Delhi in November 1945, and putting on the dock together a Hindu, a Muslim and a Sikh (P.K. Sehgal, Shah Nawaz, Gurbaksh Singh Dhillon). Bhulabhai Desai, Tejbahadur Sapru and Nehru appeared for the defence (the latter putting on his barrister’s gown after 25 years), and the Muslim League also joined the countrywide protest. On 20 November, an Intelligence Bureau note admitted that “there has seldom been a matter which has attracted so much Indian public interest and, it is safe to say, sympathy … this particular brand of sympathy cuts across communal barriers.’ Ajournalist (B. Shiva Rao) visiting the Red Fort prisoners on the same day reported that ‘There is not the slightest feeling among them of Hindu and Muslim … A majority of the men now awaiting trial in the Red Fort is Muslim. Some of these men are bitter that Mr. Jinnah is keeping alive a controversy about Pakistan.’ The British became extremely nervous about the I.N.A. spirit spreading to the Indian Army, and in January the Punjab Governor reported that a Lahore reception for released I.N.A. prisoners had been attended by Indian soldiers in uniform.

Which heading is more appropriate to assign to the above passage ?
(A) Wavell’s Journal (B) Role of Muslim League
(C) I.N.A. Trials (D) Red Fort Prisoners

The trial of P.K. Sehgal, Shah Nawaz and Gurbaksh Singh Dhillon symbolises
(A) communal harmony (B) threat to all religious persons
(C) threat to persons fighting for the freedom (D) British reaction against the natives

I.N.A. stands for
(A) Indian National Assembly (B) Indian National Association
(C) Inter-national Association (D) Indian National Army

‘There has seldom been a matter which has attracted so much Indian Public Interest and, it is safe to say, sympathy … this particular brand of sympathy cuts across communal barriers.’ Who sympathises to whom and against whom ?

(A) Muslims sympathised with Shah Nawaz against the British
(B) Hindus sympathised with P.K. Sehgal against the British
(C) Sikhs sympathised with Gurbaksh Singh Dhillon against the British
(D) Indians sympathised with the persons who were to be trialled

The majority of people waiting for trial outside the Red Fort and criticising Jinnah were the

(A) Hindus (B) Muslims (C) Sikhs (D) Hindus and Muslims both

The sympathy of Indian soldiers in uniform with the released I.N.A. prisoners at Lahore indicates

(A) Feeling of Nationalism and Fraternity (B) Rebellious nature of Indian soldiers
(C) Simply to participate in the reception party (D) None of the above

[For Blind Students Only]
Read the following passage carefully and answer the questions 35 to 40 :
London’s phenomenal growth was probably at its fastest in the sixteenth century, a period when the population was growing everywhere. Many people were attracted to the rapidly growing port, which handled upto 90% of total English foreign trade and gave rise to a wide range of ancillary industries. Other sectors provided goods and services for the court, which now settled permanently in the capital, and for the growing numbers of the rich and ambitious who were attracted by court’s presence. This concentration of the wealthy made London a great leisure centre and the main purveyor of professional services, especially in medicine and law. Population growth now tended to become cumulative. More people needed even more people to provide them with goods and services that they required to survive in a large city. London was always an unhealthy place and mortality rates were much higher than elsewhere in the country, sometime rising to catastrophic heights, as in the terrible plagues of 1603, 1625 and 1665, the last of which killed 80,000 people, 1/6th of the total population of the city. Such high death rates coupled with rather low fertility, meant that London could never grow by its own natural increase. In the century after 1650, when London continued to grow but the English population remained fairly stable, immigration to the city drained the countryside of people and gave London its highest-yet population. It has probably then, in the early eighteenth century London of Defoe and Hogarth, that the city also had its greatest significance as a school centre and a school of manners and ideas for the rest of the country.

London’s rate of population increase was at its greatest in the seventeenth century because

(A) Its death rate was the highest in Britain at that time
(B) It was a time when the population growth was declining everywhere
(C) Migrants were drawn to London because of the vast increase in trading
(D) Its high rate of mortality was offset by a low birthrate

In comparison to the rest of the country, the population decline in London in the seventeenth century was as a result of

(A) frequent outbreaks of plague (B) low rate of fertility
(C) immigration to other parts of the country (D) movement of people to other countries

After 1600 A.D., the demography of London was characterised by

(A) high death rates and birth rates
(B) three major outbreaks of disease, the last of which was responsible for the death of over 80,000 people
(C) a rise in population through natural growth
(D) population stability

Examples of the economic hegemony of London in the seventeenth century include

(A) the movement of the Court permanently to the capital
(B) 9/10th of England’s overseas trade passing through its docks
(C) a large variety of secondary industries growing up around the port of London.
(D) London becoming England’s major intellectual and cultural centre at the time of Defoe and Hogarth.

Which one of the following was not the cause of high rate of death among the people of London ?

(A) Increasing low fertility among people (B) Absence of diseases
(C) Family planning regulations of the State (D) Migration of population

What made London a great leisure centre ?

(A) Growth of ancillary industries.
(B) Concentration of wealth among people who were earlier poor.
(C) Increased rate of mortality.
(D) Due to increase in immigrant population.

June 2009

Read the following passage and answer the questions 11 to 15 :

While the British rule in India was detrimental to the economic development of the country, it did help in starting of the process of modernizing Indian society and formed several progressive institutions during that process. One of the most beneficial institutions, which were initiated by the British, was democracy. Nobody can dispute that despite its many short comings, democracy was and is far better alternative to the arbitrary rule of the rajas and nawabs, which prevailed in India in the pre-British days.
However, one of the harmful traditions of British democracy inherited by India was that of conflict instead of cooperation between elected members. This was its essential feature. The party, which got the support of the majority of elected members, formed the Government while the others constituted a standing opposition. The existence of the opposition to those in power was and is regarded as a hallmark of democracy.
In principle, democracy consists of rule by the people; but where direct rule is not possible, it’s rule by persons elected by the people. It is natural that there would be some differences of opinion among the elected members as in the rest of the society.
Normally, members of any organizations have differences of opinion between themselves on different issues but they manage to work on the basis of a consensus and they do not normally form a division between some who are in majority and are placed in power, while treating the others as in opposition.

The members of an organization usually work on consensus. Consensus simply means that after an adequate discussion, members agree that the majority opinion may prevail for the time being. Thus persons who form a majority on one issue and whose opinion is allowed to prevail may not be on the same side if there is a difference on some other issue.

It was largely by accident that instead of this normal procedure, a two party system came to prevail in Britain and that is now being generally taken as the best method of democratic rule.
Many democratically inclined persons in India regret that such a two party system was not brought about in the country. It appears that to have two parties in India – of more or less equal strength – is a virtual impossibility. Those who regret the absence of a two-party system should take the reasons into consideration.

When the two party system got established in Britain, there were two groups among the rules (consisting of a limited electorate) who had the same economic interests among themselves and who therefore formed two groups within the selected members of Parliament.

There were members of the British aristocracy (which landed interests and consisting of lord, barons etc) and members of the new commercial class consisting of merchants and artisans. These groups were more or less of equal strength and they were able to establish their separate rule at different times.

Answer the following questions :

11. In pre-British period, when India was ruled by the independent rulers :

(A) Peace and prosperity prevailed in the society
(B) People were isolated from political affairs
(C) Public opinion was inevitable for policy making
(D) Law was equal for one and all

What is the distinguishing feature of the democracy practiced in Britain ?

(A) End to the rule of might is right.
(B) Rule of the people, by the people and for the people.
(C) It has stood the test of time.
(D) Cooperation between elected members.

Democracy is practiced where :

(A) Elected members form a uniform opinion regarding policy matter.
(B) Opposition is more powerful than the ruling combine.
(C) Representatives of masses.
(D) None of these.

Which of the following is true about the British rule in India ?

(A) It was behind the modernization of the Indian society.
(B) India gained economically during that period.
(C) Various establishments were formed for the purpose of progress.
(D) None of these.

15. Who became the members of the new commercial class during that time ?

(A) British Aristocrats (B) Lord and barons
(C) Political Persons (D) Merchants and artisans

Read the passage and answer the question 31 to 35 :

The surge witnessed in mergers, amalgamations and take over of companies during the past few years is indicative of the shape of things to come. While these concepts are not new and were recognised even in the Companies Act of 1913, the compulsions have undergone a dramatic change. In the past, mergers and acquisitions were used largely as an instrument for revival of sick units or for obtaining tax benefits. It was not uncommon for a business house to merge a sick company with a profit- making one and claim tax benefits. The objective was not necessarily to achieve faster growth. The liberalisation process witnessed during the late seventies and the eighties and particularly the relaxation of some of the restrictive provisions of MRTP Act and FERA, brought about a qualitative change in the mergers and amalgamations of companies. Even so, the incentive to grow was almost non-existent and in fact some companies preferred to “demerge” by splitting one company into two or more so as to escape from the harsh provisions of the MRTP Act.

The fast pace of liberalisation since July 1991 and the time bound programme of structural reforms under pressure from the IMF and the World Bank have shaken the Indian industry from a slumber by exposing it to internal as well as international competition. Not surprisingly, the pressure is building up on every enterprise to modernise and expand to cut costs. Gone are the days of the licence and permit raj, high import duties and the prosperity guaranteed by a “seller’s market”. With the rising threat of competition and the “seller’s market” giving way to “buyer’s market” in a large number of industries, the compulsion to look for economics of scale in production and cutting down the selling cost is increasing. Simultaneously, the virtual scrapping of the MRTP provisions and relaxation in FERA have removed the disincentives to grow. Hence mergers, amalgamations and take overs have assumed greater importance. Mergers and acquisitions have now come to represent a short cut for companies to achieve accelerated growth. This is the trend world over and India cannot remain an exception as it moves towards globalisation.

What was the motive of some companies behind resorting to demerger ?

(A) to enhance their accountability
(B) to bring about technical changes
(C) to face the competition in market
(D) to elude the harsh legal provisions

What is the meaning of the phrase, “sellers market” giving way to “buyers market” ?

(A) Confrontation between sellers and buyers in a market over price

(B) Market financially in favour of consumers

(C) More demand than supply in a market

(D) Demand and supply proportionate in a market

For which of the following was the amalgamations largely used in the past ?

(A) to achieve accelerated growth

(B) serving on taxes payable to the government

(C) overcoming the provisions of revival of sick units

(D) forcing the government to adopt liberalisation process

Which of the following is true about the government’s stand on import duties ?

(A) Import duty is now raised in order to earn revenue

(B) Import duty is now lowered to encourage healthy competition

(C) Import duty is lowered in order to encourage imports

(D) Import duty is raised in order to discourage buying of foreign goods

The term “demerge” as used in the passage means :

(A) formulation of two or more companies out of an existing one

(B) re-union of companies which had split up out of one company

(C) separation of two or more companies which had merged into one
(D) renaming a company to claim tax benefits

Share to your friend

Mathematical Reasoning and Aptitude

Previous Year Question Paper 2009 to December 2018

June 2019 – Main Paper (Commerce)

  1. Two cyclists start from the same place in opposite directions. One goes towards north at 9 kmph and the other goes towards south at 10 kmph. What time will they take to be 47.5 km apart ?
    (1) 2½ hrs
    (2) 4½ hrs
    (3) 4¼ hrs
    (4) 2¼ hrs
  2. A person incurs 10% loss by selling a watch for Rs. 1,800. At what price should the watch be sold to earn 10% profit?
    1. Rs. 1980
    2. Rs. 2,160
    3. Rs. 2,200
    4. Rs. 2,360
  3. The average age of a class of 59 students is 18 years. If the age of the teacher is included, then the average increases by 3 months. The age of the teacher is
    1. 28 years
    2. 30 years
    3. 33 years
    4. 35 years
  4. The price of item A increases by 50 paise every year while the price of item B increases by 25 paise every year. If in 2008, the price of item A was Rs. 3.20 and that of B was Rs. 5.30, in which year item A will cost 40 paise more than the item B?
    1. 2016
    2. 2017
    3. 2018
    4. 2019
  5. If DRIVER = 12, GOVERNMENT = 20, BELIEVED =16, then BAT = ?
    1.6
    2. 8
    3. 10
    4. 12
  6. In a class of 35 students, Rajan is places seventh from the bottom where as Karan is placed ninth from the top. Divya is places exactly in between the two. What is the differences in the positions of Rajan and Divya?
    1. 9
    2. 10
    3. 11
    4. 13

June 2019 – Main Paper (Management)

  1. Among the five friends, Vineet is taller than Monika, but not as tall as Ram. Jacob is taller than Dalip but shorter than Monika. Who is the tallest in their group?
    1. Ram
    2. Monika
    3. Vineet
    4. Jacob
  2. A trader mixes three varieties of refined oil costing Rs. 100, Rs. 40 and Rs. 60 per litre in the ratio 2:4:3 in terms of litre, and sells the mixture at Rs. 66 per litre. What percentage of profit does he make ?
    1. 12%
    2. 10%
    3. 9%
    4. 8%
  3. If RED is code as 6720, then how would GREEN be coded ?
    1. 16717209
    2. 1677209
    3. 9207716
    4. 1677199
  4. If the average of five numbers is -20 and the sum of three of the numbers is 32, then what is the average of the other two numbers ?
    1. -66
    2. -33
    3. -34
    4. -132
  5. A man travelled from the village to the railway station at the rage of 20 kmph and walked back at the rate of 5 kmph. If the whole journey took 5 hours, find the distance of the railway station from the village.
    1. 10 km
    2. 15 km
    3. 20 km
    4. 40 km

December 2018 – Main Paper (Commerce)

December 2018 – Main Paper (Management)

November 2017

January 2017

July 2016

December 2015

June 2015

December 2014

June 2014

December 2013

September 2013

June 2013

December 2012

June 2012

December 2011

June 2011

December 2010

June 2010

December 2009

June 2009

Share to your friend

Logical Reasoning

Previous Year Question Paper 2009 to December 2018

June 2019 – Main Paper (Commerce)

  1. Consider the argument provided below:
    ‘Sound is impermanent because it is invisible’
    Identify the fallacy involved in the above argument on the basis of Indian logic
    1. Fallacy of irrelevant reason
    2. Fallacy of wrong assertion
    3. Fallacy of trivial reason
    4. Fallacy of contradictory reason
  2. The reasoning which would be helpful in finding new knowledge of facts about the world is
    1. Speculative
    2. Inductive
    3. Deductive
    4. Analogical
  3. Consider the example provided below :
    “This gavaya is like my cow”
    This is an example of which means of knowledge (pramana)
    1. comparison
    2. implication
    3. perception
    4. inference
  4. Identify the minor term in the following syllogism :
    “Some books are not edifying
    All books are interesting
    Therefore, some interesting things are not edifying”
    1. Subject of the conclusion and predicate of the second premises
    2. Predicate of the conclusion and predicate of the first premises
    3. Subject of the both premises
    4. Predicate of the both premises

June 2019 – Main Paper (Management)

  1. Consider the example given below :
    Devadatta is growing fat.
    He does not eat food during the day.
    Therefore, He must be eating food during the night, other things being equal.
    Identify the logic involved in the example provided above by choosing the correct answer from options given below :
    1. Arthapatti
    2. Anupalabdi
    3. Anumana
    4. Sabda
  2. Inductive argument proceeds from
    1. Universal to Particular
    2. Particular to Universal
    3. Universal to Universal
    4. Particular to Particular
  3. Consider the argument provided below :
    “Sound is permanent because it is a product like the jar”.
    Identify the fallacy involved in the above argument on the basis on Indian logic from the options given below.
    1. Fallacy of contradictory reason
    2. Fallacy of wrong assertion
    3. Fallacy of irrelevant reason
    4. Fallacy of trivial reason
  4. No Musicians are Japenese
    All Barbers are Musicians
    No Barbers are Japenese
    Identify the Middle Term in the above syllogism
    1. Subject in the first premise and predicate in the second premise
    2. Predicate in the first premise and subject in the second premise
    3. Predicate in the both premises
    4. Subject in both the premises
  5.  If 0.75 x = 0.02 y, then the value of
    y  xy + xis(1)
    (2)
    (3)
    (4)

December 2018- Main Paper (Commerce)

December 2018- Main Paper (Management)

November 2017

January 2017

July 2016

December 2015

June 2015

December 2014

June 2014

December 2013

September 2013

June 2013

December 2012

June 2012

December 2011

June 2011

December 2010

June 2010

December 2009

June 2009

Share to your friend

Data Interpretation

Previous Year Question Paper 2009 to June 2019

June 2019 – Main Paper (Commerce)

The year-wise student strength in a university from different zones of the country in addition to those students who are from foreign countries is shown in table given below :

Student Strength (in hundred)

Year North Zone South Zone Central Zone West Zone East Zone Foreign Countries
2014 15.1 10.2 6.2 4.0 8.2 1.2
2015 16.2 11.5 5.8 4.1 7.8 1.6
2016 17.2 11.8 6.0 4.5 8.5 1.7
2017 17.4 10.8 6.5 4.8 7.6 1.8
2018 18.0 10.6 6.4 5.4 8.6 2.5
  1. Which was the year when the students from east zone had maximum representation (%) in the total students strength of the university?
    1. 2016
    2. 2018
    3. 2014
    4. 2015
  2. What is the mean of annual growth rates (%) of the student strength of north zone ?
    1. 6.24
    2. 4.51
    3. 3.86
    4. 4.82
  3. During the period 2014-2018, which zone exhibits cyclic variation ?
    1. East Zone
    2. West Zone
    3. Central Zone
    4. South Zone
  4. Which category of students have registered overall maximum growth rate (%) from 2014 to 2018 ?
    1. Foreign countries
    2. West zone
    3. North zone
    4. East zone
  5. What was the percentage (%) of student from foreign countries in the year 2018 ?
    (1) ∼ 4.21 %
    (2) ∼ 3.52 %
    (3) ∼ 5.24 %
    (4) ∼ 4.85 %

June 2019 – Main Paper (Management)

  1. What was the percentage decline in the production of fertilizers from 2013 to 2014 ?
    1. 20.4%
    2. 21.4%
    3. 22.6%
    4. 24.2%
  2. The average production of 2012 and 2013 was exactly equal to the average production of which of the following pairs of years ?
    1. 2011 and 2012
    2. 2011 and 2017
    3. 2012 and 2014
    4. 2015 and 2016
  3. In which year, the percentage increase in production as compared to the previous year was maximum ?
    1. 2012
    2. 2013
    3. 2015
    4. 2018
  4. What was the approximate percentage increase in production of fertilizers in 2018 compared to that in 2011?
    1. 157%
    2. 165%
    3. 177%
    4. 182%
  5. In how many of the given years was the production of fertilizers more than the average production of the given years?
    1. 2
    2. 3
    3. 4
    4. 5

December 2018 – Main Paper (Commerce)

December 2018 – Main Paper (Commerce)

November 2017

January 2017

July 2016

December 2015

June 2015

December 2014

June 2014

December 2013

September 2013

June 2013

December 2012

June 2012

December 2011

June 2011

December 2010

June 2010

December 2009

June 2009

Share to your friend

Information and Communication Technology 

Previous Year Question Paper 2009 to June 2019

June 2019 – Main Paper (Commerce)

  1. The following teaching aids can be used by a teacher in the classroom as a live interactive device :
    (a) E-mail
    (b) Skype
    (c) On-line Chat
    (d)Tele-conferencing
    (e) Face-book
    (f) Blog
    Select your answer from the options given below :
    (1) (a),(b) and (c)
    (2) (b),(c) and (d)
    (3) (c),(e) and (f)
    (4) (a), (b) and (d)
  2. ERIC, MEDLINE and ABI/INFORM are instances of
    1. Citation services
    2. Electronic databases
    3. Printed databases
    4. Libraries
  3. Identify the encompassing term that, involves the use of electronic platforms to conduct a company’s business.
    1. E-Business
    2. E-Commerce
    3. E-Marketing
    4. E-Governance
  4. Which of the following is not considered as a form of secondary storage ?
    1. RAM
    2. Floppy Disk
    3. Optical Disk
    4. Flash Drive
  5. Which of the following devices enables a computer to transmit data over telephone lines ?
    1. CPU
    2. Hub
    3. Switch
    4. Modem
  6. Which of the following is generally associated with the domain name of an educational institution ?
    1. .org
    2. .edu
    3. .com
    4. .inst

June 2019- Main Paper (Commerce)

  1. Match the following
    Set – I
    (a) First Generation
    (b) Second Generation
    (c) Third Generation
    (d) Fourth Generation
    Set – II
    (i) Transistor
    (ii) VLSI microprocessor
    (iii) Vacuum tube
    (iv) Integrated circuit
    Select the correct option from the following
    (a) (b) (c) (d)
    (1) (iii) (iv) (i) (ii)
    (2) (iii) (i) (iv) (ii)
    (3) (iii) (i) (ii) (iv)
    (4) (i) (iii) (iv) (ii)
  2. An Audio and Video compression, the term RGB means
    1. Red, Grey Blue
    2. Red, Green, Blue
    3. Red, Green, Black
    4. Red, Grey, Blue
  3. One Tera Byte (TB) of memory is equal to
    1. 1024 KB
    2. 1024 X 1024 KB
    3. 1024 X 1024 X 1024 KB
    4. 1024 X1024 X 1024 BYTES
  4. A/An ———————-is a computer program that spreads by inserting copies of itself into other executable code or documents.
    1. Operating System
    2. Computer Virus
    3. Firewell
    4. Anti-virus
  5. Which of the following is not a/an web browser?
    1. Internet Explorer
    2. Mozilla Firefox
    3. Google chrome
    4. Yahoo

December 2018- Main Paper (Commerce)

December 2018- Main Paper (Management)

November 2017

January 2017

July 2016

December 2015

June 2015

December 2014

June 2014

December 2013

September 2013

June 2013

December 2012

June 2012

December 2011

June 2011

December 2010

June 2010

December 2009

June 2009

Share to your friend

Higher Education System

Previous Year Question Paper 2009 to June 2019

June 2019 – Main Paper (Commerce)

  1. Which of the following commission/committee in the post-independent India has paid attention all the levels of education ?
    1. Radhakrishnan Commission
    2. Mudaliar Commission
    3. Kothari Commission
    4. Acharya Rammurthi Committees
  2. Identify the scheme which aims to encourage Social Science Research in Policy relevant areas so as to provide vital inputs in policy formulation, implementation and evaluation
    1. ICSSR Fellowship
    2. Cognitive Initiatives
    3. ICSSR-IMPRESS
    4. ICSSR Research Projects
  3. The purpose of Gururkul system of education is to
    1. Promote equality and excellence
    2. Minimise stress in learning
    3. Empowering for future learning
    4. Encourage self-help
  4. NMEICT is an acronym for
    1. National Massive Education through Information and Communication Technology
    2. National Mission on Education through Information and Communication Technology
    3. National Multipurpose Education through Information and Communication Technology
    4. National Mode of Education through Information and Communication Technology
  5. The scheme ‘Unnat Bharat Abhiyan’ aims to
    1. Increase strength of students in colleges in India
    2. Link higher educational institution in the country with at least 5 villages
    3. Increase number of Higher Education
    4. Increase number of colleges in India

June 2019 – Main Paper (Management)

  1. The report of which of the following Education Commissions carries the sub-title ‘Education for National Development’ ?
    1. Radhakrishnan Commission
    2. Mudaliar Commission
    3. Calcutta University Education Commission
    4. Kothari Commission
  2. In which year the modern Indian University got established ?
    1. 1957
    2. 1857
    3. 1947
    4. 1964
  3. In which era given below, higher education in India got a set back?
    1. British Era
    2. Buddhist Era
    3. Mughal Era
    4. Port-Independence Era
  4. The expansion of SANKALP is,
    1. Sanskrit Awareness and Knowledge Acquisition of Language Programme
    2. Skills Acquisition and Karatae Awareness for Livelihood Promotion
    3. Skill Acquisition and Knowledge Awareness for Livelihood Promotion
    4. Sanskrit Awareness and Knowledge Acquisiton for Livelihood Promotion

December 2018 – Main Paper (Commerce)

  1. 1. For pursuing a Ph.D. Programme through our University system, what has become a mandatory provision now?
    1. Guidance from a supervisor outside the University
    2. Enrolment for Ph.D.
    3. Course work
    4. Compulsory attendance for three years
    2. In the institutions of higher education in India which of the following has the formal authority to approve the courses and programmes of studies ?
    1. Departmental Council
    2. Academic Council
    3. Board of Studies
    4. The University Court/ Senate3. In the following two sets, Set-I mentions an apex level institution in India while Set-II indicates their formal concerns. Match the two sets and give your answer from the code.
    Set-I
    (Appex level Institution)
    (a) University Grants Commession
    (b) All India Council of Technical Education     
    (c) National Council of Teacher Education
    (d) National Assessment and Accreditation Council
    Set II
    (Formal concerns)
    (i) Adjudication of legal matters in education
    (ii) Coordination, recognition and quality issues related to teacher education institutions
    (iii) Coordination and quality maintenance in institutions of higher education
    (iv) Recognition and quality issues in technical institutions
    (v) Evaluation of quality and determining grade of excellence
    a b c d
    1. (v) (iv) (iii (ii)
    2. (iii) (iv) (ii) (v)
    3. (ii) (iii) (i) (iv)
    4. (i) (ii) (iii) (iv)

    4. Below are given two sets- Set-I provides stages of value development while Set-II indicates their critical features from the point of view of value education. Match the two sets and choose your answer from the code.
    Set-I
    (Stages and value development)
    (a) value collection
    (b) value assessment
    (c) value clarification
    (d) value consolidation

    Set II
    (Critical features in terms of value education)
    (i) Providing opportunity for imitation
    (ii) Characterization through integration of values
    (iii) Providing hints for inner evaluation
    (iv) Exposure to other values
    (v) Discussions and debates

    a   b    c    d
    1. (i) (ii) (iii) (iv)
    2. (ii) (iii) (i) (v)
    3. (i) (iii) (iv) (ii)
    4. (iii) (i) (ii) (v)

    5. To improve access and quality in higher education, the Knowledge Commission recommendations include which of the following ?

    (a) Use of ICT for production of knowledge
    (b) Closure of non-performing Universities in the country
    (c) Establishment of a network of institutions of higher learning
    (d) Increase in the enrollment of students in the institutions of higher learning
    Code :
    1. (a), (b), (c) only
    2. (a), (b), (c), (d) only
    3. (b), (c), (d) only
    4. (a), (b), (d) only

December 2018 (Management)

35. Match the following :
List – I List – II
(a) Open University (i) Odisha State Open University, Odisha
(b) Dual Mode University (ii) Amity University
(c) Mixed Mode University (iii) Indian Institute of Science, Bengaluru
(d) Deemed University (iv) University of Mumbai, Mumbai
(e) Specialised University (v) School of Planning and Architecture, New Delhi
The correct code is :
(a) (b) (c) (d) (e)
1. (i) (iv) (ii) (iii) (v)
2. (v) (i) (iv) (ii) (iii)
3. (iii) (ii) (v) (i) (iv)
4. (ii) (iii) (i) (iv) (v)
36. One of the first states to establish a higher education council was :
1. Jammu and Kshmir
2. Goa
3. Andhra Pradesh
4. Sikkim
38. The number of open universities and directorates of distance education whose courses are recognised by UGC as on August 9, 2018, is “
1. 58
2. 53
3. 49
4. 62
39. A study was carried out on the drought resilience in India. Which of the following States show highest level of handling drought ?
1. Sikkim, Punjab, Arunachal Pradesh
2. Jharkhand, Chhatisgarh, Jammu and Kashmir
3. Telangana, Andhra Pradesh, Tamil Nadu
4. Odisha, Karnataka, Kerala
44. State higher education councils are established under the instruction of :
1. CABE
2. RUSA
3. NITI Ayog
4. PUSA

48. Which private institutions in the following list have been granted the status of ‘Institutes of Eminence’ by the MHRD?
(i) BITS, Pilani
(ii) MAHE, Manipal
(iii) Symbiosis, Pune
(iv) Amity University, Noida
Choose the correct answer from the code given below :
Code :
1. (i) and (ii)
2. (ii) and (iv)
3. (ii) and (iii)
4. (i) and (iii)

November 2017

July 2018 Set P

46. India has the largest Higher Education System in the World after :
(a) The United States of America (b) Australia
(c) China (d) United Kingdom (U.K.)
Select the correct answer from the code given below :
(1) (a), (b), (c) and (d) (2) (a), (b) and (c) only
(3) (a), (c) and (d) only (4) (a) and (c) only

47. Prime Minister Research Fellowship is for students pursuing Ph. D programme in :

(1) State and Central Universities
(2) Central Universities, IISc, IITs, NITs, IISERs and IIITs
(3) IISc, IITs, NITs, IISERs, IIITs, State and Central Universities
(4) IITs and IISc

Polity

48. Leader of the Opposition is a member of committees which select :

(a) the Central Information Commissioner
(b) the Central Vigilance Commissioner
(c) the Chairperson of National Human Rights Commission
(d) the Chairperson of National Commission for Women

Select the correct answer from the code given below :
(1) (a), (b), (c) and (d) (2) (a), (b) and (c) only
(3) (a), (c) and (d) only (4) (a), (b) and (d) only

49. Which of the following statements are correct about gender budgeting ?

(a) It is a separate budget addressing the specific needs of women.
(b) It assesses the impact of government budget on women.
(c) It is an accounting exercise.
(d) It is another budgeting innovation.

Select the correct answer from the code given below :

(1) (b) and (d) only (2) (a) and (d) only
(3) (a), (c) and (d) only (4) (b), (c) and (d) only

50. Which of the following are the barriers to citizen-centric administration in India ?

(a) Wooden and inflexible attitude of the civil servants
(b) Ineffective implementation of laws and rules
(c) Awareness of rights and duties of citizens
(d) Lack of job opportunities for the youth

Select the correct answer from the code given below :

(1) (a), (b), (c) and (d) (2) (a), (b) and (c) only
(3) (a), (b) and (d) only (4) (a) and (b) only

NOVEMBER 2017 Set A

46. Which of the following are the goals of higher education in India ?
(a) Access (b) Equity (c) Quality and Excellence
(d) Relevance (e) Value based education (f) Compulsory and free education
Select the correct answer from the code given below :
(1) (a), (b) and (e) only (2) (a), (b), (e) and (f)
(3) (a), (b), (c), (d) and (e) (4) (a), (b), (c), (d), (e) and (f)

47. Which of the following has been ranked the best college in the country (2017) as per the
National Institutional Ranking Framework (NIRF) ?
(1) Miranda House, Delhi (2) St. Stephen’s College, Delhi
(3) Fergusson College, Pune (4) Maharaja’s College, Mysore

48. Which of the following universities has received the Visitor’s Award for the best Central University in India in Feb. 2017 ?
(1) Jawaharlal Nehru University (2) Banaras Hindu University
(3) Tezpur University (4) University of Hyderabad

Polity

49. Who among the following can be removed by the President without Parliament’s resolution ?

(1) Judge of a High Court (2) Governor of a State
(3) Chief Election Commissioner (4) Comptroller and Auditor – General

50. Which of the following come(s) within the ambit of the term ‘corruption’ ?

(a) Misuse of official position
(b) Deviation from rules, laws and norms
(c) Non-action when action is required
(d) Harm to public good

Select the correct answer from the code given below :

(1) (a) only (2) (a) and (b) only
(3) (a), (b) and (d) (4) (a), (b), (c) and (d)

JANUARY 2017 Set W

49. Which of the following are the demerits of globalisation of higher education ?
(a) Exposure to global curriculum
(b) Promotion of elitism in education
(c) Commodification of higher education
(d) Increase in the cost of education

Select the correct answer from the codes given below :
Codes :
(1) (a) and (d) (2) (a), (c) and (d)
(3) (b), (c) and (d) (4) (a), (b), (c) and (d)

50. Which of the following statements are correct about deemed universities ?
(a) The Governor of the State is the chancellor of deemed universities.
(b) They can design their own syllabus and course work.
(c) They can frame their own guidelines regarding admission and fees.
(d) They can grant degrees.

Select the correct answer from the codes given below :
Codes :
(1) (a), (b) and (c) (2) (b), (c) and (d)
(3) (a), (c) and (d) (4) (a), (b), (c) and (d)

51. The purpose of value education is best served by focussing on
(1) Cultural practices prevailing in the society.
(2) Norms of conduct laid down by a social group.
(3) Concern for human values.
(4) Religious and moral practices and instructions.

52. Which of the following statements are correct ?
(a) Rajya Sabha is a permanent House which can be dissolved only during national emergency.
(b) Rajya Sabha does not represent the local interests of the States.
(c) Members of the Rajya Sabha are not bound to vote at the dictates of the states they
represent.
(d) No Union territory has a representative in the Rajya Sabha. Select the correct answer from
the codes given below :
Codes :
(1) (a) and (d) (2) (b) and (c)
(3) (b), (c) and (d) (4) (a), (b), (c) and (d)

53. Which of the following are not necessarily the immediate consequences of the
proclamation of the President’s Rule in a State ?
(a) Dissolution of the State Assembly.
(b) Removal of the Council of Ministers in the State.
(c) Takeover of the State administration by the Union Government.
(d) Appointment of a new Chief Secretary.

Select the correct answer from the codes given below :

Codes :

(1) (a) and (d) (2) (a), (b) and (c)
(3) (a), (b), (c) and (d) (4) (b) and (c)

54. Instead of holding the office during the pleasure of the President who among the following
hold(s) office during good behaviour ?

(a) Governor of a State (b) Attorney General of India
(c) Judges of the High Court (d) Administrator of a Union Territory

Select the correct answer from the codes given below :

Codes :

(1) (a) only (2) (c) only
(3) (a) and (c) (4) (a), (b), (c) and (d)

47. Which of the following universities/institutes is ranked first in the India Rankings 2016 ?

(1) Jawaharlal Nehru University, New Delhi (2) Central University, Hyderabad
(3) Indian Institute of Science, Bangalore (4) Institute of Chemical Technology, Mumbai

48. Which of the following statements are correct about distance education in India ?

(a) It supplements formal education. (b) It reduces the cost of education.
(c) It replaces the formal education. (d) It enhances access to education.

Select the correct answer from the codes given below :

Codes :

(1) (a), (b), (c) and (d) (2) (a), (c) and (d)
(3) (a), (b) and (d) (4) (b), (c) and (d)

49. Which of the following are statutory bodies ?

(a) Election Commission of India
(b) University Grants Commission (UGC)
(c) All India Council for Technical Education (AICTE)
(d) National Assessment and Accreditation Council (NAAC) Select the correct answer from the codes given below :
Codes :
(1) (a), (b) and (c) (2) (b), (c) and (d)
(3) (b) and (c) (4) (b) and (d)

50. Which of the following statements are not correct in respect of Rajya Sabha ?
(a) It has same powers as those of Lok Sabha.
(b) It can pass a vote of no-confidence against the Council of Ministers.
(c) It can amend a money bill.
(d) It can be dissolved during emergency.
Select the correct answer from the codes given below :
Codes :
(1) (b) and (c) (2) (b), (c) and (d)
(3) (a), (b) and (c) (4) (a), (b), (c) and (d)

51. Which of the following statement(s) is/are correct about the powers of the Governor of a
State ?
(a) He has the power to dissolve the Legislative Assembly.
(b) He has the power to appoint judges of the State High Court.
(c) He has the power to grant pardon in case of death sentence.
(d) He has diplomatic powers.
Select the correct answer from the codes given below :
Codes :
(1) (a) only (2) (a) and (b)
(3) (a), (b) and (d) (4) (a), (b) and (c)

52. In which of the following cases, the principle of natural justice (Hear the other party)
must be followed ?
(a) Dismissal of an employee
(b) Supersession of a municipality
(c) Threat to national security
(d) Disciplinary action against a student or an employee Select the correct answer from the
codes given below :
Codes :
(1) (a) and (b) (2) (a) and (d)
(3) (a), (b) and (d) (4) (a), (b), (c) and (d)

59. For advancing knowledge, the latest strategy used by mankind is
(1) consulting an authority (2) deductive reasoning
(3) scientific thinking (4) inductive reasoning

JUNE 2016 Set A

3. Assertion (A) : The purpose of higher education is to promote critical and creative thinking abilities among students.
Reason (R) : These abilities ensure job placements.

Choose the correct answer from the following code :

(1) Both (A) and (R) are true and (R) is the correct explanation of (A).
(2) Both (A) and (R) are true but (R) is not the correct explanation of (A).
(3) (A) is true but (R) is false.
(4) (A) is false but (R) is true.

53. Which of the following core values among the institutions of higher education
Are promoted by the NAAC (National Assessment and Accreditation Council) ?
(a) Contributing to national development.
(b) Fostering global competencies among the students.
(c) Inculcating a value system among students and teachers.
(d) Promoting the optimum utilization of the infrastructure. Select the correct answer from the codes given below :
Codes :
(1) (b), (c) and (d) (2) (a), (b) and (c)
(3) (a), (c) and (d) (4) (a), (b), (c) and (d)

55. The National Judicial Appointments Commission (NJAC) has been declared
unconstitutional by
(1) The Supreme Court of India
(2) The High Court
(3) The High Court and the Supreme Court both
(4) The President of India

56. Which of the following statements about the Indian political system is/are correct ?
(a) The President is both Head of the State and Head of the Government.
(b) Parliament is Supreme.
(c) The Supreme Court is the guardian of the Constitution.
(d) The Directive Principles of State Policy are justiciable. Select the correct answer from the
codes given below :
(1) (a), (b), (c) and (d) (2) (b), (c) and (d)
(3) (b) and (c) (4) (c) only

57. Which of the following are the fundamental duties ?
(a) To respect the National Flag.
(b) To protect and improve the natural environment.
(c) For a parent to provide opportunities for education to his/her child.
(d) To protect monuments and places of national importance. Select the correct answer from the
codes given :
Codes :
(1) (a), (b) and (c) (2) (a), (b) and (d)
(3) (a), (c) and (d) (4) (a), (b), (c) and (d)

58. Which of the following statements are correct in respect of Niti Aayog ?
(a) It is a constitutional body.
(b) It is a statutory body.
(c) It is neither a constitutional body nor a statutory body.
(d) It is a think-tank.
Select the correct answer from the codes given below :
(1) (a) and (d) (2) (b) and (d)
(3) (c) and (d) (4) (b), (c) and (d)

59. A college level assistant professor has planned his/her lectures with an intent to develop
cognitive dimensions of students centered on skills of analysis and synthesis. Below, given are two sets of items Set – I consisting of levels of cognitive interchange and Set – II comprising basic requirements for promoting them. Match the two sets and indicate your answer by choosing the correct alternative from the code :

Set – I Set – II
(Levels of Cognitive Interchange) (Basic requirements for promoting cognitive interchange)

a. Memory level i. Giving opportunity for discriminating examples and non-examples of a point.
b. Understanding level ii. Recording the important points made during the presentations.
c. Reflective level iii. Asking the students to discuss various items of information.
iv. ritically analyzing the points to be made and discussed.
Codes :
a b c

(1) ii iv i
(2) iii iv ii
(3) ii i iv
(4) i ii iii

DECEMBER 2015 Set P

1. Greater the handicap of the students coming to the educational institutions, greater the
demand on the :

(1) Family (2) Society (3) Teacher (4) State

5. As Chairman of an independent commission on education, Jacques Delors report to UNESCO was titled :

(1) International Commission on Education Report
(2) Millennium Development Report
(3) Learning : The Treasure Within
(4) World Declaration on Education for All

55. Which of the following are the objectives of Rashtriya Uchchatar Shiksha Abhiyan (RUSA) ?

(a) To improve the overall quality of state institutions.
(b) To ensure adequate availability of quality faculty.
(c) To create new institutions through upgradation of existing autonomous colleges.
(d) To downgrade universities with poor infrastructure into autonomous colleges.

Select the correct answer from the codes given below :

(1) (a), (b), (c) and (d) (2) (a), (b) and (c)
(3) (a), (c) and (d) (4) (a), (b) and (d)

56. The grounds on which discrimination in admission to educational institutions is constitutionally prohibited are :
(a) Religion (b) Sex (c) Place of birth (d) Nationality

Select the correct answer from the codes given below :
(1) (b), (c) and (d) (2) (a), (b) and (c)
(3) (a), (b) and (d) (4) (a), (b), (c) and (d)

57. Which of the following statements are correct about Lok Sabha ?
(a) The Constitution puts a limit on the size of the Lok Sabha.
(b) The size and shape of the Parliamentary Constituencies is determined by the Election Commission.
(c) First past the Post electoral system is followed.
(d) The Speaker of Lok Sabha does not have a casting vote in case of an equality of votes.

Select the correct answer from the codes given below :

(1) (a) and (c) (2) (a), (b) and (c)
(3) (a), (c) and (d) (4) (a), (b), (c) and (d)

58. Public Order as an item in the Constitution figures in :
(1) the Union List (2) the State List
(3) the Concurrent List (4) the Residuary Powers

59. The term of office of the Advocate General of a State is :
(1) 4 years
(2) 5 years
(3) 6 years or 65 years of age whichever is earlier
(4) not fixed

60. Which among the following States has the highest number of seats in the Lok Sabha ?
(1) Maharashtra (2) Rajasthan (3) Tamil Nadu (4) West Bengal

JUNE 2015 Set A

55. The session of the parliament is summoned by :

(1) The President
(2) The Prime Minister
(3) The Speaker of the Lok Sabha
(4) The Speaker of the Lok Sabha and the Chairman of the Rajya Sabha

56. Civil Service Day is celebrated in India on :

(1) 21st April (2) 24th April (3) 21st June (4) 7th July

57. The South Asia University is situated in the city of :

(1) Colombo (2) Dhaka (3) New Delhi (4) Kathmandu

58. The University Grants Commission was established with which of the following aims ?

(a) Promotion of research and development in higher education
(b) Identifying and sustaining institutions of potential learning
(c) Capacity building of teachers
(d) Providing autonomy to each and every higher educational institution in India Select the

correct answer from the codes given below :
(1) (a), (b), (c) and (d) (2) (a), (b) and (c)
(3) (b), (c) and (d) (4) (a), (b) and (d)

59. The Gross Enrolment Ratio (GER) in institutions of higher education in India at present (2015) is about :

(1) 8 percent (2) 12 percent (3) 19 percent (4) 23 percent

60. The total number of central universities in India in April 2015 was :

(1) 0 8 (2) 1 4 (3) 2 7 (4) 4 3

DECEMBER 2014 Set W

42. The maximum number of fake institutions / universities as identified by the UGC in the year 2014 are in the State / Union territory of

(A) Bihar (B) Uttar Pradesh (C) Tamil Nadu (D) Delhi

43. Which of the following institutions are empowered to confer or grant degrees under the UGC Act, 1956 ?

1. A university established by an Act of Parliament.
2. A university established by an Act of Legislature.
3. A university / institution established by a linguistic minority.
4. An institution which is a deemed to be university.

Select the correct answer from the codes given below :

(A) 1 and 2 ` (C) 1, 2 and 4 (B) 1, 2 and 3 (D) 1, 2, 3 and 4

44. Which of the following are the tools of good governance ?

1. Social Audit
2. Separation of Powers
3. Citizen’s Charter
4. Right to Information

Select the correct answer from the codes given below :

(A) 1, 3 and 4 (B) 2, 3 and 4

(C) 1 and 4 (D) 1, 2, 3 and 4

45. Which of the following powers, the President has in relation to Lok Sabha ?
1. Summoning

2. Adjournment – sine die

3. Prorogation

4. Dissolution

Select the correct answer from the codes given below :

(A) 1 and 4

(B) 1, 2 and 3

(C) 1, 3 and 4

(D) 1, 2, 3 and 4

46. The interval between two sessions of parliament must not exceed

(A) 3 months

(B) 6 months

(C) 4 months

(D) 100 days

47. Right to Privacy as a Fundamental Right is implied in

(A) Right to Freedom

(B) Right to Life and Personal Liberty

(C) Right to Equality

(D) Right against Exploitation

48. Which of the following organizations deals with ‘capacity building program’ on Educational Planning ?

(A) NCERT (B) UGC (C) NAAC (D) NUEPA

49. “Education is the manifestation of perfection already in man” was stated by

(A) M.K. Gandhi (B) R.N. Tagore (C) Swami Vivekanand (D) Sri Aurobindo

JUNE 2014 Set W

38. Who among the following is the de facto executive head of the Planning Commission ?

(A) Chairman (B) Deputy Chairman
(C) Minister of State for Planning (D) Member Secretary

39. Education as a subject of legislation figures in the

(A) Union List (B) State List
(C) Concurrent List (D) Residuary Powers

40. Which of the following are Central Universities ?

1. Pondicherry University 2. Vishwa Bharati
3. H.N.B. Garhwal University 4. Kurukshetra University

Select the correct answer from the code given below :

Codes :
(A) 1, 2 and 3 (B) 1, 3 and 4 (C) 2, 3 and 4 (D) 1, 2 and 4

42. Which of the following universities has adopted the meta university concept ?

(A) Assam University (B) Delhi University
(C) Hyderabad University (D) Pondicherry University

43. Which of the following statements are correct about a Central University ?

1. Central University is established under an Act of Parliament.
2. The President of India acts as the visitor of the University.
3. The President has the power to nominate some members to the Executive Committee or the Board of Management of the University.
4. The President occasionally presides over the meetings of the Executive Committee or Court.

Select the correct answer from the code given below :

Codes :

(A) 1, 2 and 4 (B) 1, 3 and 4
(C) 1, 2 and 3 (D) 1, 2, 3 and 4

47. The e-content generation for under-graduate courses has been assigned by the Ministry of Human Resource Development to

(A) INFLIBNET (B) Consortium for Educational Communication
(C) National Knowledge Commission (D) Indira Gandhi National Open University

DECEMBER 2013 Set W
23. Virtual reality provides

(A) Sharp pictures (B) Individual audio
(C) Participatory experience (D) Preview of new films

24. The first virtual university of India came up in

(A) Andhra Pradesh (B) Maharashtra
(C) Uttar Pradesh (D) Tamil Nadu

32. Which one of the following Councils has been disbanded in 2013 ?

(A) Distance Education Council (DEC)
(B) National Council for Teacher Education (NCTE)
(C) National Council of Educational Research and Training (NCERT)
(D) National Assessment and Accreditation Council (NAAC)

33. Which of the following statements are correct about the National Assessment and Accreditation Council ?

1. It is an autonomous institution.
2. It is tasked with the responsibility of assessing and accrediting institutions of higher education.
3. It is located in Delhi.
4. It has regional offices.

Select the correct answer from the codes given below :

Codes :

(A) 1 and 3 (B) 1 and 2 (C) 1, 2 and 4 (D) 2, 3 and 4

34. The power of the Supreme Court of India to decide disputes between two or more States falls under its

(A) Advisory Jurisdiction (B) Appellate Jurisdiction
(C) Original Jurisdiction (D) Writ Jurisdiction

35. Which of the following statements are correct ?

1. There are seven Union Territories in India.
2. Two Union Territories have Legislative Assemblies
3. One Union Territory has a High Court.
4. One Union Territory is the capital of two States.

Select the correct answer from the codes given below :

(A) 1 and 3 only (B) 2 and 4 only (C) 2, 3 and 4 only (D) 1, 2, 3 and 4

36. Which of the following statements are correct about the Central Information Commission ?
1. The Central Information Commission is a statutory body.
2. The Chief Information Commissioner and other Information Commissioners are appointed by the President of India.
3. The Commission can impose a penalty upto a maximum of Rs 25,000/-
4. It can punish an errant officer.

Select the correct answer from the codes given below :

Codes :

(A) 1 and 2 only (B) 1, 2 and 4 (C) 1, 2 and 3 (D) 2, 3 and 4

37. Who among the following conducted the CNN-IBN – The Hindu 2013 Election Tracker Survey across 267 constituencies in 18 States ?

(A) The Centre for the Study of Developing Societies (CSDS)
(B) The Association for Democratic Reforms (ADR)
(C) CNN and IBN
(D) CNN, IBN and The Hindu

58. The Verstehen School of Understanding was popularised by

(A) German Social Scientists (B) American Philosophers
(C) British Academicians (D) Italian Political Analysts

29. Which of the following is/are a minority institution(s) ?

1. Punjabi University, Patiala 2. Osmania University, Hyderabad
3. Kashmir University, Srinagar 4. St. Stephens College, Delhi

Select the correct answer from the code given below :

Codes :

(A) 1 and 2 (B) 2 and 4 (C) 2 only (D) 4 only

30. Which of the following statements are correct about the National Advisory Council (NAC) ?

1. The National Advisory Council is a statutory body.
2. It is headed by the Prime Minister of India.
3. It facilitates constant interaction with the leaders of civil society.
4. It provides policy and legislative inputs to the Government of India.

Select the correct answer from the code given below :

Codes :

(A) 1, 2 and 3 (B) 2, 3 and 4 (C) 1, 3 and 4 (D) 3 and 4

31. Which of the following are voluntary provisions in the 73rd Constitutional Amendment Act (1992) ?

1. Minimum age of 21 for contesting elections to Panchayats.
2. Indirect elections to the post of Chairperson of Panchayats at the intermediate and district levels.
3. Representation of Members of Parliament and State Legislature on Panchayati Raj institutions.
4. Reservation of seats for backward classes.

Select the correct answer from the code given below :

Codes :

(A) 1, 2 and 4 (B) 2, 3 and 4 (C) 1, 2 and 3 (D) 3 and 4

32. In which of the following States the segment of population, which is in majority, enjoys the benefit of reservation of seats in the State Assembly ?

(A) Meghalaya and Mizoram (B) Assam and Nagaland
(C) Madhya Pradesh and Assam (D) Rajasthan and Arunachal Pradesh

33. Which of the following are the ways of acquiring Indian citizenship ?

1. Birth 2. Descent 3. Naturalisation 4. Incorporation of territory

Select the correct answer from the code given below :

Codes :

(A) 1 and 2 (B) 1 and 4 (C) 1, 2 and 3 (D) 1, 2, 3 and 4

34. Which of the following statements about the Union Public Service Commission are correct ?
1. UPSC is a Constitutional body.
2. It serves as an advisory body.
3. It is called upon to advise the Government in regard to representation of the Scheduled Castes
and Scheduled Tribes in the Civil Service.
4. It is consulted on appointments of Chairman and members of Tribunals and Commissions.

Select the correct answer from the code given below :

Codes :
(A) 1, 2 and 3 (B) 1, 2 and 4 (C) 1, 3 and 4 (D) 1 and 2

46. In which of the following categories the enrolment of students in higher education in 2010-11 was beyond the percentage of seats reserved ?

(A) OBC students (B) SC students (C) ST students (D) Woman students

47. Which one of the following statements is not correct about the University Grants Commission
(UGC) ?

(A) It was established in 1956 by an Act of Parliament.
(B) It is tasked with promoting and coordinating higher education.
(C) It receives Plan and Non-Plan funds from the Central Government.
(D) It receives funds from State Governments in respect of State Universities.

48. Consider the statement which is followed by two arguments (I) and (II) :

Statement : Should India switch over to a two party system ?
Arguments : (I) Yes, it will lead to stability of Government.
(II) No, it will limit the choice of voters.

(A) Only argument (I) is strong. (B) Only argument (II) is strong.
(C) Both the arguments are strong. (D) Neither of the arguments is strong.

49. Consider the statement which is followed by two arguments (I) and (II) :
Statement : Should persons with criminal background be banned from contesting elections ?
Arguments : (I) Yes, it will decriminalise politics.
(II) No, it will encourage the ruling party to file frivolous cases against their political opponents.

(A) Only argument (I) is strong. (B) Only argument (II) is strong.
(C) Both the arguments are strong. (D) Neither of the arguments is strong.

50. Which of the following statement(s) is/are correct about a Judge of the Supreme Court of India ?
1. A Judge of the Supreme Court is appointed by the President of India.
2. He holds office during the pleasure of the President.
3. He can be suspended, pending an inquiry.
4. He can be removed for proven misbehaviour or incapacity.

Select the correct answer from the codes given below :

Codes :

(A) 1, 2 and 3 (B) 1, 3 and 4
(C) 1 and 3 (D) 1 and 4

51. In the warrant of precedence, the Speaker of the Lok Sabha comes next only to

(A) The President (B) The Vice-President
(C) The Prime Minister (D) The Cabinet Ministers

DECEMBER 2012 Set W

38. India has the highest number of students in colleges after

(A) the U.K. (B) the U.S.A. (C) Australia (D) Canada

39. Which of the following statement(s) is/are not correct about the Attorney General of India ?
1. The President appoints a person, who is qualified to be a Judge of a High Court, to be the Attorney General of India.
2. He has the right of audience in all the Courts of the country.
3. He has the right to take part in the proceedings of the Lok Sabha and the Rajya Sabha.
4. He has a fixed tenure.

Select the correct answer from the codes given below :

Codes :

(A) 1 and 4 (B) 2, 3 and 4 (C) 3 and 4 (D) 3 only

42. Match List – I with List – II and select the correct answer from the codes given below :

List – I List – II

(a) Poverty Reduction Programme (i) Mid-day Meals
(b) Human Development Scheme (ii) Indira Awas Yojana (IAY)
(c) Social Assistance Scheme (iii) National Old Assistance Age Pension (NOAP)
(d) Minimum Need Scheme (iv) MNREGA

Codes :

(a) (b) (c) (d)

(A) (iv) (i) (iii) (ii)
(B) (ii) (iii) (iv) (i)
(C) (iii) (iv) (i) (ii)
(D) (iv) (iii) (ii) (i)

40. Which of the following prefix President Pranab Mukherjee desires to be discontinued while interacting with Indian dignitaries as well as in official notings ?

1. His Excellency 2. Mahamahim 3. Hon’ble 4. Shri/Smt.

Select the correct answer from the codes given below :

Codes :

(A) 1 and 3 (B) 2 and 3
(C) 1 and 2 (D) 1, 2 and 3

41. Which of the following can be done under conditions of financial emergency ?
1. State Legislative Assemblies can be abolished.
2. Central Government can acquire control over the budget and expenditure of States.
3. Salaries of the Judges of the High Courts and the Supreme Court can be reduced.
4. Right to Constitutional Remedies can be suspended.

Select the correct answer from the codes given below :

Codes :

(A) 1, 2 and 3 (B) 2, 3 and 4 (C) 1 and 2 (D) 2 and 3

42. Match List – I with List – II and select the correct answer from the codes given below :

List – I List – II

(a) Poverty Reduction Programme (i) Mid-day Meals
(b) Human Development Scheme (ii) Indira Awas Yojana (IAY)
(c) Social Assistance Scheme (iii) National Old Age Pension (NOAP)
(d) Minimum Need Scheme (iv) MNREGA

Codes :

(a) (b) (c) (d)
(A) (iv) (i) (iii) (ii)
(B) (ii) (iii) (iv) (i)
(C) (iii) (iv) (i) (ii)
(D) (iv) (iii) (ii) (i)

2. MC National University of Journalism and Communication is located at
(A) Lucknow (B) Bhopal (c) Chennai (d) Mumbai

37. Indian Institute of Advanced Study is located at

(A) Dharmshala (B) Shimla (C) Solan (D) Chandigarh

38. Indicate the number of Regional Offices of National Council of Teacher Education.

(A) 04 (B) 05 (C) 06 (D) 08

39. Which of the following rights was considered the “Heart and Soul” of the Indian Constitution by Dr. B.R. Ambedkar ?

(A) Freedom of Speech (B) Right to Equality
(C) Right to Freedom of Religion (D) Right to Constitutional Remedies

40. Who among the following created the office of the District Collector in India ?

(A) Lord Cornwallis (B) Warren Hastings
(C) The Royal Commission on Decentralisation (D) Sir Charles Metcalfe

41. The Fundamental Duties of a citizen include

1. Respect for the Constitution, the National Flag and the National Anthem
2. To develop the scientific temper.
3. Respect for the Government.
4. To protect Wildlife.

Choose the correct answer from the codes given below :

Codes :

(A) 1, 2 and 3 (B) 1, 2 and 4
(C) 2, 3 and 4 (D) 1, 3, 4 and 2

42. The President of India takes oath

(A) to uphold the sovereignty and integrity of India.
(B) to bear true faith and allegiance to the Constitution of India.
(C) to uphold the Constitution and Laws of the country.
(D) to preserve, protect and defend the Constitution and the law of the country.

37. The aim of value education to inculcate in students is

(A) the moral values (B) the social values
(C) the political values (D) the economic values

38. Indicate the number of Regional Offices of University Grants Commission of India.

(A) 10 (B) 07 (C) 08 (D) 09

39. One-rupee currency note in India bears the signature of

(A) The President of India (B) Finance Minister of India
(C) Governor, Reserve Bank of India (D) Finance Secretary of Government of India

40. Match the List – I with the List – II and select the correct answer from the codes given below :

List – I List – II

(Commissions and Committees) (Year)

(a) First Administrative Reforms Commission (i) 2005
(b) Paul H. Appleby Committee I (ii) 1962
(c) K. Santhanam Committee (iii) 1966
(d) Second Administrative Reforms Commission (iv) 1953

Codes :

(a) (b) (c) (d)
(A) (i) (iii) (ii) (iv)
(B) (iii) (iv) (ii) (i)
(C) (iv) (ii) (iii) (i)
(D) (ii) (i) (iv) (iii)

41. Constitutionally the registration and recognition of political parties is the function performed by

(A) The State Election Commission of respective States
(B) The Law Ministry of Government of India
(C) The Election Commission of India
(D) Election Department of the State Governments

42. The members of Gram Sabha are

(A) Sarpanch, Upsarpanch and all elected Panchas
(B) Sarpanch, Upsarpanch and Village level worker
(C) Sarpanch, Gram Sevak and elected Panchas
(D) Registered voters of Village Panchayat

41. The first Open University in India was set up in the State of

(A) Andhra Pradesh (B) Delhi (C) Himachal Pradesh (D) Tamil Nadu

42. Most of the Universities in India are funded by

(A) the Central Government (B) the State Governments
(C) the University Grants Commission (D) Private bodies and Individuals

43. Which of the following organizations looks after the quality of Technical and Management education in India ?

(A) NCTE (B) MCI (C) AICTE (D) CSIR

44. Consider the following statements :

Identify the statement which implies natural justice.

(A) The principle of natural justice is followed by the Courts.
(B) Justice delayed is justice denied.
(C) Natural justice is an inalienable right of a citizen
(D) A reasonable opportunity of being heard must be given.

45. The President of India is

(A) the Head of State
(B) the Head of Government
(C) both Head of the State and the Head of the Government
(D) None of the above

46. Who among the following holds office during the pleasure of the President of India ?

(A) Chief Election Commissioner
(B) Comptroller and Auditor General of India
(C) Chairman of the Union Public Service Commission
(D) Governor of a State

41. India’s first Defence University is in the State of

(A) Haryana (B) Andhra Pradesh (C) Uttar Pradesh (D) Punjab

42. Most of the Universities in India

(A) conduct teaching and research only (B) affiliate colleges and conduct examinations
(C) conduct teaching/research and examinations (D) promote research only

43. Which one of the following is not a Constitutional Body ?

(A) Election Commission (B) Finance Commission
(C) Union Public Service Commission (D) Planning Commission

44. Which one of the following statements is not correct ?

(A) Indian Parliament is supreme.
(B) The Supreme Court of India has the power of judicial review.
(C) There is a division of powers between the Centre and the States.
(D) There is a Council of Ministers to aid and advise the President.

45. Which one of the following statements reflects the republic character of Indian democracy ?

(A) Written Constitution
(B) No State religion
(C) Devolution of power to local Government institutions
(D) Elected President and directly or indirectly elected Parliament

46. Who among the following appointed by the Governor can be removed by only the President of India ?
(A) Chief Minister of a State
(B) A member of the State Public Service Commission
(C) Advocate-General
(D) Vice-Chancellor of a State University

41. Which one of the following is the oldest Archival source of data in India ?

(A) National Sample Surveys (B) Agricultural Statistics
(C) Census (D) Vital Statistics

49. The Right to Information Act, 2005 makes the provision of
(A) Dissemination of all types of information by all Public authorities to any person.
(B) Establishment of Central, State and District Level Information Commissions as an appellate body.
(C) Transparency and accountability in Public authorities.
(D) All of the above

56. The accreditation process by National Assessment and Accreditation Council (NAAC) differs from that of National Board of Accreditation (NBA) in terms of

(A) Disciplines covered by both being the same, there is duplication of efforts.
(B) One has institutional grading approach and the other has programme grading approach.
(C) Once get accredited by NBA or NAAC, the institution is free from renewal of grading, which is not a progressive decision.
(D) This accreditation amounts to approval of minimum standards in the quality of education in the institution concerned.

57. Which option is not correct ?

(A) Most of the educational institutions of National repute in scientific and technical sphere fall under 64th entry of Union list.
(B) Education, in general, is the subject of concurrent list since 42nd Constitutional Amendment
Act 1976.
(C) Central Advisory Board on Education (CABE) was first established in 1920.
(D) India had implemented the right to Free and Compulsory Primary Education in 2002 through 86th Constitutional Amendment.

58. Which statement is not correct about the “National Education Day” of India ?

(A) It is celebrated on 5th September every year.
(B) It is celebrated on 11 November every year.
(C) It is celebrated in the memory of India’s first Union Minister of Education, Dr. Abul Kalam Azad.
(D) It is being celebrated since 2008.

59. Match List-I with List-II and select the correct answer from the codes given below :

List – I List – II
(Articles of the Constitution) (Institutions)

(a) Article 280 (i) Administrative Tribunals
(b) Article 324 (ii) Election Commission of India
(c) Article 323 (iii) Finance Commission at Union level
(d) Article 315 (iv) Union Public Service Commission

Codes :
(a) (b) (c) (d)
(A) (i) (ii) (iii) (iv)
(B) (iii) (ii) (i) (iv)
(C) (ii) (iii) (iv) (i)
(D) (ii) (iv) (iii) (i)

60. Deemed Universities declared by UGC under Section 3 of the UGC Act 1956, are not permitted to

(A) offer programmes in higher education and issue degrees.
(B) give affiliation to any institute of higher education.
(C) open off-campus and off-shore campus anywhere in the country and overseas respectively without the permission of the UGC.
(D) offer distance education programmes without the approval of the Distance Education Council.

1. The University which telecasts interaction educational programmes through its own channel is
(A) Osmania University
(B) University of Pune
(C) Annamalai University
(D) Indira Gandhi National Open University (IGNOU)

3. Who has signed an MOU for Accreditation of Teacher Education Institutions in India ?

(A) NAAC and UGC (B) NCTE and NAAC
(C) UGC and NCTE (D) NCTE and IGNOU

41. Which of the following sources of data is not based on primary data collection ?
(A) Census of India (B) National Sample Survey
(C) Statistical Abstracts of India (D) National Family Health Survey

55. The recommendation of National Knowledge Commission for the establishment of 1500 Universities is to

(A) create more teaching jobs
(B) ensure increase in student enrolment in higher education
(C) replace or substitute the privately managed higher education institutions by public institutions
(D) enable increased movement of students from rural areas to urban areas

56. According to Article 120 of the Constitution of India, the business in Parliament shall be transacted in

(A) English only
(B) Hindi only
(C) English and Hindi both
(D) All the languages included in Eighth Schedule of the Constitution

57. The Parliament in India is composed of

(A) Lok Sabha & Rajya Sabha
(B) Lok Sabha, Rajya Sabha & Vice President
(C) Lok Sabha, Rajya Sabha & President
(D) Lok Sabha, Rajya Sabha with their Secretariats

58. The Parliament in India is composed of

(A) Lok Sabha & Rajya Sabha
(B) Lok Sabha, Rajya Sabha & Vice President
(C) Lok Sabha, Rajya Sabha & President
(D) Lok Sabha, Rajya Sabha with their Secretariats

59. The enrolment in higher education in India is contributed both by Formal System of Education and by System of Distance Education. Distance education contributes

(A) 50% of formal system
(B) (B) 25% of formal system
(C) 10% of the formal system
(D) Distance education system’s contribution is not taken into account while considering the figures of enrolment in higher education

60. Assertion (A) : The U.G.C. Academic Staff Colleges came into existence to improve
the quality of teachers.
Reason (R) : University and college teachers have to undergo both orientation and refresher courses.

(A) Both (A) and (R) are true and (R) is the correct explanation.
(B) Both (A) and (R) are correct but (R) is not the correct explanation of (A).
(C) (A) is correct and (R) is false.
(D) (A) is false and (R) is correct.

38. Chandrayan I was launched on 22nd October, 2008 in India from :

(A) Bangalore (B) Sri Harikota (C) Chennai (D) Ahmedabad

46. The Kothari Commission’s report was entitled on :

(A) Education and National Development (B) Learning to be adventure
(C) Diversification of Education (D) Education and socialization in democracy

47. Which of the following is not a Dual mode University ?

(A) Delhi University (B) Bangalore University
(C) Madras University (D) Indira Gandhi National Open University

48. Which part of the Constitution of India is known as “Code of Administrators” ?

(A) Part I (B) Part II (C) Part III (D) Part IV

49. Which article of the constitution provides safeguards to Naga Customary and their social
practices against any act of Parliament ?

(A) Article 371 A (B) Article 371 B (C) Article 371 C (D) Article 263

50. Which one of the following is not the tool of good governance ?

(A) Right to information (B) Citizens’ Charter
(C) Social Auditing (D) Judicial Activism

January 2017

July 2016

December 2015

June 2015

December 2014

June 2014

December 2013

September 2013

June 2013

December 2012

June 2012

December 2011

June 2011

December 2010

June 2010

December 2009

June 2009

error: Content is protected !!